Endocrine

Ace your homework & exams now with Quizwiz!

Which sign/symptom is one of the first indicators of hyperthyroidism that is often noticed by the patient? a. Eyelid or globe lag b. Vision changes or tiring of the eyes c. Protruding eyes d. Photophobia

b

37. Which statement about pheochromocytoma is correct? a. It is most often malignant. b. It is a catecholamine-producing tumor. c. It is found only in the adrenal medulla. d. It is manifested by hypotension.

b

The patient has multiple thyroid nodules resulting in thyroid hyperfunction. What is the most likely cause of this hyperthyroidism? a. Thyroid carcinoma b. Graves' disease c. Toxic multinodular goiter d. Pituitary hyperthyroidism

c

45. What is the most common cause of endogenous hypercortisolism, or Cushing's disease? a. Pituitary hypoplasia b. Insufficient ACTH production c. Adrenocortical hormone deficiency d. Hyperplasia of the adrenal cortex

d

31. Which disease involves a disorder of the islets of Langerhans? a. Diabetes insipidus b. Diabetes mellitus c. Addison's disease d. Cushing's disease

b Diabetes mellitus

55. What is the most common cause of death from myxedema coma? a. Myocardial infarction b. Acute kidney failure c. High serum level of iodide d. Respiration failure

d

73. The client is diagnosed with hypothyroidism. Which signs/symptoms should the nurse expect the client to exhibit? 1. Complaints of extreme fatigue and hair loss. 2. Exophthalmos and complaints of nervousness. 3. Complaints of profuse sweating and flushed skin. 4. Tetany and complaints of stiffness of the hands.

1. A decrease in thyroid hormone causes decreased metabolism, which leads to fatigue and hair loss. TEST-TAKING HINT: Often if the test taker does not know the specific signs/symptoms of the disease but knows the function of the system affected by the disease, some possible answers can be ruled out. Tetany and stiffness of the hands are related to calcium, the level of which is influenced by the parathyroid gland, not the thyroid gland; therefore, option "4" can be ruled out.

75. The client diagnosed with hypothyroidism is prescribed the thyroid hormone levothyroxine (Synthroid). Which assessment data indicate the medication has been effective? 1. The client has a three (3)-pound weight gain. 2. The client has a decreased pulse rate. 3. The client's temperature is WNL. 4. The client denies any diaphoresis.

3. The client with hypothyroidism frequently has a subnormal temperature, so a temperature WNL indicates the medication is effective. TEST-TAKING HINT: One way of determining the effectiveness of medication is to determine if the signs/symptoms of the disease are no longer noticeable.

79. Which statement made by the client makes the nurse suspect the client is experiencing hyperthyroidism? 1. "I just don't seem to have any appetite anymore." 2. "I have a bowel movement about every three (3) to four (4) days." 3. "My skin is really becoming dry and coarse." 4. "I have noticed all my collars are getting tighter."

4. The thyroid gland (in the neck) enlarges as a result of the increased need for thyroid hormone production; an enlarged gland is called a goiter. TEST-TAKING HINT: If the test taker does not know the answer, sometimes thinking about the location of the gland or organ causing the problem may help the test taker select or rule out specific options.

An emergency nurse cares for a client who is experiencing an acute adrenal crisis. Which action should the nurse take first? a. Obtain intravenous access. b. Administer hydrocortisone succinate (Solu-Cortef). c. Assess blood glucose. d. Administer insulin and dextrose.

A (All actions are appropriate for the client with adrenal crisis. However, therapy is given intravenously, so the priority is to establish IV access. Solu-Cortef is the drug of choice. Blood glucose is monitored hourly and treatment is provided as needed. Insulin and dextrose are used to treat any hyperkalemia.)

13. While assessing a client with Graves' disease, the nurse notes that the client's temperature has risen 1° F. Which action should the nurse take first? a. Turn the lights down and shut the client's door. b. Call for an immediate electrocardiogram (ECG). c. Calculate the client's apical-radial pulse deficit. d. Administer a dose of acetaminophen (Tylenol).

A. A temperature increase of 1° F may indicate the development of thyroid storm, and the provider needs to be notified. But before notifying the provider, the nurse should take measures to reduce environmental stimuli that increase the risk of cardiac complications. The nurse can then call for an ECG. The apical-radial pulse deficit would not be necessary, and Tylenol is not needed because the temperature increase is due to thyroid activity.

The nurse is preparing the room for the client returning from a thyroidectomy. Which items are important for the nurse to have available for this client? (Select all that apply.) A. Calcium gluconate B. Emergency tracheotomy kit C. Furosemide (Lasix) D. Hypertonic saline E. Oxygen F. Suction

A. B. E. F. Calcium gluconate should be available at the bedside to treat hypocalcemia and tetany that might occur if the parathyroid glands have been injured during the surgery. Equipment for an emergency tracheotomy must be kept at the bedside in the event that hemorrhage or edema should occlude the airway. Respiratory distress can result from swelling or damage to the laryngeal nerve leading to spasm, so it is important that the nurse work with respiratory therapy to have oxygen ready at the bedside for the client on admission. Because of the potential for increased secretions, it is important that a working suction device is present at the bedside for admission of the client from the operating room. Furosemide might be useful in the postoperative client to assist with urine output; however, this is not of added importance for this client. Hypertonic saline would not be of benefit to this client as the client is not hyponatremic.

A nurse cares for a client with a deficiency of aldosterone. Which assessment finding should the nurse correlate with this deficiency? a. Increased urine output b. Vasoconstriction c. Blood glucose of 98 mg/dL d. Serum sodium of 144 mEq/L

ANS: A Aldosterone, the major mineralocorticoid, maintains extracellular fluid volume. It promotes sodium and water reabsorption and potassium excretion in the kidney tubules. A client with an aldosterone deficiency will have increased urine output. Vasoconstriction is not related. These sodium and glucose levels are normal; in aldosterone deficiency, the client would have hyponatremia and hyperkalemia.

A nurse prepares to administer prescribed regular and NPH insulin. Place the nurse's actions in the correct order to administer these medications. 1. Inspect bottles for expiration dates. 2. Gently roll the bottle of NPH between the hands. 3. Wash your hands. 4. Inject air into the regular insulin. 5. Withdraw the NPH insulin. 6. Withdraw the regular insulin. 7. Inject air into the NPH bottle. 8. Clean rubber stoppers with an alcohol swab.

ANS: 3, 1, 2, 8, 7, 4, 6, 5 After washing hands, it is important to inspect the bottles and then to roll the NPH to mix the insulin. Rubber stoppers should be cleaned with alcohol after rolling the NPH and before sticking a needle into either bottle. It is important to inject air into the NPH bottle before placing the needle in a regular insulin bottle to avoid mixing of regular and NPH insulin. The shorter-acting insulin is always drawn up first.

Which assessment findings indicate hyperthyroidism? (Select all that apply.) a. Weight loss with increased appetite b. Constipation c. Increased heart rate d. Insomnia e. Decreased libido f. Heat intolerance

a, c, d, f

A nurse teaches a client with type 2 diabetes mellitus who is prescribed glipizide (Glucotrol). Which statement should the nurse include in this client's teaching? a. "Change positions slowly when you get out of bed." b. "Avoid taking nonsteroidal anti-inflammatory drugs (NSAIDs)." c. "If you miss a dose of this drug, you can double the next dose." d. "Discontinue the medication if you develop a urinary infection."

ANS: B NSAIDs potentiate the hypoglycemic effects of sulfonylurea agents. Glipizide is a sulfonylurea. The other statements are not applicable to glipizide.

A nurse cares for a client with excessive production of thyrocalcitonin (calcitonin). For which electrolyte imbalance should the nurse assess? a. Potassium b. Sodium c. Calcium d. Magnesium

ANS: C Parafollicular cells produce thyrocalcitonin (calcitonin), which regulates serum calcium levels. Calcitonin has no impact on potassium, sodium, or magnesium balances.

A nurse teaches a client with diabetes mellitus who is experiencing numbness and reduced sensation. Which statement should the nurse include in this client's teaching to prevent injury? a. "Examine your feet using a mirror every day." b. "Rotate your insulin injection sites every week." c. "Check your blood glucose level before each meal." d. "Use a bath thermometer to test the water temperature."

ANS: D Clients with diminished sensory perception can easily experience a burn injury when bathwater is too hot. Instead of checking the temperature of the water by feeling it, they should use a thermometer. Examining the feet daily does not prevent injury, although daily foot examinations are important to find problems so they can be addressed. Rotating insulin and checking blood glucose levels will not prevent injury.

After teaching a client who is recovering from an endoscopic trans-nasal hypophysectomy, the nurse assesses the clients understanding. Which statement made by the client indicates a correct understanding of the teaching? a. I will wear dark glasses to prevent sun exposure. b. Ill keep food on upper shelves so I do not have to bend over. c. I must wash the incision with peroxide and redress it daily. d. I shall cough and deep breathe every 2 hours while I am awake.

B (After this surgery, the client must take care to avoid activities that can increase intracranial pressure. The client should avoid bending from the waist and should not bear down, cough, or lie flat. With this approach, there is no incision to clean and dress. Protection from sun exposure is not necessary after this procedure.)

3. A malfunctioning anterior pituitary gland can result in which disorder? (Select all that apply.) a. Pituitary hypofunction b. Pituitary hyperfunction c. DI d. Hypothyroidism e. Osteoporosis

a, b, d, e Pituitary hypofunction Pituitary hyperfunction Hypothyroidism Osteoporosis

A nurse cares for a client with adrenal hyperfunction. The client screams at her husband, bursts into tears, and throws her water pitcher against the wall. She then tells the nurse, I feel like I am going crazy. How should the nurse respond? a. I will ask your doctor to order a psychiatric consult for you. b. You feel this way because of your hormone levels. c. Can I bring you information about support groups? d. I will close the door to your room and restrict visitors.

B (Hypercortisolism can cause the client to show neurotic or psychotic behavior. The client needs to know that these behavior changes do not reflect a true psychiatric disorder and will resolve when therapy results in lower and steadier blood cortisol levels. The client needs to understand this effect and does not need a psychiatrist, support groups, or restricted visitors at this time.)

A nurse cares for a client who is prescribed vasopressin (DDAVP) for diabetes insipidus. Which assessment findings indicate a therapeutic response to this therapy? (Select all that apply.) a. Urine output is increased. b. Urine output is decreased. c. Specific gravity is increased. d. Specific gravity is decreased. e. Urine osmolality is increased. f. Urine osmolality is decreased.

B, D, E (Diabetes insipidus causes urine output to be greatly increased, with a low urine osmolality, as evidenced by a low specific gravity. Effective treatment results in decreased urine output that is more concentrated, as evidenced by an increased specific gravity.)

A nurse teaches a client with Cushings disease. Which dietary requirements should the nurse include in this clients teaching? (Select all that apply.) a. Low calcium b. Low carbohydrate c. Low protein d. Low calories e. Low sodium

B, D, E (The client with Cushings disease has weight gain, muscle loss, hyperglycemia, and sodium retention. Dietary modifications need to include reduction of carbohydrates and total calories to prevent or reduce the degree of hyperglycemia. Sodium retention causes water retention and hypertension. Clients are encouraged to restrict their sodium intake moderately. Clients often have bone density loss and need more calcium. Increased protein intake will help decrease muscle loss.)

14. After teaching a client who is recovering from a complete thyroidectomy, the nurse assesses the client's understanding. Which statement made by the client indicates a need for additional instruction? a. "I may need calcium replacement after surgery." b. "After surgery, I won't need to take thyroid medication." c. "I'll need to take thyroid hormones for the rest of my life." d. "I can receive pain medication if I feel that I need it."

B. After the client undergoes a thyroidectomy, the client must be given thyroid replacement medication for life. He or she may also need calcium if the parathyroid is damaged during surgery, and can receive pain medication postoperatively.

12. A nurse cares for a client newly diagnosed with Graves' disease. The client's mother asks, "I have diabetes mellitus. Am I responsible for my daughter's disease?" How should the nurse respond? a. "The fact that you have diabetes did not cause your daughter to have Graves' disease. No connection is known between Graves' disease and diabetes." b. "An association has been noted between Graves' disease and diabetes, but the fact that you have diabetes did not cause your daughter to have Graves' disease." c. "Graves' disease is associated with autoimmune diseases such as rheumatoid arthritis, but not with a disease such as diabetes mellitus." d. "Unfortunately, Graves' disease is associated with diabetes, and your diabetes could have led to your daughter having Graves' disease."

B. An association between autoimmune diseases such as rheumatoid arthritis and diabetes mellitus has been noted. The predisposition is probably polygenic, and the mother's diabetes did not cause her daughter's Graves' disease. The other statements are inaccurate.

41. Which are the types of radiographic tests that may be used for an endocrine assessment? (Select all that apply.) a. Ultrasonography b. Skull x-ray c. Chest x-ray d. Magnetic resonance imaging (MRI) e. Computed tomography (CT)

a, b, d, e Ultrasonography, Skull x-ray, Magnetic resonance imaging (MRI), Computed tomography (CT)

A nurse assesses clients who are at risk for diabetes mellitus. Which client is at greatest risk? a. A 29-year-old Caucasian b. A 32-year-old African-American c. A 44-year-old Asian d. A 48-year-old American Indian

Diabetes is a particular problem among African Americans, Hispanics, and American Indians. The incidence of diabetes increases in all races and ethnic groups with age. Being both an American Indian and middle-aged places this client at highest risk.

1. A nurse assesses a client with hyperthyroidism who is prescribed lithium carbonate. Which assessment finding should alert the nurse to a side effect of this therapy? a. Blurred and double vision b. Increased thirst and urination c. Profuse nausea and diarrhea d. Decreased attention and insomnia

B. Lithium antagonizes antidiuretic hormone and can cause symptoms of diabetes insipidus. This manifests with increased thirst and urination. Lithium has no effect on vision, gastric upset, or level of consciousness.

Which type of thyroid cancer often occurs as part of multiple endocrine neoplasia (MEN) type II? A. Anaplastic B. Follicular C. Medullary D. Papillary

C. Medullary carcinoma commonly occurs as part of MEN type II, which is a familial endocrine disorder. Anaplastic carcinoma is an aggressive tumor that invades surrounding tissue. Follicular carcinoma occurs more frequently in older clients and may metastasize to bone and lung. Papillary carcinoma is the most common type of thyroid cancer. It is slow growing and, if the tumor is confined to the thyroid gland, the outlook for a cure is good with surgical management.

A nurse plans care for a client with Cushings disease. Which action should the nurse include in this clients plan of care to prevent injury? a. Pad the siderails of the clients bed. b. Assist the client to change positions slowly. c. Use a lift sheet to change the clients position. d. Keep suctioning equipment at the clients bedside.

C (Cushings syndrome or disease greatly increases the serum levels of cortisol, which contributes to excessive bone demineralization and increases the risk for pathologic bone fracture. Padding the siderails and assisting the client to change position may be effective, but these measures will not protect him or her as much as using a lift sheet. The client should not require suctioning.)

A nurse cares for a client who is recovering from a hypophysectomy. Which action should the nurse take first? a. Keep the head of the bed flat and the client supine. b. Instruct the client to cough, turn, and deep breathe. c. Report clear or light yellow drainage from the nose. d. Apply petroleum jelly to lips to avoid dryness.

C (A light yellow drainage or a halo effect on the dressing is indicative of a cerebrospinal fluid leak. The client should have the head of the bed elevated after surgery. Although deep breathing is important postoperatively, coughing should be avoided to prevent cerebrospinal fluid leakage. Although application of petroleum jelly to the lips will help with dryness, this instruction is not as important as reporting the yellowish drainage.)

A nurse plans care for a client with a growth hormone deficiency. Which action should the nurse include in this clients plan of care? a. Avoid intramuscular medications. b. Place the client in protective isolation. c. Use a lift sheet to re-position the client. d. Assist the client to dangle before rising.

C (In adults, growth hormone is necessary to maintain bone density and strength. Adults with growth hormone deficiency have thin, fragile bones. Avoiding IM medications, using protective isolation, and assisting the client as he or she moves from sitting to standing will not serve as safety measures when the client is deficient in growth hormone.)

A nurse assesses a client who is recovering from a transsphenoidal hypophysectomy. The nurse notes nuchal rigidity. Which action should the nurse take first? a. Encourage range-of-motion exercises. b. Document the finding and monitor the client. c. Take vital signs, including temperature. d. Assess pain and administer pain medication.

C (Nuchal rigidity is a major manifestation of meningitis, a potential postoperative complication associated with this surgery. Meningitis is an infection; usually the client will also have a fever and tachycardia. Range-of-motion exercises are inappropriate because meningitis is a possibility. Documentation should be done after all assessments are completed and should not be the only action. Although pain medication may be a palliative measure, it is not the most appropriate initial action.)

The nurse is providing discharge instructions to a client on spironolactone (Aldactone) therapy. Which comment by the client indicates a need for further teaching? A. "I must call the provider if I am more tired than usual." B. "I need to increase my salt intake." C. "I should eat a banana every day." D. "This drug will not control my heart rate."

C. Spironolactone increases potassium levels, so potassium supplements and foods rich in potassium, such as bananas, should be avoided to prevent hyperkalemia. While taking spironolactone, symptoms of hyponatremia such as drowsiness and lethargy must be reported; the client may need increased dietary sodium. Spironolactone will not have an effect on the client's heart rate.

Ch.64 p. 1335, Prioritization, Delegation, and Supervision The patient, a 21-year-old college student, was brought to the emergency department (ED) by his roommate. He reports abdominal pain, polyuria for the past 2 days, vomiting several times prior to arrival, and extreme thirst. He appears flushed, and his lips and mucous membranes are dry and cracked. His skin turgor is poor. He demonstrates deep rapid respirations; there is a fruit odor to his breath. He has type 1 diabetes and "may have skipped a few doses of insulin because of cramming for final exams." He is alert and talking but is having trouble focusing on your questions. Vital signs: Blood pressure 110/60 Pulse 110/min Respirations 32/min Temperature 100.8° F Fingerstick glucose 485 mg/dL Oxygen saturation 99% 1. You have completed triage assessment and history, should you now notify his parents for permission to treat him? Why or why not?

Calling the parents is not necessary because as the patient is old enough to sign consent for himself. (If he were under age, the treatment would not be delayed if the parents were unavailable in an emergency situation.) Calling the primary health care provider is usually done by the ED physician after the preliminary workup is completed. (Policies for calling private physicians may vary among institutions. Be sure to check the policy at your facility.)

11. A nurse cares for a client who is recovering from a parathyroidectomy. When taking the client's blood pressure, the nurse notes that the client's hand has gone into flexion contractions. Which laboratory result does the nurse correlate with this condition? a. Serum potassium: 2.9 mEq/L b. Serum magnesium: 1.7 mEq/L c. Serum sodium: 122 mEq/L d. Serum calcium: 6.9 mg/dL

D. Hypocalcemia destabilizes excitable membranes and can lead to muscle twitches, spasms, and tetany. This effect of hypocalcemia is enhanced in the presence of tissue hypoxia. The flexion contractions (Trousseau's sign) that occur during blood pressure measurement are indicative of hypocalcemia, not the other electrolyte imbalances, which include hypokalemia, hyponatremia, and hypomagnesemia.

2. A nurse assesses a client who is recovering from a total thyroidectomy and notes the development of stridor. Which action should the nurse take first? a. Reassure the client that the voice change is temporary. b. Document the finding and assess the client hourly. c. Place the client in high-Fowler's position and apply oxygen. d. Contact the provider and prepare for intubation.

D. Stridor on exhalation is a hallmark of respiratory distress, usually caused by obstruction resulting from edema. One emergency measure is to remove the surgical clips to relieve the pressure. This might be a physician function. The nurse should prepare to assist with emergency intubation or tracheostomy while notifying the provider or the Rapid Response Team. Stridor is an emergency situation; therefore, reassuring the client, documenting, and reassessing in an hour do not address the urgency of the situation. Oxygen should be applied, but this action will not keep the airway open.

Ch.64 p. 1332, Safety; Quality Improvement; Teamwork and Collaboration The patient is a 60 year-old-woman who is 1 day postoperative after a total knee replacement. She has type 2 diabetes and just recently was switched from oral antidiabetic drugs to an insulin regimen. She let her nurse know that her on-demand lunch has been ordered. The nurse tests her blood and gives her the prescribed short-acting insulin dose. An hour later, the physical therapist finds her pale, confused, and clammy. Her lunch tray is on her table and appears totally untouched. 1. Is her condition consistent with hyperglycemia or hypoglycemia? Explain your choice.

Her condition is consistent with hypoglycemia, especially because she received insulin about an hour ago. Manifestations of hypoglycemia include weakness; difficulty thinking; confusion; sweating; and cool, pale skin, and manifestations of hyperglycemia include warm, moist skin and possible fruity breath odor. Hyperglycemia does not change level of consciousness until it is severe.

34. The nurse is preparing for a patient to return from thyroid surgery. What priority equipment does the nurse ensure is immediately available? (Select all that apply.) a. Tracheostomy equipment b. Calcium gluconate or calcium chloride for IV administration c. Mechanical ventilator d. Humidified oxygten e. Suction equipment f. Pillow

a, b, d, e, f

The nurse is preparing for a patient to return from thyroid surgery. What priority equipment does the nurse ensure is immediately available? (Select all that apply.) a. Tracheostomy equipment b. Calcium gluconate or calcium chloride for IV administration c. Mechanical ventilator d. Humidified oxygen e. Suction equipment f. Pillows

a, b, d, e, f

31. Which signs and symptoms are assessment findings indicative of thyroid storm? (Select all that apply.) a. Abdominal pain and nausea b. Hypothermia c. Elevated temperature d. Tachycardia e. Elevated systolic blood pressure f. Bradycardia

a, c, d, e

Ch.64 p. 1335, Prioritization, Delegation, and Supervision The patient, a 21-year-old college student, was brought to the emergency department (ED) by his roommate. He reports abdominal pain, polyuria for the past 2 days, vomiting several times prior to arrival, and extreme thirst. He appears flushed, and his lips and mucous membranes are dry and cracked. His skin turgor is poor. He demonstrates deep rapid respirations; there is a fruit odor to his breath. He has type 1 diabetes and "may have skipped a few doses of insulin because of cramming for final exams." He is alert and talking but is having trouble focusing on your questions. Vital signs: Blood pressure 110/60 Pulse 110/min Respirations 32/min Temperature 100.8° F Fingerstick glucose 485 mg/dL Oxygen saturation 99% 2. Should you apply oxygen at this time? Why or why not?

Oxygen is not needed. Although his respiratory rate is above normal, he is not hypoxemic. Applying oxygen would serve no useful purpose.

Ch.64 p. 1335, Prioritization, Delegation, and Supervision The patient, a 21-year-old college student, was brought to the emergency department (ED) by his roommate. He reports abdominal pain, polyuria for the past 2 days, vomiting several times prior to arrival, and extreme thirst. He appears flushed, and his lips and mucous membranes are dry and cracked. His skin turgor is poor. He demonstrates deep rapid respirations; there is a fruit odor to his breath. He has type 1 diabetes and "may have skipped a few doses of insulin because of cramming for final exams." He is alert and talking but is having trouble focusing on your questions. Vital signs: Blood pressure 110/60 Pulse 110/min Respirations 32/min Temperature 100.8° F Fingerstick glucose 485 mg/dL Oxygen saturation 99% 5. In caring for this patient, what immediate intervention do you anticipate the ED physician will order to be performed first? Provide a rationale for your choice.

The patient needs IV fluids to correct fluid deficit that places him at risk for hypovolemic shock. Also, he needs carefully regulated insulin therapy at this time, which is best accomplished by the IV route. Subcutaneous insulin does not absorb fast enough and is inappropriate for emergency situations.

44. Bone changes in the older adult are often seen with endocrine dysfunction and increased secretion of which substance? a. PTH b. Calcitonin c. Insulin d. Testosterone

a

Ch.64 p. 1329, Patient-Centered Care During a clinic visit, you are reviewing the records of a 39-year-old patient who was diagnosed 5 years ago with type 2 diabetes. You discover that, although he has always been extremely near-sighted, he has not seen an ophthalmologist for 4 years. He has gained 12 lbs since his last visit a year ago. His laboratory values show a fasting blood glucose level of 96 mg/dL, an A1C of 8.2%, a total cholesterol of 322 mg/dL, and an LDL of 190 mg/dL. When you ask him about ophthalmology follow-up and point out his laboratory values, he replies that because he is taking prescribed antidiabetic medication, he believes that he won't have all the diabetes complications that his father had. He further tells you that he did have his eyes checked by an optometrist to make sure his prescription was accurate but that because he was younger than 40 years old, he does not need intraocular pressure measurements. 2. Should you address his weight gain? Why or why not?

Yes, you should address his weight. A major pathophysiological problem with type 2 diabetes is insulin resistance. Increasing weight correlates to greater insulin resistance. Even modest weight loss can improve the sensitivity of insulin receptors to insulin. The excess weight is contributing even more to his risk for cardiovascular events, as evidenced by the high blood lipid levels.

48. The nurse determines a priority patient problem of altered self-concept in a female patient with Cushing's syndrome who expresses concern about the changes in her general appearance. What is the expected outcome for this patient? a. To verbalize an understanding that treatment will reverse many of the problems b. To ventilate about the frustration of these lifelong physical changes c. To verbalize ways to cope with the changes such as joining a support group or changing style of dress d. To achieve a personal desired level of sexual functioning

a

49. Which drug is an adrenal cytotoxic agent used for inoperable adrenal tumors? a. Mitotane (Lysodren) b. Aminoglutethimide (Cytadren) c. Cyprohepatadine (Periactin) d. Fludrocortisone (Florinef)

a

50. A patient has hyperparathyroidism and high levels of serum calcium. Which initial treatment does the nurse prepare to administer to the patient? a. Furosemide (Lasix) with IV saline b. Calcitonin c. Oral phosphates d. Mmithramycin

a

54. Which patient is at risk for developing secondary adrenal insufficiency? a. Patient who suddenly stops taking high-dose steroid therapy b. Patient who tapers the dosages of steroid therapy c. Patient deficient in ADH d. Patient with an adrenal tumor causing excessive secretion of ACTH

a

9. The binding of a hormone to a specific receptor site is an example of which endocrine process? a. "Lock and key" manner b. Negative feedback mechanism c. Neuroendocrine regulation d. "Fight-or-flight" response

a "Lock and key" manner

11. Which hormone is directly suppressed when circulating levels of cortisol are above normal? a. Corticotropin-releasing hormone (CRH) b. ADH c. Adrenocorticotropic hormone (ACTH) d. Growth hormone-releasing hormone (GH-RH)

a Corticotropin-releasing hormone (CRH)

9. When analyzing laboratory values, the nurse expects to find which value as a direct result of overproduction of GH? a. Hyperglycemia b. Hyperphosphatemia c. Hypocalcemia d. Hypercalcemia

a Hyperglycemia

20. A patient who has been diagnosed with Graves' disease is to receive RAI in the oral form of 131 I as a treatment. What instructions does the nurse include in the teaching plan about preventing radiation exposure to others? (Select all that apply.) a. Do not share a toilet with others for 2 weeks after treatment. b. Flush the toilet three times after each use. c. Wash clothing separately from others in the household. d. Limit contact with pregnant women, infants, and children. e. Do not use a laxative within 12 weeks of having the treatment.

a, b, c, d

15. Which statements about hyperthyroidism are accurate? (Select all that apply.) a. It is most commonly caused by Graves' disease . b. It can be caused by overuse of thyroid replacement medication. c. It occurs more often in men between the ages of 20-40. d. Weight gain is a common manifestation. e. Serum T3 and T4 results will be elevated.

a, b, e

Which statements about hyperthyroidism are accurate? (Select all that apply.) a. It is most commonly caused by Graves' disease. b. It can be caused by overuse of thyroid replacement medication. c. It occurs more often in men between the ages of 20- 40. d. Weight gain is a common manifestation. e. Serum T3 and T4 results will be elevated.

a, b, e

35. In addition to IV fluids, a patient with SIADH is on a fluid restriction as low as 500 to 600 mL/24 hours. Which serum and urine results demonstrate effectiveness of this treatment? (Select all that apply.) a. Decreased urine specific gravity b. Decreased serum sodium c. Increased urine output d. Increased urine specific gravity e. Increased serum sodium f. Decreased urine output

a, c, e

10. What is the most common cause of hyperthyroidism? a. Radiation to thyroid b. Graves' disease c. Thyroid cancer d. Thyroiditis

b

12. The nurse is assessing a patient diagnosed with hyperthyroidism and observes dry, waxy swelling of the front surfaces of the lower legs. How does the nurse interpret this finding? a. Globe lag b. Pretibial myxedema c. Exophthalmos d. Goiter

b

17. The nurse is providing instructions to a patient who is taking the anti-thyroid medication propylthiouracil (PTU). The nurse instructs the patient to notify the health care provider immediately if which sign/symptom occurs? a. Weight gain b. Dark-colored urine c. Cold intolerance d. Headache

b

19. A patient who has been diagnosed with Graves' disease is going to receive radioactive iodine (RAI) in the oral form of 131 I. What does the nurse teach the patient about how this drug works? a. It destroys the hormones T3 and T4. b. It destroys the tissue that produces thyroid hormones. c. It blocks thyroid hormone production. d. It prevents T4 from being converted to T3.

b

19. Preoperative care for a patient who has had a transphenoidal hypophysectomy includes which intervention? a. Encouraging coughing and deep-breathing to decrease pulmonary complications b. Testing nasal drainage for glucose to determine whether it contains CSF c. Keeping the bed flat to decrease central CSF leakage d. Assisting the patient with brushing the teeth to reduce risk of infection

b

38. After hospitalization for myxedema, a patient is prescribed thyroid replacement medication. Which statement by the patient demonstrates a correct understanding of this therapy? a. "I'll be taking this medication until my symptoms are completely resolved." b. "I'll be taking thyroid medication for the rest of my life." c. "Now that I'm feeling better, no changes in my medication will be necessary." d. "I'm taking this medication to prevent symptoms of an overactive thyroid gland."

b

41. What is the hallmark of thyroid cancer? a. Aggressive tumors b. Elevated serum thyroglobulin level c. Metastasis to other organs d. Invasion of blood vessels

b

The laboratory results for a 53-year-old patient indicate a low T3 level and elevated thyroidstimulating hormone (TSH). What do these results indicate? a. Hyperthyroidism b. Hypothyroidism c. Malfunctioning pituitary gland d. Normal laboratory values for this age

b

What is the most common cause of hyperthyroidism? a. Radiation to thyroid b. Graves' disease c. Thyroid cancer d. Thyroiditis

b

12. The maintenance of internal body temperature at approximately 98.6F (37C) is an example of which endocrine process? a. "Lock and key" manner b. Neuroendocrine regulation c. Positive feedback mechanism d. Stimulus-response theory

b Nueroendocrine regulation

25. What does the nurse instruct patients with permanent DI to do? (Select all that apply.) a. Continue vasopressin therapy until symptoms disappear. b. Monitor for recurrent of polydipsia and polyuria. c. Monitor and record weight daily. d. Check urine specific gravity three times a week. e. Wear a medical alert bracelet.

b, c, e

47. Which laboratory findings does the nurse expect to find with Cushing's syndrome? (Select all that apply.) a. Decreased serum sodium b. Increased serum glucose c. Increased serum sodium d. Increased serum potassium e. Decreased serum calcium

b, c, e

5. A nurse cares for a client who presents with bradycardia secondary to hypothyroidism. Which medication should the nurse anticipate being prescribed to the client? a. Atropine sulfate b. Levothyroxine sodium (Synthroid) c. Propranolol (Inderal) d. Epinephrine (Adrenalin)

b. The treatment for bradycardia from hypothyroidism is to treat the hypothyroidism using levothyroxine sodium. If the heart rate were so slow that it became an emergency, then atropine or epinephrine might be an option for short-term management. Propranolol is a beta blocker and would be contraindicated for a client with bradycardia.

13. Which statement best describes globe lag in a patient with hyperthyroidism? a. Abnormal protrusion of the eyes b. Upper eyelid fails to descend when the patient gazes downward c. Upper eyelid pulls back faster than the eyeball when the patient gazes upward d. Inability of both eyes to focus on an object simultaneously

c

Which factor is a hallmark assessment finding that signifies hyperthyroidism? a. Weight loss b. Increased libido c. Heat intolerance d. Diarrhea

c

Which statement best describes globe lag in a patient with hyperthyroidism? a. Abnormal protrusion of the eyes b. Upper eyelid fails to descend when the patient gazes downward c. Upper eyelid pulls back faster than the eyeball when the patient gazes upward d. Inability of both eyes to focus on an object simultaneously

c

4. The assessment findings of a male patient with anterior pituitary tumor include reports of changes in secondary sex characteristics, such as episodes of impotence and decreased libido. The nurse explains to the patient that these findings are a result of overproduction of which hormone? a. Gonadotropins inhibiting prolactin (PRL) b. Thyroid hormone inhibiting PRL c. PRL inhibiting secretion of gonadotropins d. Steroids inhibiting production of sex hormones

c PRL inhibiting secretion of gonadotropins

40. Which instructions are included when teaching a patient about urine collection for endocrine studies? (Select all that apply.) a. Fast before starting the urine collection. b. Measure the urine in mL rather than ounces. c. Empty the bladder completely, and then start timing. d. Time the test for exactly the instructed number of hours. e. Avoid taking any unnecessary drugs during endocrine testing. f. Empty the bladder at the end of the time period and keep that specimen.

c, d, e, f Empty the bladder completely, and then start timing. Time the test for exactly the instructed number of hours. Avoid taking any unnecessary drugs during endocrine testing. Empty the bladder at the end of the time period and keep that specimen.

3. A nurse assesses a client who is recovering from a subtotal thyroidectomy. On the second postoperative day the client states, "I feel numbness and tingling around my mouth." What action should the nurse take? a. Offer mouth care. b. Loosen the dressing. c. Assess for Chvostek's sign. d. Ask the client orientation questions.

c. Numbness and tingling around the mouth or in the fingers and toes are manifestations of hypocalcemia, which could progress to cause tetany and seizure activity. The nurse should assess the client further by testing for Chvostek's sign and Trousseau's sign. Then the nurse should notify the provider. Mouth care, loosening the dressing, and orientation questions do not provide important information to prevent complications of low calcium levels.

52. The nurse is teaching a patient being discharged after bilateral adrenalcectomy. What medication information does the nurse emphasize in the teaching plan? a. The dosage of steroid replacement drugs will be consistent throughout the patient's lifetime. b. The steroid drugs should be taken in the evening so as not to interfere with sleep. c. The patient should take the drugs on an empty stomach. d. The patient should learn how to give himself an intramuscular injection of hydrocortisone.

d

16. Which statement about the gonads is correct? a. Gonads are reproductive glands found in males only. b. The function of the hormones begins at birth in low, undetectable levels. c. The placenta secretes testosterone for the development of male external genitalia. d. External genitalia maturation is stimulated by gonadotropins during puberty.

d External genitalia maturation is stimulated by gonadotropins during puberty.

5. Which factor is a main assessment finding that signifies hypothyroidism? a. Irritability b. Cold intolerance c. Diarrhea d. Fatigue

b

54. In older adults, assessment findings of fatigue, altered thought processes, dry skin, and constipation are often mistaken for signs of aging rather than assessment findings for which endocrine disorder? a. Hyperthyroidism b. Hypothyroidism c. Hyperparathyroidism d. Hypoparathyroidism

b

6. Which sign/symptom is one of the first indicators of hyperthyroidism that is often noticed by the patient? a. Eyelid or globe lag b. Vision changes or tiring of the eyes c. Protruding eyes d. Photophobia

b

52. Which medication therapies does the nurse expect patients with hypoparathyroidism to receive? (Select all that apply.) a. Calcium chlroide b. Calcium gluconate c. Calitrol (Rocaltrol) d. Propranolol (Inderal) e. Ergocalciferol

a, b, c, e

Which signs and symptoms are assessment findings indicative of thyroid storm? (Select all that apply.) a. Abdominal pain and nausea b. Hypothermia c. Elevated temperature d. Tachycardia e. Elevated systolic blood pressure f. Bradycardia

a, c, d, e

1. Which glands are parts of the endocrine system? (Select all that apply.) a. Thyroid b. Occipital c. Parathyroid d. Adrenal e. Pituitary

a, c, d, e Thyroid, Parathyroid, Adrenal, Pituitary

2. Which assessment findings indicate hyperthyroidism? (Select all that apply.) a. Weight loss with increased appetite b. Constipation c. Increased heart rate d. Insomnia e. Decreased libido f. Heat intolerance

a, c, d, f

Which laboratory result is consistent with a diagnosis of hyperthyroidism? a. Decreased serum triiodothyronine (T3) and thyroxine (T4) levels b. Elevated serum thyrotropin-releasing hormone (TRH) level c. Decreased radioactive iodine uptake d. Increased serum T3 and T4

d

Management of the patient with hyperthyroidism focuses on which goals? (Select all that apply.) a. Blocking the effects of excessive thyroid secretion b. Treating the signs and symptoms the patient experiences c. Establishing euthyroid function d. Preventing spread of the disease e. Maintaining an environment of reduced stimulation

a, b, c, e

32. In SIADH, which laboratory value does the nurse expect to find? (Select all that apply.) a. Increased sodium in urine b. Elevated serum sodium level c. Increased specific gravity (concentrated urine) d. Decreased serum osmolarity e. Decreased urine specific gravity

a, c, d

56. Which conditions may precipitate myxedema coma? (Select all that apply.) a. Rapid withdrawal of thyroid medication b. Vitamin D deficiency c. Untreated hypothyroidism d. Surgery e. Excessive exposure to iodine

a, c, d

56. Which interventions are necessary for a patient with acute adrenal insufficiency (Addisonian crisis)? (Select all that apply.) a. IV infusion of normal saline b. IV infusion of 3% saline c. Hourly glucose monitoring d. Insulin administration e. IV potassium therapy

a, c, d

Which factor is a main assessment finding that signifies hypothyroidism? a. Irritability b. Cold intolerance c. Diarrhea d. Fatigue

b

Which factors are considered to be triggers for thyroid storm? (Select all that apply.) a. Infection b. Cold temperatures c. Vigorous palpation of a goiter d. Diabetic ketoacidosis e. Extremely warm temperatures

a, c, d

4. Which factor is a hallmark assessment finding that signifies hyperthyroidism? a. Weight loss b. Increased libido c. Heat intolerance d. Diarrhea

c

The nurse is assessing a patient with Graves' disease and observes an abnormal protrusion of both eyeballs. How does the nurse document this assessment finding? a. Globe lag b. Pretibial myxedema c. Exophthalmos d. Goiter

c

A nurse develops a dietary plan for a client with diabetes mellitus and new-onset microalbuminuria. Which component of the client's diet should the nurse decrease? a. Carbohydrates b. Proteins c. Fats d. Total calories

ANS: B Restriction of dietary protein to 0.8 g/kg of body weight per day is recommended for clients with microalbuminuria to delay progression to renal failure. The client's diet does not need to be decreased in carbohydrates, fats, or total calories.

A nurse cares for a client experiencing diabetic ketoacidosis who presents with Kussmaul respirations. Which action should the nurse take? a. Administration of oxygen via face mask b. Intravenous administration of 10% glucose c. Implementation of seizure precautions d. Administration of intravenous insulin

ANS: D The rapid, deep respiratory efforts of Kussmaul respirations are the body's attempt to reduce the acids produced by using fat rather than glucose for fuel. Only the administration of insulin will reduce this type of respiration by assisting glucose to move into cells and to be used for fuel instead of fat. The client who is in ketoacidosis may not experience any respiratory impairment and therefore does not need additional oxygen. Giving the client glucose would be contraindicated. The client does not require seizure precautions.

Ch.61 p. 1264, Health Promotion and Maintenance Which precaution or action is most important for the nurse to teach the client who is to collect a 24-hour urine specimen for endocrine testing? A. Eat a normal diet during the collection period. B. Wear gloves when you urinate to prevent contamination of the specimen. C. Urinate at the end of 24 hours and add that sample to the collection container. D. Avoid walking, running, dancing, or any vigorous exercise during the collection period.

Answer: C Rationale: When a 24-hour urine specimen is started, the specimen should reflect all the urine produced during the specified time. The very first voiding is discarded because the urine has spent some time in the bladder and will not reflect what is happening during the actual 24 hours of the collection. The time of this discard is the beginning of the 24-hour collection period. The test requires that all urine voided after the start time be collected, including the specimen collected by emptying the bladder at end of the 24 hours, which marks the end of the test.

A client has hyperparathyroidism. Which incident witnessed by the nurse requires the nurse's intervention? A. The client eating a morning meal of cereal and fruit B. The physical therapist walking with the client in the hallway C. Unlicensed assistive personnel pulling the client up in bed by the shoulders D. Visitors talking with the client about going home

C. The client with hyperparathyroidism is at risk for pathologic fracture. All members of the health care team must move the client carefully. A lift sheet should be used to re-position the client. The client with hyperparathyroidism is not restricted from eating and should maintain a balanced diet. The client can benefit from moderate exercise and physical therapy, and is not restricted from having visitors.

46. Which are physical findings of Cushing's disease? (Select all that apply.) a. "Moon-faced" appearance b. Decreased amount of body hair c. Truncal obesity d. Coarse facial features e. Thin, easily damaged skin f. Extremity muscle wasting

a, c, e, f

26. Which statement about the pancreas is correct? a. Endocrine functions of the pancreas include secretion of digestive enzymes. b. Exocrine functions of the pancreas include secretion of glucagon and insulin. c. The islets of Langerhans are the only source of somatostatin secretion. d. Somatostatin inhibits pancreatic secretion of glucagon and insulin.

d Somatostatin inhibits pancreatic secretion of glucagon and insulin.

The nurse is assessing a patient diagnosed with hyperthyroidism and observes dry, waxy swelling of the front surfaces of the lower legs. How does the nurse interpret this finding? a. Globe lag b. Pretibial myxedema c. Exophthalmos d. Goiter

b

24. Which statements about DI are accurate? (Select all that apply.) a. It is caused by ADH deficiency. b. It is characterized by a decrease in urination. c. Urine output of greater than 4 L/24 hours is the first diagnostic indication. d. The water loss increases plasma osmolarity. e. Nephrogenic DI can be caused by lithium (Eskalith).

a, c, d

28. Which factors are considered to be triggers for thyroid storm? (Select all that apply.) a. Infection b. Cold temperatures c. Vigorous palpation of a goiter d. Diabetic ketoacidosis e. Extremely warm temperatures

a, c, d

The nurse is providing instructions to a patient who is taking the antithyroid medication propylthiouracil (PTU). The nurse instructs the patient to notify the health care provider immediately if which sign/symptom occurs? a. Weight gain b. Dark-colored urine c. Cold intolerance d. Headache

b

53. Which statement about a patient with hyperaldosteronism after a successful unilateral adrenalectomy is correct? a. The low-sodium diet must be continued postoperatively. b. Glucocorticoid replacement therapy is temporary. c. Spironolactone (Aldactone) must be taken for life. d. Additional measures are needed to control hypertension.

b

Which condition is a life-threatening emergency and serious complication of untreated or poorly treated hypothyroidism? a. Endemic goiter b. Myxedema coma c. Toxic multinodular goiter d. Thyroiditis

b

15. Which statement about pituitary hormones is correct? a. ACTH acts on the adrenal medulla. b. Follicle-stimulating hormone (FSH) stimulates sperm production in men. c. Growth hormone promotes protein catabolism. d. Vasopressin decreases systolic blood pressure.

b Follicle-stimulating hormone (FSH) stimulates sperm production in men.

37. The nurse is assessing a patient after thyroid surgery and discovers harsh, high-pitched respiratory sounds. What is the nurse's best first action? a. Administer oxygen at 5 L via nasal cannula. b. Administer IV calcium chlroide. c. Notify the Rapid Response Team. d. Suction the patient for oral secretions.

c

45. In addition to regulation of calcium levels, PTH and calcitonin regulate the circulating blood levels of which substance? a. Potassium b. Sodium c. Phsophate d. Chloride

c

48. A patient with continuous spasms of the muscles is diagnosed with hypoparathyroidism. The muscle spasms are a clinical manifestation of which condition? a. Nerve damage b. Seizures c. Tetany e. Decreased ppotassium

c

74. The nurse identifies the client problem "risk for imbalanced body temperature" for the client diagnosed with hypothyroidism. Which intervention should be included in the plan of care? 1. Discourage the use of an electric blanket. 2. Assess the client's temperature every two (2) hours. 3. Keep the room temperature cool. 4. Space activities to promote rest.

1. External heat sources (heating pads, electric or warming blankets) should be discouraged because they increase the risk of peripheral vasodilation and vascular collapse. TEST-TAKING HINT: The test taker must always know exactly what the question is asking. Option "4" can be ruled out because it does not address body temperature. If the test taker knows the normal function of the thyroid gland, this may help identify the answer; decreased metabolism will cause the client to be cold.

80. The 68-year-old client diagnosed with hyperthyroidism is being treated with radioactive iodine therapy. Which interventions should the nurse discuss with the client? 1. Explain it will take up to a month for symptoms of hyperthyroidism to subside. 2. Teach the iodine therapy will have to be tapered slowly over one (1) week. 3. Discuss the client will have to be hospitalized during the radioactive therapy. 4. Inform the client after therapy the client will not have to take any medication.

1. Radioactive iodine therapy is used to destroy the overactive thyroid cells. After treatment, the client is followed closely for three (3) to four (4) weeks until the euthyroid state is reached. TEST-TAKING HINT: Some questions require the test taker to be knowledgeable of the information, especially medical treatments, and there are no specific hints to help the test taker answer the question.

81. The nurse is teaching the client diagnosed with hyperthyroidism. Which information should be taught to the client? Select all that apply. 1. Notify the HCP if a three (3)-pound weight loss occurs in two (2) days. 2. Discuss ways to cope with the emotional lability. 3. Notify the HCP if taking over-the-counter medication. 4. Carry a medical identification card or bracelet. 5. Teach how to take thyroid medications correctly.

1. Weight loss indicates the medication may not be effective and will probably need to be increased. 2. The client needs to know emotional highs and lows are secondary to hyperthyroidism. With treatment, this emotional lability will subside. 3. Any over-the-counter medications (for example, alcohol-based medications) may negatively affect the client's hyperthyroidism or medications being used for treatment. 4. This will help any HCP immediately know of the client's condition, especially if the client is unable to tell the HCP. TEST-TAKING HINT: This alternate-type question instructs the test taker to select all the interventions that apply. The test taker must read and evaluate each option as to whether it applies or not.

77. The client is admitted to the intensive care department diagnosed with myxedema coma. Which assessment data warrant immediate intervention by the nurse? 1. Serum blood glucose level of 74 mg/dL. 2. Pulse oximeter reading of 90%. 3. Telemetry reading showing sinus bradycardia. 4. The client is lethargic and sleeps all the time.

2. A pulse oximeter reading of less than 93% is significant. A 90% pulse oximeter reading indicates a Pao2 of approximately 60 on an arterial blood gas test; this is severe hypoxemia and requires immediate intervention. TEST-TAKING HINT: The words "warrant immediate intervention" means the test taker should select an option that is abnormal for the disease process or a life-threatening symptom.

84. Which signs/symptoms should make the nurse suspect the client is experiencing a thyroid storm? 1. Obstipation and hypoactive bowel sounds. 2. Hyperpyrexia and extreme tachycardia. 3. Hypotension and bradycardia. 4. Decreased respirations and hypoxia.

2. Hyperpyrexia (high fever) and heart rate above 130 beats per minute are signs of thyroid storm, a severely exaggerated hyperthyroidism. TEST-TAKING HINT: If the test taker does not have the knowledge to answer the question, the test taker should look at the options closely. Options "1," "3," and "4" all have signs/symptoms of "decrease"—hypoactive, hypotension, and hypoxia. The test taker should select the option that does not match.

83. The nurse is preparing to administer the following medications. Which medication should the nurse question administering? 1. The thyroid hormone to the client who does not have a T3, T4 level. 2. The regular insulin to the client with a blood glucose level of 210 mg/dL. 3. The loop diuretic to the client with a potassium level of 3.3 mEq/L. 4. The cardiac glycoside to the client who has a digoxin level of 1.4 mg/dL.

3. This potassium level is below normal, which is 3.5 to 5.5 mEq/L. Therefore, the nurse should question administering this medication because loop diuretics cause potassium loss in the urine. TEST-TAKING HINT: When administering medication, the nurse must know when to question the medication, how to know it is effective, and what must be taught to keep the client safe while taking the medication. The test taker may want to turn the question around and say, "I should give this medication."

78. Which medication order should the nurse question in the client diagnosed with untreated hypothyroidism? 1. Thyroid hormones. 2. Oxygen. 3. Sedatives. 4. Laxatives.

3. Untreated hypothyroidism is characterized by an increased susceptibility to the effects of most hypnotic and sedative agents; therefore, the nurse should question this medication. TEST-TAKING HINT: When a question asks which order the nurse should question, three of the options are medications the nurse expects to administer to the client. Sometimes saying, "The nurse administers this medication," may help the test taker select the correct answer.

82. The nurse is providing an in-service on thyroid disorders. One of the attendees asks the nurse, "Why don't the people in the United States get goiters as often?" Which statement by the nurse is the best response? 1. "It is because of the screening techniques used in the United States." 2. "It is a genetic predisposition rare in North Americans." 3. "The medications available in the United States decrease goiters." 4. "Iodized salt helps prevent the development of goiters in the United States."

4. Almost all of the iodine entering the body is retained in the thyroid gland. A deficiency in iodine will cause the thyroid gland to work hard and enlarge, which is called a goiter. Goiters are commonly seen in geographical regions having an iodine deficiency. Most table salt in the United States has iodine added. TEST-TAKING HINT: The nurse must know about disease processes. There is no testtaking hint to help with knowledge

76. Which nursing intervention should be included in the plan of care for the client diagnosed with hyperthyroidism? 1. Increase the amount of fiber in the diet. 2. Encourage a low-calorie, low-protein diet. 3. Decrease the client's fluid intake to 1,000 mL/day. 4. Provide six (6) small, well-balanced meals a day.

4. The client with hyperthyroidism has an increased appetite; therefore, well-balanced meals served several times throughout the day will help with the client's constant hunger. TEST-TAKING HINT: If the test taker knows the metabolism is increased with hyperthyroidism, then increasing the food intake is the most appropriate choice.

A nurse cares for a client with chronic hypercortisolism. Which action should the nurse take? a. Wash hands when entering the room. b. Keep the client in airborne isolation. c. Observe the client for signs of infection. d. Assess the clients daily chest x-ray.

A (Excess cortisol reduces the number of circulating lymphocytes, inhibits maturation of macrophages, reduces antibody synthesis, and inhibits production of cytokines and inflammatory chemicals. As a result, these clients are at greater risk of infection and may not have the expected inflammatory manifestations when an infection is present. The nurse needs to take precautions to decrease the clients risk. It is not necessary to keep the client in isolation. The client does not need a daily chest x-ray.)

A client with hyperaldosteronism is being treated with spironolactone (Aldactone) before surgery. Which precautions does the nurse teach this client? a. Read the label before using salt substitutes. b. Do not add salt to your food when you eat. c. Avoid exposure to sunlight. d. Take Tylenol instead of aspirin for pain.

A (Spironolactone is a potassium-sparing diuretic used to control potassium levels. Its use can lead to hyperkalemia. Although the goal is to increase the clients potassium, unknowingly adding potassium can cause complications. Some salt substitutes are composed of potassium chloride and should be avoided by clients on spironolactone therapy. Depending on the client, he or she may benefit from a low-sodium diet before surgery, but this may not be necessary. Avoiding sunlight and Tylenol is not necessary.)

A nurse plans care for an older adult who is admitted to the hospital for pneumonia. The client has no known drug allergies and no significant health history. Which action should the nurse include in this client's plan of care? a. Initiate Airborne Precautions. b. Offer fluids every hour or two. c. Place an indwelling urinary catheter. d. Palpate the client's thyroid gland.

A normal age-related endocrine change is decreased antidiuretic hormone (ADH) production. This results in a more diluted urine output, which can lead to dehydration. If no contraindications are known, the nurse should offer (or delegate) the client something to drink at least every 2 hours. A client with simple pneumonia would not require Airborne Precautions. Indwelling urinary catheterization is not necessary for this client and would increase the client's risk for infection. The nurse should plan a toileting schedule and assist the client to the bathroom if needed. Palpating the client's thyroid gland is a part of a comprehensive examination but is not specifically related to this client.

4. A nurse assesses a client with hypothyroidism who is admitted with acute appendicitis. The nurse notes that the client's level of consciousness has decreased. Which actions should the nurse take? (Select all that apply.) a. Infuse intravenous fluids. b. Cover the client with warm blankets. c. Monitor blood pressure every 4 hours. d. Maintain a patent airway. e. Administer oral glucose as prescribed.

A, B, D. A client with hypothyroidism and an acute illness is at risk for myxedema coma. A decrease in level of consciousness is a symptom of myxedema. The nurse should infuse IV fluids, cover the client with warm blankets, monitor blood pressure every hour, maintain a patent airway, and administer glucose intravenously as prescribed.

A nurse assesses clients with potential endocrine disorders. Which clients are at high risk for hypopituitarism? (Select all that apply.) a. A 20-year-old female with benign pituitary tumors b. A 32-year-old male with diplopia c. A 41-year-old female with anorexia nervosa d. A 55-year-old male with hypertension e. A 60-year-old female who is experiencing shock f. A 68-year-old male who has gained weight recently

A, C, D, E (Pituitary tumors, anorexia nervosa, hypertension, and shock are all conditions that can cause hypopituitarism. Diplopia is a manifestation of hypopituitarism, and weight gain is a manifestation of Cushings disease and syndrome of inappropriate antidiuretic hormone. They are not risk factors for hypopituitarism.)

A nurse assesses a client with anterior pituitary hyperfunction. Which clinical manifestations should the nurse expect? (Select all that apply.) a. Protrusion of the lower jaw b. High-pitched voice c. Enlarged hands and feet d. Kyphosis e. Barrel-shaped chest f. Excessive sweating

A, C, D, E, F (Anterior pituitary hyperfunction typically will cause protrusion of the lower jaw, deepening of the voice, enlarged hands and feet, kyphosis, barrel-shaped chest, and excessive sweating.)

3. A nurse teaches a client with hyperthyroidism. Which dietary modifications should the nurse include in this client's teaching? (Select all that apply.) a. Increased carbohydrates b. Decreased fats c. Increased calorie intake d. Supplemental vitamins e. Increased proteins

A, C, E. The client is hypermetabolic and has an increased need for carbohydrates, calories, and proteins. Proteins are especially important because the client is at risk for a negative nitrogen balance. There is no need to decrease fat intake or take supplemental vitamins.

A nurse assesses a client who potentially has hyperaldosteronism. Which serum laboratory values should the nurse associate with this disorder? (Select all that apply.) a. Sodium: 150 mEq/L b. Sodium: 130 mEq/L c. Potassium: 2.5 mEq/L d. Potassium: 5.0 mEq/L e. pH: 7.28 f. pH: 7.50

A, C, F (Aldosterone increases reabsorption of sodium and excretion of potassium. Hyperaldosteronism causes hypernatremia, hypokalemia, and metabolic alkalosis. Hyponatremia, hyperkalemia, and acidosis are manifestations of adrenal insufficiency.)

5. A nurse teaches a client who is prescribed an unsealed radioactive isotope. Which statements should the nurse include in this client's education? (Select all that apply.) a. "Do not share utensils, plates, and cups with anyone else." b. "You can play with your grandchildren for 1 hour each day." c. "Eat foods high in vitamins such as apples, pears, and oranges." d. "Wash your clothing separate from others in the household." e. "Take a laxative 2 days after therapy to excrete the radiation."

A, D, E. A client who is prescribed an unsealed radioactive isotope should be taught to not share utensils, plates, and cups with anyone else; to avoid contact with pregnant women and children; to avoid eating foods with cores or bones, which will leave contaminated remnants; to wash clothing separate from others in the household and run an empty cycle before washing other people's clothing; and to take a laxative on days 2 and 3 after receiving treatment to help excrete the contaminated stool faster.

After receiving change-of-shift report about these four clients, which client does the nurse attend to first? A. Client with acute adrenal insufficiency who has a blood glucose of 36 mg/dL B. Client with diabetes insipidus who has a dose of desmopressin (DDAVP) due C. Client with hyperaldosteronism who has a serum potassium of 3.4 mEq/L D. Client with pituitary adenoma who is reporting a severe headache

A. A glucose level of 36 mg/dL is considered an emergency; this client must be assessed and treated immediately. Although it is important to maintain medications on schedule, the client requiring a dose of desmopressin is not the first client who needs to be seen. A serum potassium of 3.4 mEq/L in the client with hyperaldosteronism may be considered normal (or slightly hypokalemic), based on specific hospital levels. The client reporting a severe headache needs to be evaluated as soon as possible after the client with acute adrenal insufficiency. As an initial measure, the RN could delegate obtaining vital signs to unlicensed assistive personnel.

A client with iatrogenic Cushing's syndrome is a resident in a long-term care facility. Which nursing action included in the client's care would be best to delegate to unlicensed assistive personnel (UAP)? A. Assist with personal hygiene and skin care. B. Develop a plan of care to minimize risk for infection. C. Instruct the client on the reasons to avoid overeating. D. Monitor for signs and symptoms of fluid retention.

A. Assisting a client with bathing and skin care is included in UAP scope of practice. It is not within their scope of practice to develop a plan of care, although they will play a very important role in following the plan of care. Client teaching requires a broad education and should not be delegated to UAP. Monitoring for signs and symptoms of fluid retention is part of client assessment, which requires a higher level of education and clinical judgment.

57. A patient in the emergency department who reports lethargy, muscle weakness, nausea, vomiting, and weight loss over the past weeks is diagnosed with Addisonian crisis (acute adrenal insufficiency). Which drug(s) does the nurse expect to administer to this patient? a. Beta blocker to control the hypertension and dysrhythmias b. Solu-Cortef IV along with IM injections of hydrocortisone c. IV fluids of D5NS with KCl added for dehydration d. Spironolactone (Aldactone) to promote diuresis

b

A client has been admitted to the medical intensive care unit with a diagnosis of diabetes insipidus (DI) secondary to lithium overdose. Which medication is used to treat the DI? A. Desmopressin (DDAVP) B. Dopamine hydrochloride (Intropin) C. Prednisone D. Tolvaptan (Samsca)

A. Desmopressin is the drug of choice for treatment of severe DI. It may be administered orally, nasally, or by intramuscular or intravenous routes. Dopamine hydrochloride is a naturally occurring catecholamine and inotropic vasopressor; it would not be used to treat DI. Prednisone would not be used to treat DI. Tolvaptan is a selective competitive arginine vasopressin receptor 2 antagonist and is not used with DI.

A client with diabetes insipidus (DI) has dry lips and mucous membranes and poor skin turgor. Which intervention does the nurse provide first? A. Force fluids B. Offer lip balm C. Perform a 24-hour urine test D. Withhold desmopressin acetate (DDAVP)

A. Dry lips and mucous membranes and poor skin turgor are indications of dehydration, which can occur with DI. This is a serious condition that must be treated rapidly. Encouraging fluids is the initial step, provided the client is able to tolerate oral intake. Lip balm may make the client more comfortable, but does not address the problem of dehydration. A 24-hour urine test will identify loss of electrolytes and adrenal androgen metabolites, but will not correct the dehydration that this client is experiencing. Desmopressin acetate is a synthetic form of antidiuretic hormone that is given to reduce urine production; it improves DI and should not be withheld.

A client with pheochromocytoma is admitted for surgery. What does the nurse do for the admitting assessment? A. Avoids palpating the abdomen B. Monitors for pulmonary edema with a chest x-ray C. Obtains a 24-hour urine specimen on admission D. Places the client in a room with a roommate for distraction

A. The abdomen must not be palpated in a client with pheochromocytoma because this action could cause a sudden release of catecholamines and severe hypertension. The tumor on the adrenal gland causes sympathetic hyperactivity, increasing blood pressure and heart rate, not pulmonary edema. A 24-hour urine collection will already have been completed to determine the diagnosis of pheochromocytoma. A client diagnosed with a pheochromocytoma may feel anxious as part of the disease process; providing a roommate for distraction will not reduce the client's anxiety.

The nurse is teaching a client about how to monitor therapy effectiveness for syndrome of inappropriate antidiuretic hormone. What does the nurse tell the client to look for? A. Daily weight gain of less than 2 pounds B. Dry mucous membranes C. Increasing heart rate D. Muscle spasms

A. The client must monitor daily weights because this assesses the degree of fluid restriction needed. A weight gain of 2 pounds or more daily or a gradual increase over several days is cause for concern. Dry mucous membranes are a sign of dehydration and an indication that therapy is not effective. An increased heart rate indicates increased fluid retention or dehydration and hypovolemia, and either condition is an indication that therapy is not effective. Muscle spasms are associated with hyponatremia and are an indication of a change in the client's neurologic status. Untreated hyponatremia can lead to seizures and coma.

A client is being discharged with propylthiouracil (PTU). Which statement by the client indicates a need for further teaching by the nurse? A. "I can return to my job at the nursing home." B. "I must call if my urine is dark." C. "I must faithfully take the drug every 8 hours." D. "I need to report weight gain."

A. The client should avoid large crowds and people who are ill because PTU reduces blood cell counts and the immune response, which increases the risk for infection. The client does not, however, need to remain completely at home. Dark urine may indicate liver toxicity or failure, and the client must notify the provider immediately. Taking PTU regularly at the same time each day provides better drug levels and ensures better drug action. The client must notify the provider of weight gain because this may indicate hypothyroidism; a lower drug dose may be required.

A client with syndrome of inappropriate antidiuretic hormone is admitted with a serum sodium level of 105 mEq/L. Which request by the health care provider does the nurse address first? A. Administer infusion of 150 mL of 3% NaCl over 3 hours. B. Draw blood for hemoglobin and hematocrit. C. Insert retention catheter and monitor urine output. D. Weigh the client on admission and daily thereafter.

A. The client with a sodium level of 105 mEq/L is at high risk for seizures and coma. The priority intervention is to increase the sodium level to a more normal range. Ideally, 3% NaCl should be infused through a central line or with a small needle through a large vein to prevent irritation. Monitoring laboratory values for fluid balance and monitoring urine output are important, but are not the top priority. Monitoring client weight will help in the assessment of fluid balance; however, this is also not the top priority.

The nurse manager for the medical-surgical unit is making staff assignments. Which client will be most appropriate to assign to a newly graduated RN who has completed a 6-week unit orientation? A. Client with chronic hypothyroidism and dementia who takes levothyroxine (Synthroid) daily B. Client with follicular thyroid cancer who has vocal hoarseness and difficulty swallowing C. Client with Graves' disease who is experiencing increasing anxiety and diaphoresis D. Client with hyperparathyroidism who has just arrived on the unit after a parathyroidectomy

A. The client with chronic hypothyroidism and dementia is the most stable of the clients described and would be most appropriate to assign to an inexperienced RN. A client with vocal hoarseness and difficulty swallowing is at higher risk for complications and requires close observation by a more experienced nurse. Increasing anxiety and diaphoresis in a client with Graves' disease can be an indication of impending thyroid storm, which is an emergency; this is not a situation to be managed by a newly graduated RN. A client who has just arrived on the unit after a parathyroidectomy requires close observation for bleeding and airway compromise and requires assessment by an experienced nurse.

An emergency department nurse assesses a client with ketoacidosis. Which clinical manifestation should the nurse correlate with this condition? a. Increased rate and depth of respiration b. Extremity tremors followed by seizure activity c. Oral temperature of 102° F (38.9° C) d. Severe orthostatic hypotension

ANS: A Ketoacidosis decreases the pH of the blood, stimulating the respiratory control areas of the brain to buffer the effects of increasing acidosis. The rate and depth of respiration are increased (Kussmaul respirations) in an attempt to excrete more acids by exhalation. Tremors, elevated temperature, and orthostatic hypotension are not associated with ketoacidosis.

A client presents to the emergency department with a history of adrenal insufficiency. The following laboratory values are obtained: Na+ 130 mEq/L, K+ 5.6 mEq/L, and glucose 72 mg/dL. Which is the first request that the nurse anticipates? A. Administer insulin and dextrose in normal saline to shift potassium into cells. B. Give spironolactone (Aldactone) 100 mg orally. C. Initiate histamine2 (H2) blocker therapy with ranitidine for ulcer prophylaxis. D. Obtain arterial blood gases to assess for peaked T waves.

A. This client is hyperkalemic. The nurse should anticipate a request to administer 20 to 50 units of insulin with 20 to 50 mg of dextrose in normal saline as an IV infusion to shift potassium into the cells. Spironolactone is a potassium-sparing diuretic that helps the body keep potassium, which the client does not need. Although H2 blocker therapy would be appropriate for this client, it is not the first priority. Arterial blood gases are not used to assess for peaked T waves associated with hyperkalemia; an electrocardiogram needs to be obtained instead.

A preoperative nurse assesses a client who has type 1 diabetes mellitus prior to a surgical procedure. The client's blood glucose level is 160 mg/dL. Which action should the nurse take? a. Document the finding in the client's chart. b. Administer a bolus of regular insulin IV. c. Call the surgeon to cancel the procedure. d. Draw blood gases to assess the metabolic state.

ANS: A Clients who have type 1 diabetes and are having surgery have been found to have fewer complications, lower rates of infection, and better wound healing if blood glucose levels are maintained at between 140 and 180 mg/dL throughout the perioperative period. The nurse should document the finding and proceed with other operative care. The need for a bolus of insulin, canceling the procedure, or drawing arterial blood gases is not required.

A nurse teaches a client who is diagnosed with diabetes mellitus. Which statement should the nurse include in this client's plan of care to delay the onset of microvascular and macrovascular complications? a. "Maintain tight glycemic control and prevent hyperglycemia." b. "Restrict your fluid intake to no more than 2 liters a day." c. "Prevent hypoglycemia by eating a bedtime snack." d. "Limit your intake of protein to prevent ketoacidosis."

ANS: A Hyperglycemia is a critical factor in the pathogenesis of long-term diabetic complications. Maintaining tight glycemic control will help delay the onset of complications. Restricting fluid intake is not part of the treatment plan for clients with diabetes. Preventing hypoglycemia and ketosis, although important, are not as important as maintaining daily glycemic control.

After teaching a client who is recovering from pancreas transplantation, the nurse assesses the client's understanding. Which statement made by the client indicates a need for additional education? a. "If I develop an infection, I should stop taking my corticosteroid." b. "If I have pain over the transplant site, I will call the surgeon immediately." c. "I should avoid people who are ill or who have an infection." d. "I should take my cyclosporine exactly the way I was taught."

ANS: A Immunosuppressive agents should not be stopped without the consultation of the transplantation physician, even if an infection is present. Stopping immunosuppressive therapy endangers the transplanted organ. The other statements are correct. Pain over the graft site may indicate rejection. Anti-rejection drugs cause immunosuppression, and the client should avoid crowds and people who are ill. Changing the routine of anti-rejection medications may cause them to not work optimally.

A nurse prepares to administer insulin to a client at 1800. The client's medication administration record contains the following information: • Insulin glargine: 12 units daily at 1800 • Regular insulin: 6 units QID at 0600, 1200, 1800, 2400 Based on the client's medication administration record, which action should the nurse take? a. Draw up and inject the insulin glargine first, and then draw up and inject the regular insulin. b. Draw up and inject the insulin glargine first, wait 20 minutes, and then draw up and inject the regular insulin. c. First draw up the dose of regular insulin, then draw up the dose of insulin glargine in the same syringe, mix, and inject the two insulins together. d. First draw up the dose of insulin glargine, then draw up the dose of regular insulin in the same syringe, mix, and inject the two insulins together.

ANS: A Insulin glargine must not be diluted or mixed with any other insulin or solution. Mixing results in an unpredictable alteration in the onset of action and time to peak action. The correct instruction is to draw up and inject first the glargine and then the regular insulin right afterward.

A nurse cares for a client with diabetes mellitus who is visually impaired. The client asks, "Can I ask my niece to prefill my syringes and then store them for later use when I need them?" How should the nurse respond? a. "Yes. Prefilled syringes can be stored for 3 weeks in the refrigerator in a vertical position with the needle pointing up." b. "Yes. Syringes can be filled with insulin and stored for a month in a location that is protected from light." c. "Insulin reacts with plastic, so prefilled syringes are okay, but you will need to use glass syringes." d. "No. Insulin syringes cannot be prefilled and stored for any length of time outside of the container."

ANS: A Insulin is relatively stable when stored in a cool, dry place away from light. When refrigerated, prefilled plastic syringes are stable for up to 3 weeks. They should be stored in the refrigerator in the vertical position with the needle pointing up to prevent suspended insulin particles from clogging the needle.

A nurse cares for a client who has a family history of diabetes mellitus. The client states, "My father has type 1 diabetes mellitus. Will I develop this disease as well?" How should the nurse respond? a. "Your risk of diabetes is higher than the general population, but it may not occur." b. "No genetic risk is associated with the development of type 1 diabetes mellitus." c. "The risk for becoming a diabetic is 50% because of how it is inherited." d. "Female children do not inherit diabetes mellitus, but male children will."

ANS: A Risk for type 1 diabetes is determined by inheritance of genes coding for HLA-DR and HLA-DQ tissue types. Clients who have one parent with type 1 diabetes are at increased risk for its development. Diabetes (type 1) seems to require interaction between inherited risk and environmental factors, so not everyone with these genes develops diabetes. The other statements are not accurate.

After teaching a client with diabetes mellitus to inject insulin, the nurse assesses the client's understanding. Which statement made by the client indicates a need for additional teaching? a. "The lower abdomen is the best location because it is closest to the pancreas." b. "I can reach my thigh the best, so I will use the different areas of my thighs." c. "By rotating the sites in one area, my chance of having a reaction is decreased." d. "Changing injection sites from the thigh to the arm will change absorption rates."

ANS: A The abdominal site has the fastest rate of absorption because of blood vessels in the area, not because of its proximity to the pancreas. The other statements are accurate assessments of insulin administration.

A nurse assesses a client with diabetes mellitus and notes the client only responds to a sternal rub by moaning, has capillary blood glucose of 33 g/dL, and has an intravenous line that is infiltrated with 0.45% normal saline. Which action should the nurse take first? a. Administer 1 mg of intramuscular glucagon. b. Encourage the client to drink orange juice. c. Insert a new intravenous access line. d. Administer 25 mL dextrose 50% (D50) IV push.

ANS: A The client's blood glucose level is dangerously low. The nurse needs to administer glucagon IM immediately to increase the client's blood glucose level. The nurse should insert a new IV after administering the glucagon and can use the new IV site for future doses of D50 if the client's blood glucose level does not rise. Once the client is awake, orange juice may be administered orally along with a form of protein such as a peanut butter.

A nurse assesses a client who has diabetes mellitus and notes the client is awake and alert, but shaky, diaphoretic, and weak. Five minutes after administering a half-cup of orange juice, the client's clinical manifestations have not changed. Which action should the nurse take next? a. Administer another half-cup of orange juice. b. Administer a half-ampule of dextrose 50% intravenously. c. Administer 10 units of regular insulin subcutaneously. d. Administer 1 mg of glucagon intramuscularly.

ANS: A This client is experiencing mild hypoglycemia. For mild hypoglycemic manifestations, the nurse should administer oral glucose in the form of orange juice. If the symptoms do not resolve immediately, the treatment should be repeated. The client does not need intravenous dextrose, insulin, or glucagon.

A nurse cares for a client who is prescribed a 24-hour urine collection. The unlicensed assistive personnel (UAP) reports that, while pouring urine into the collection container, some urine splashed his hand. Which action should the nurse take next? a. Ask the UAP if he washed his hands afterward. b. Have the UAP fill out an incident report. c. Ask the laboratory if the container has preservative in it. d. Send the UAP to Employee Health right away.

ANS: A For safety, the nurse should find out if the UAP washed his or her hands. The UAP should do this for two reasons. First, it is part of Standard Precautions to wash hands after client care. Second, if the container did have preservative in it, this would wash it away. The preservative may be caustic to the skin. The nurse can call the laboratory while the UAP is washing hands, if needed. The UAP would then need to fill out an incident or exposure report and may or may not need to go to Employee Health. The UAP also needs further education on Standard Precautions, which include wearing gloves.

A nurse teaches a client who has been prescribed a 24-hour urine collection to measure excreted hormones. The client asks, "Why do I need to collect urine for 24 hours instead of providing a random specimen?" How should the nurse respond? a. "This test will assess for a hormone secreted on a circadian rhythm." b. "The hormone is diluted in urine; therefore, we need a large volume." c. "We are assessing when the hormone is secreted in large amounts." d. "To collect the correct hormone, you need to urinate multiple times."

ANS: A Some hormones are secreted in a pulsatile, or circadian, cycle. When testing for these substances, a collection that occurs over 24 hours will most accurately reflect hormone secretion. Dilution of hormones in urine, secretion of hormone amounts, and ability to collect the correct hormone are not reasons to complete a 24-hour urine test.

A nurse assesses a client with diabetes mellitus. Which clinical manifestation should alert the nurse to decreased kidney function in this client? a. Urine specific gravity of 1.033 b. Presence of protein in the urine c. Elevated capillary blood glucose level d. Presence of ketone bodies in the urine

ANS: B Renal dysfunction often occurs in the client with diabetes. Proteinuria is a result of renal dysfunction. Specific gravity is elevated with dehydration. Elevated capillary blood glucose levels and ketones in the urine are consistent with diabetes mellitus but are not specific to renal function.

A nurse assesses a client who is prescribed a medication that stimulates beta1 receptors. Which assessment finding should alert the nurse to urgently contact the health care provider? a. Heart rate of 50 beats/min b. Respiratory rate of 18 breaths/min c. Oxygenation saturation of 92% d. Blood pressure of 144/69 mm Hg

ANS: A Stimulation of beta1 receptor sites in the heart has positive chronotropic and inotropic actions. The nurse expects an increase in heart rate and increased cardiac output. The client with a heart rate of 50 beats/min would be cause for concern because this would indicate that the client was not responding to the medication. The other vital signs are within normal limits and do not indicate a negative response to the medication.

A nurse collaborates with an unlicensed assistive personnel (UAP) to provide care for a client who is prescribed a 24-hour urine specimen collection. Which statement should the nurse include when delegating this activity to the UAP? a. "Note the time of the client's first void and collect urine for 24 hours." b. "Add the preservative to the container at the end of the test." c. "Start the collection by saving the first urine of the morning." d. "It is okay if one urine sample during the 24 hours is not collected."

ANS: A The collection of a 24-hour urine specimen is often delegated to a UAP. The nurse must ensure that the UAP understands the proper process for collecting the urine. The 24-hour urine collection specimen is started after the client's first urination. The first urine specimen is discarded because there is no way to know how long it has been in the bladder, but the time of the client's first void is noted. The client adds all urine voided after that first discarded specimen during the next 24 hours. When the 24-hour mark is reached, the client voids one last time and adds this specimen to the collection. The preservative, if used, must be added to the container at the beginning of the collection. All urine samples need to be collected for the test results to be accurate.

A nurse assesses a client diagnosed with adrenal hypofunction. Which client statement should the nurse correlate with this diagnosis? a. "I have a terrible craving for potato chips." b. "I cannot seem to drink enough water." c. "I no longer have an appetite for anything." d. "I get hungry even after eating a meal."

ANS: A The nurse correlates a client's salt craving with adrenal hypofunction. Excessive thirst is related to diabetes insipidus or diabetes mellitus. Clients who have hypothyroidism often have a decrease in appetite. Excessive hunger is associated with diabetes mellitus.

A nurse assesses clients who have endocrine disorders. Which assessment findings are paired correctly with the endocrine disorder? (Select all that apply.) a. Excessive thyroid-stimulating hormone - Increased bone formation b. Excessive melanocyte-stimulating hormone - Darkening of the skin c. Excessive parathyroid hormone - Synthesis and release of corticosteroids d. Excessive antidiuretic hormone - Increased urinary output e. Excessive adrenocorticotropic hormone - Increased bone resorption

ANS: A, B Thyroid-stimulating hormone targets thyroid tissue and stimulates the formation of bone. Melanocyte-stimulating hormone stimulates melanocytes and promotes pigmentation or the darkening of the skin. Parathyroid hormone stimulates bone resorption. Antidiuretic hormone targets the kidney and promotes water reabsorption, causing a decrease in urinary output. Adrenocorticotropic hormone targets the adrenal cortex and stimulates the synthesis and release of corticosteroids.

A nurse provides diabetic education at a public health fair. Which disorders should the nurse include as complications of diabetes mellitus? (Select all that apply.) a. Stroke b. Kidney failure c. Blindness d. Respiratory failure e. Cirrhosis

ANS: A, B, C Complications of diabetes mellitus are caused by macrovascular and microvascular changes. Macrovascular complications include coronary artery disease, cerebrovascular disease, and peripheral vascular disease. Microvascular complications include nephropathy, retinopathy, and neuropathy. Respiratory failure and cirrhosis are not complications of diabetes mellitus.

A nurse cares for clients with hormone disorders. Which are common key features of hormones? (Select all that apply.) a. Hormones may travel long distances to get to their target tissues. b. Continued hormone activity requires continued production and secretion. c. Control of hormone activity is caused by negative feedback mechanisms. d. Most hormones are stored in the target tissues for use later. e. Most hormones cause target tissues to change activities by changing gene activity.

ANS: A, B, C Hormones are secreted by endocrine glands and travel through the body to reach their target tissues. Hormone activity can increase or decrease according to the body's needs, and continued hormone activity requires continued production and secretion. Control is maintained via negative feedback. Hormones are not stored for later use, and they do not alter genetic activity

A nurse collaborates with the interdisciplinary team to develop a plan of care for a client who is newly diagnosed with diabetes mellitus. Which team members should the nurse include in this interdisciplinary team meeting? (Select all that apply.) a. Registered dietitian b. Clinical pharmacist c. Occupational therapist d. Health care provider e. Speech-language pathologist

ANS: A, B, D When planning care for a client newly diagnosed with diabetes mellitus, the nurse should collaborate with a registered dietitian, clinical pharmacist, and health care provider. The focus of treatment for a newly diagnosed client would be nutrition, medication therapy, and education. The nurse could also consult with a diabetic educator. There is no need for occupational therapy or speech therapy at this time.

A nurse teaches a client with diabetes mellitus about foot care. Which statements should the nurse include in this client's teaching? (Select all that apply.) a. "Do not walk around barefoot." b. "Soak your feet in a tub each evening." c. "Trim toenails straight across with a nail clipper." d. "Treat any blisters or sores with Epsom salts." e. "Wash your feet every other day."

ANS: A, C Clients who have diabetes mellitus are at high risk for wounds on the feet secondary to peripheral neuropathy and poor arterial circulation. The client should be instructed to not walk around barefoot or wear sandals with open toes. These actions place the client at higher risk for skin breakdown of the feet. The client should be instructed to trim toenails straight across with a nail clipper. Feet should be washed daily with lukewarm water and soap, but feet should not be soaked in the tub. The client should contact the provider immediately if blisters or sores appear and should not use home remedies to treat these wounds.

A nurse assesses a client who is experiencing diabetic ketoacidosis (DKA). For which manifestations should the nurse monitor the client? (Select all that apply.) a. Deep and fast respirations b. Decreased urine output c. Tachycardia d. Dependent pulmonary crackles e. Orthostatic hypotension

ANS: A, C, E DKA leads to dehydration, which is manifested by tachycardia and orthostatic hypotension. Usually clients have Kussmaul respirations, which are fast and deep. Increased urinary output (polyuria) is severe. Because of diuresis and dehydration, peripheral edema and crackles do not occur.

A nurse cares for a client with a hypofunctioning anterior pituitary gland. Which hormones should the nurse expect to be affected by this condition? (Select all that apply.) a. Thyroid-stimulating hormone b. Vasopressin c. Follicle-stimulating hormone d. Calcitonin e. Growth hormone

ANS: A, C, E Thyroid-stimulating hormone, follicle-stimulating hormone, and growth hormone all are secreted by the anterior pituitary gland. Vasopressin is secreted from the posterior pituitary gland. Calcitonin is secreted from the thyroid gland.

8. The laboratory results for a 53-year-old patient indicate a low T3 level and elevated thyroid-stimulating hormone (TSH). What do these results indicate? a. Hyperthyroidism b. Hypothyroidism c. Malfunctioning pituitary gland d. Normal laboratory values for this age

b

A nurse assesses clients at a health fair. Which clients should the nurse counsel to be tested for diabetes? (Select all that apply.) a. 56-year-old African-American male b. Female with a 30-pound weight gain during pregnancy c. Male with a history of pancreatic trauma d. 48-year-old woman with a sedentary lifestyle e. Male with a body mass index greater than 25 kg/m2 f. 28-year-old female who gave birth to a baby weighing 9.2 pounds

ANS: A, D, E, F Risk factors for type 2 diabetes include certain ethnic/racial groups (African Americans, American Indians, Hispanics), obesity and physical inactivity, and giving birth to large babies. Pancreatic trauma and a 30-pound gestational weight gain are not risk factors.

A nurse cares for a client who is diagnosed with acute rejection 2 months after receiving a simultaneous pancreas-kidney transplant. The client states, "I was doing so well with my new organs, and the thought of having to go back to living on hemodialysis and taking insulin is so depressing." How should the nurse respond? a. "Following the drug regimen more closely would have prevented this." b. "One acute rejection episode does not mean that you will lose the new organs." c. "Dialysis is a viable treatment option for you and may save your life." d. "Since you are on the national registry, you can receive a second transplantation."

ANS: B An episode of acute rejection does not automatically mean that the client will lose the transplant. Pharmacologic manipulation of host immune responses at this time can limit damage to the organ and allow the graft to be maintained. The other statements either belittle the client or downplay his or her concerns. The client may not be a candidate for additional organ transplantation.

A nurse is teaching a client with diabetes mellitus who asks, "Why is it necessary to maintain my blood glucose levels no lower than about 60 mg/dL?" How should the nurse respond? a. "Glucose is the only fuel used by the body to produce the energy that it needs." b. "Your brain needs a constant supply of glucose because it cannot store it." c. "Without a minimum level of glucose, your body does not make red blood cells." d. "Glucose in the blood prevents the formation of lactic acid and prevents acidosis."

ANS: B Because the brain cannot synthesize or store significant amounts of glucose, a continuous supply from the body's circulation is needed to meet the fuel demands of the central nervous system. The nurse would want to educate the client to prevent hypoglycemia. The body can use other sources of fuel, including fat and protein, and glucose is not involved in the production of red blood cells. Glucose in the blood will encourage glucose metabolism but is not directly responsible for lactic acid formation.

After teaching a client with type 2 diabetes mellitus, the nurse assesses the client's understanding. Which statement made by the client indicates a need for additional teaching? a. "I need to have an annual appointment even if my glucose levels are in good control." b. "Since my diabetes is controlled with diet and exercise, I must be seen only if I am sick." c. "I can still develop complications even though I do not have to take insulin at this time." d. "If I have surgery or get very ill, I may have to receive insulin injections for a short time."

ANS: B Clients with diabetes need to be seen at least annually to monitor for long-term complications, including visual changes, microalbuminuria, and lipid analysis. The client may develop complications and may need insulin in the future.

A nurse reviews the medication list of a client with a 20-year history of diabetes mellitus. The client holds up the bottle of prescribed duloxetine (Cymbalta) and states, "My cousin has depression and is taking this drug. Do you think I'm depressed?" How should the nurse respond? a. "Many people with long-term diabetes become depressed after a while." b. "It's for peripheral neuropathy. Do you have burning pain in your feet or hands?" c. "This antidepressant also has anti-inflammatory properties for diabetic pain." d. "No. Many medications can be used for several different disorders."

ANS: B Damage along nerves causes peripheral neuropathy and leads to burning pain along the nerves. Many drugs, including duloxetine (Cymbalta), can be used to treat peripheral neuropathy. The nurse should assess the client for this condition and then should provide an explanation of why this drug is being used. This medication, although it is used for depression, is not being used for that reason in this case. Duloxetine does not have anti-inflammatory properties. Telling the client that many medications are used for different disorders does not provide the client with enough information to be useful.

A nurse cares for a client with diabetes mellitus who asks, "Why do I need to administer more than one injection of insulin each day?" How should the nurse respond? a. "You need to start with multiple injections until you become more proficient at self-injection." b. "A single dose of insulin each day would not match your blood insulin levels and your food intake patterns." c. "A regimen of a single dose of insulin injected each day would require that you eat fewer carbohydrates." d. "A single dose of insulin would be too large to be absorbed, predictably putting you at risk for insulin shock."

ANS: B Even when a single injection of insulin contains a combined dose of different-acting insulin types, the timing of the actions and the timing of food intake may not match well enough to prevent wide variations in blood glucose levels. One dose of insulin would not be appropriate even if the client decreased carbohydrate intake. Additional injections are not required to allow the client practice with injections, nor will one dose increase the client's risk of insulin shock.

A nurse cares for a client who has diabetes mellitus. The nurse administers 6 units of regular insulin and 10 units of NPH insulin at 0700. At which time should the nurse assess the client for potential problems related to the NPH insulin? a. 0800 b. 1600 c. 2000 d. 2300

ANS: B Neutral protamine Hagedorn (NPH) is an intermediate-acting insulin with an onset of 1.5 hours, peak of 4 to 12 hours, and duration of action of 22 hours. Checking the client at 0800 would be too soon. Checking the client at 2000 and 2300 would be too late. The nurse should check the client at 1600.

A patient has been prescribed thyroid hormone for treatment of hypothyroidism. Within what time frame does the patient expect improvement in mental awareness with this treatment? a. A few days b. 2 weeks c. 1 month d. 3 months

b

A nurse teaches a client about self-monitoring of blood glucose levels. Which statement should the nurse include in this client's teaching to prevent bloodborne infections? a. "Wash your hands after completing each test." b. "Do not share your monitoring equipment." c. "Blot excess blood from the strip with a cotton ball." d. "Use gloves when monitoring your blood glucose."

ANS: B Small particles of blood can adhere to the monitoring device, and infection can be transported from one user to another. Hepatitis B in particular can survive in a dried state for about a week. The client should be taught to avoid sharing any equipment, including the lancet holder. The client should be taught to wash his or her hands before testing. The client would not need to blot excess blood away from the strip or wear gloves.

A nurse reviews laboratory results for a client with diabetes mellitus who is prescribed an intensified insulin regimen: • Fasting blood glucose: 75 mg/dL • Postprandial blood glucose: 200 mg/dL • Hemoglobin A1c level: 5.5% How should the nurse interpret these laboratory findings? a. Increased risk for developing ketoacidosis b. Good control of blood glucose c. Increased risk for developing hyperglycemia d. Signs of insulin resistance

ANS: B The client is maintaining blood glucose levels within the defined ranges for goals in an intensified regimen. Because the client's glycemic control is good, he or she is not at higher risk for ketoacidosis or hyperglycemia and is not showing signs of insulin resistance.

A nurse cares for a client who is prescribed pioglitazone (Actos). After 6 months of therapy, the client reports that his urine has become darker since starting the medication. Which action should the nurse take? a. Assess for pain or burning with urination. b. Review the client's liver function study results. c. Instruct the client to increase water intake. d. Test a sample of urine for occult blood.

ANS: B Thiazolidinediones (including pioglitazone) can affect liver function; liver function should be assessed at the start of therapy and at regular intervals while the client continues to take these drugs. Dark urine is one indicator of liver impairment because bilirubin is increased in the blood and is excreted in the urine. The nurse should check the client's most recent liver function studies. The nurse does not need to assess for pain or burning with urination and does not need to check the urine for occult blood. The client does not need to be told to increase water intake.

A nurse teaches a client with diabetes mellitus about sick day management. Which statement should the nurse include in this client's teaching? a. "When ill, avoid eating or drinking to reduce vomiting and diarrhea." b. "Monitor your blood glucose levels at least every 4 hours while sick." c. "If vomiting, do not use insulin or take your oral antidiabetic agent." d. "Try to continue your prescribed exercise regimen even if you are sick."

ANS: B When ill, the client should monitor his or her blood glucose at least every 4 hours. The client should continue taking the medication regimen while ill. The client should continue to eat and drink as tolerated but should not exercise while sick.

A nurse assesses a client who has diabetes mellitus. Which arterial blood gas values should the nurse identify as potential ketoacidosis in this client? a. pH 7.38, HCO3- 22 mEq/L, PCO2 38 mm Hg, PO2 98 mm Hg b. pH 7.28, HCO3- 18 mEq/L, PCO2 28 mm Hg, PO2 98 mm Hg c. pH 7.48, HCO3- 28 mEq/L, PCO2 38 mm Hg, PO2 98 mm Hg d. pH 7.32, HCO3- 22 mEq/L, PCO2 58 mm Hg, PO2 88 mm Hg

ANS: B When the lungs can no longer offset acidosis, the pH decreases to below normal. A client who has diabetic ketoacidosis would present with arterial blood gas values that show primary metabolic acidosis with decreased bicarbonate levels and a compensatory respiratory alkalosis with decreased carbon dioxide levels.

A nurse teaches an older adult with a decreased production of estrogen. Which statement should the nurse include in this client's teaching to decrease injury? a. "Drink at least 2 liters of fluids each day." b. "Walk around the neighborhood for daily exercise." c. "Bathe your perineal area twice a day." d. "You should check your blood glucose before meals."

ANS: B An older adult client with decreased production of estrogen is at risk for decreased bone density and fractures. The nurse should encourage the client to participate in weight-bearing exercises such as walking. Drinking fluids and performing perineal care will decrease vaginal drying but not decrease injury. Older adults often have a decreased glucose tolerance, but this is not related to a decrease in estrogen.

A nurse cares for a client who has excessive catecholamine release. Which assessment finding should the nurse correlate with this condition? a. Decreased blood pressure b. Increased pulse c. Decreased respiratory rate d. Increased urine output

ANS: B Catecholamines are responsible for the fight-or-flight stress response. Activation of the sympathetic nervous system can be correlated with tachycardia. Catecholamines do not decrease blood pressure or respiratory rate, nor do they increase urine output.

A patient who has been diagnosed with Graves' disease is going to receive radioactive iodine (RAI) in the oral form of 131I. What does the nurse teach the patient about how this drug works? a. It destroys the hormones T3 and T4. b. It destroys the tissue that produces thyroid hormones. c. It blocks thyroid hormone production. d. It prevents T4 from being converted to T3.

b

A nurse assesses a female client who presents with hirsutism. Which question should the nurse ask when assessing this client? a. "How do you plan to pay for your treatments?" b. "How do you feel about yourself?" c. "What medications are you prescribed?" d. "What are you doing to prevent this from happening?"

ANS: B Hirsutism, or excessive hair growth on the face and body, can result from endocrine disorders. This may cause a disruption in body image, especially for female clients. The nurse should inquire into the client's body image and self-perception. Asking about the client's financial status or current medications does not address the client's immediate problem. The client is not doing anything to herself to cause the problem, nor can the client prevent it from happening.

A nurse cares for a client who is prescribed a drug that blocks a hormone's receptor site. Which therapeutic effect should the nurse expect? a. Greater hormone metabolism b. Decreased hormone activity c. Increased hormone activity d. Unchanged hormone response

ANS: B Hormones cause activity in the target tissues by binding with their specific cellular receptor sites, thereby changing the activity of the cell. When receptor sites are occupied by other substances that block hormone binding, the cell's response is the same as when the level of the hormone is decreased.

A nurse assesses a client who is being treated for hyperglycemic-hyperosmolar state (HHS). Which clinical manifestation indicates to the nurse that the therapy needs to be adjusted? a. Serum potassium level has increased. b. Blood osmolarity has decreased. c. Glasgow Coma Scale score is unchanged. d. Urine remains negative for ketone bodies.

ANS: C A slow but steady improvement in central nervous system functioning is the best indicator of therapy effectiveness for HHS. Lack of improvement in the level of consciousness may indicate inadequate rates of fluid replacement. The Glasgow Coma Scale assesses the client's state of consciousness against criteria of a scale including best eye, verbal, and motor responses. An increase in serum potassium, decreased blood osmolality, and urine negative for ketone bodies do not indicate adequacy of treatment.

A nurse assesses a client who has a 15-year history of diabetes and notes decreased tactile sensation in both feet. Which action should the nurse take first? a. Document the finding in the client's chart. b. Assess tactile sensation in the client's hands. c. Examine the client's feet for signs of injury. d. Notify the health care provider.

ANS: C Diabetic neuropathy is common when the disease is of long duration. The client is at great risk for injury in any area with decreased sensation because he or she is less able to feel injurious events. Feet are common locations for neuropathy and injury, so the nurse should inspect them for any signs of injury. After assessment, the nurse should document findings in the client's chart. Testing sensory perception in the hands may or may not be needed. The health care provider can be notified after assessment and documentation have been completed.

After teaching a client who is newly diagnosed with type 2 diabetes mellitus, the nurse assesses the client's understanding. Which statement made by the client indicates a need for additional teaching? a. "I should increase my intake of vegetables with higher amounts of dietary fiber." b. "My intake of saturated fats should be no more than 10% of my total calorie intake." c. "I should decrease my intake of protein and eliminate carbohydrates from my diet." d. "My intake of water is not restricted by my treatment plan or medication regimen."

ANS: C The client should not completely eliminate carbohydrates from the diet, and should reduce protein if microalbuminuria is present. The client should increase dietary intake of complex carbohydrates, including vegetables, and decrease intake of fat. Water does not need to be restricted unless kidney failure is present.

A nurse assesses a client with diabetes mellitus 3 hours after a surgical procedure and notes the client's breath has a "fruity" odor. Which action should the nurse take? a. Encourage the client to use an incentive spirometer. b. Increase the client's intravenous fluid flow rate. c. Consult the provider to test for ketoacidosis. d. Perform meticulous pulmonary hygiene care.

ANS: C The stress of surgery increases the action of counterregulatory hormones and suppresses the action of insulin, predisposing the client to ketoacidosis and metabolic acidosis. One manifestation of ketoacidosis is a "fruity" odor to the breath. Documentation should occur after all assessments have been completed. Using an incentive spirometer, increasing IV fluids, and performing pulmonary hygiene will not address this client's problem.

A nurse cares for a client who is prescribed a serum catecholamine test. Which action should the nurse take when obtaining the sample? a. Discard the first sample and then begin the collection. b. Draw the blood sample after the client eats breakfast. c. Place the sample on ice and send to the laboratory immediately. d. Add preservatives before sending the sample to the laboratory.

ANS: C A blood sample for catecholamine must be placed on ice and taken to the laboratory immediately. This sample is not urine, and therefore the first sample should not be discarded nor should preservatives be added to the sample. The nurse should use the appropriate tube and obtain the sample based on which drugs are administered, not dietary schedules.

A nurse evaluates laboratory results for a male client who reports fluid secretion from his breasts. Which hormone value should the nurse assess first? a. Posterior pituitary hormones b. Adrenal medulla hormones c. Anterior pituitary hormones d. Parathyroid hormone

ANS: C Breast fluid and milk production are induced by the presence of prolactin, secreted from the anterior pituitary gland. The other hormones would not cause fluid secretion from the client's breast.

When teaching a client recently diagnosed with type 1 diabetes mellitus, the client states, "I will never be able to stick myself with a needle." How should the nurse respond? a. "I can give your injections to you while you are here in the hospital." b. "Everyone gets used to giving themselves injections. It really does not hurt." c. "Your disease will not be managed properly if you refuse to administer the shots." d. "Tell me what it is about the injections that are concerning you."`

ANS: D Devote as much teaching time as possible to insulin injection and blood glucose monitoring. Clients with newly diagnosed diabetes are often fearful of giving themselves injections. If the client is worried about giving the injections, it is best to try to find out what specifically is causing the concern, so it can be addressed. Giving the injections for the client does not promote self-care ability. Telling the client that others give themselves injections may cause the client to feel bad. Stating that you don't know another way to manage the disease is dismissive of the client's concerns.

After teaching a client who has diabetes mellitus and proliferative retinopathy, nephropathy, and peripheral neuropathy, the nurse assesses the client's understanding. Which statement made by the client indicates a correct understanding of the teaching? a. "I have so many complications; exercising is not recommended." b. "I will exercise more frequently because I have so many complications." c. "I used to run for exercise; I will start training for a marathon." d. "I should look into swimming or water aerobics to get my exercise."

ANS: D Exercise is not contraindicated for this client, although modifications based on existing pathology are necessary to prevent further injury. Swimming or water aerobics will give the client exercise without the worry of having the correct shoes or developing a foot injury. The client should not exercise too vigorously.

A nurse reviews the medication list of a client recovering from a computed tomography (CT) scan with IV contrast to rule out small bowel obstruction. Which medication should alert the nurse to contact the provider and withhold the prescribed dose? a. Pioglitazone (Actos) b. Glimepiride (Amaryl) c. Glipizide (Glucotrol) d. Metformin (Glucophage)

ANS: D Glucophage should not be administered when the kidneys are attempting to excrete IV contrast from the body. This combination would place the client at high risk for kidney failure. The nurse should hold the metformin dose and contact the provider. The other medications are safe to administer after receiving IV contrast.

A nurse teaches a client who is prescribed an insulin pump. Which statement should the nurse include in this client's discharge education? a. "Test your urine daily for ketones." b. "Use only buffered insulin in your pump." c. "Store the insulin in the freezer until you need it." d. "Change the needle every 3 days."

ANS: D Having the same needle remain in place through the skin for longer than 3 days drastically increases the risk for infection in or through the delivery system. Having an insulin pump does not require the client to test for ketones in the urine. Insulin should not be frozen. Insulin is not buffered.

A nurse reviews laboratory results for a client with diabetes mellitus who presents with polyuria, lethargy, and a blood glucose of 560 mg/dL. Which laboratory result should the nurse correlate with the client's polyuria? a. Serum sodium: 163 mEq/L b. Serum creatinine: 1.6 mg/dL c. Presence of urine ketone bodies d. Serum osmolarity: 375 mOsm/kg

ANS: D Hyperglycemia causes hyperosmolarity of extracellular fluid. This leads to polyuria from an osmotic diuresis. The client's serum osmolarity is high. The client's sodium would be expected to be high owing to dehydration. Serum creatinine and urine ketone bodies are not related to the polyuria.

A nurse reviews the laboratory results of a client who is receiving intravenous insulin. Which should alert the nurse to intervene immediately? a. Serum chloride level of 98 mmol/L b. Serum calcium level of 8.8 mg/dL c. Serum sodium level of 132 mmol/L d. Serum potassium level of 2.5 mmol/L

ANS: D Insulin activates the sodium-potassium ATPase pump, increasing the movement of potassium from the extracellular fluid into the intracellular fluid, resulting in hypokalemia. In hyperglycemia, hypokalemia can also result from excessive urine loss of potassium. The chloride level is normal. The calcium and sodium levels are slightly low, but this would not be related to hyperglycemia and insulin administration.

A nurse teaches a client with type 1 diabetes mellitus. Which statement should the nurse include in this client's teaching to decrease the client's insulin needs? a. "Limit your fluid intake to 2 liters a day." b. "Animal organ meat is high in insulin." c. "Limit your carbohydrate intake to 80 grams a day." d. "Walk at a moderate pace for 1 mile daily."

ANS: D Moderate exercise such as walking helps regulate blood glucose levels on a daily basis and results in lowered insulin requirements for clients with type 1 diabetes mellitus. Restricting fluids and eating organ meats will not reduce insulin needs. People with diabetes need at least 130 grams of carbohydrates each day.

After teaching a client with type 2 diabetes mellitus who is prescribed nateglinide (Starlix), the nurse assesses the client's understanding. Which statement made by the client indicates a correct understanding of the prescribed therapy? a. "I'll take this medicine during each of my meals." b. "I must take this medicine in the morning when I wake." c. "I will take this medicine before I go to bed." d. "I will take this medicine immediately before I eat."

ANS: D Nateglinide is an insulin secretagogue that is designed to increase meal-related insulin secretion. It should be taken immediately before each meal. The medication should not be taken without eating as it will decrease the client's blood glucose levels. The medication should be taken before meals instead of during meals.

A nurse assesses a client with diabetes mellitus who self-administers subcutaneous insulin. The nurse notes a spongy, swelling area at the site the client uses most frequently for insulin injection. Which action should the nurse take? a. Apply ice to the site to reduce inflammation. b. Consult the provider for a new administration route. c. Assess the client for other signs of cellulitis. d. Instruct the client to rotate sites for insulin injection.

ANS: D The client's tissue has been damaged from continuous use of the same site. The client should be educated to rotate sites. The damaged tissue is not caused by cellulitis or any type infection, and applying ice may cause more damage to the tissue. Insulin can only be administered subcutaneously and intravenously. It would not be appropriate or practical to change the administration route.

A nurse cares for a client who has type 1 diabetes mellitus. The client asks, "Is it okay for me to have an occasional glass of wine?" How should the nurse respond? a. "Drinking any wine or alcohol will increase your insulin requirements." b. "Because of poor kidney function, people with diabetes should avoid alcohol." c. "You should not drink alcohol because it will make you hungry and overeat." d. "One glass of wine is okay with a meal and is counted as two fat exchanges."

ANS: D Under normal circumstances, blood glucose levels will not be affected by moderate use of alcohol when diabetes is well controlled. Because alcohol can induce hypoglycemia, it should be ingested with or shortly after a meal. One alcoholic beverage is substituted for two fat exchanges when caloric intake is calculated. Kidney function is not impacted by alcohol intake. Alcohol is not associated with increased hunger or overeating.

After teaching a young adult client who is newly diagnosed with type 1 diabetes mellitus, the nurse assesses the client's understanding. Which statement made by the client indicates a correct understanding of the need for eye examinations? a. "At my age, I should continue seeing the ophthalmologist as I usually do." b. "I will see the eye doctor when I have a vision problem and yearly after age 40." c. "My vision will change quickly. I should see the ophthalmologist twice a year." d. "Diabetes can cause blindness, so I should see the ophthalmologist yearly."

ANS: D Diabetic retinopathy is a leading cause of blindness in North America. All clients with diabetes, regardless of age, should be examined by an ophthalmologist (rather than an optometrist or optician) at diagnosis and at least yearly thereafter.

A nurse prepares to palpate a client's thyroid gland. Which action should the nurse take when performing this assessment? a. Stand in front of the client instead of behind the client. b. Ask the client to swallow after palpating the thyroid. c. Palpate the right lobe with the nurse's left hand. d. Place the client in a sitting position with the chin tucked down.

ANS: D The client should be in a sitting position with the chin tucked down as the examiner stands behind the client. The nurse feels for the thyroid isthmus while the client swallows and turns the head to the right, and the nurse palpates the right lobe with the right hand. The technique is repeated in the opposite fashion for the left lobe.

Ch.62 p. 1280, Health Promotion and Maintenance The client who is about to have a unilateral adrenalectomy for an adenoma that is causing hypercortisolism asks the nurse if she will have to continue the severe sodium restriction after surgery. What is the nurse's best response? A. "No, once the tumor has been removed and your cortisol levels have normalized, you will not retain excess sodium anymore." B. "No, after surgery you will have to take oral cortisol, which can easily be controlled so that your sodium levels do not rise." C. Yes, the fact that you are retaining sodium and have high blood pressure is related to your age and lifestyle, not the tumor." D. "Yes, sodium is very bad for people and everyone needs to eliminate sodium completely from their diets for the rest of their lives."

Answer: A Rationale: A tumor secreting excessive amounts of cortisol is this patient's reason for needing to severely restrict her sodium. After the tumor is removed, she will not have hypercortisolism but may have to take oral cortisol until the remaining adrenal gland begins to secrete sufficient cortisol. She will no longer experience severe sodium retention. Although people in North America tend to have high-sodium diets and many could stand to reduce their sodium intake, sodium is an essential element and cannot be eliminated from the diet.

Ch.64 p. 1317, Health Promotion and Maintenance Which statement made by a client who is learning about self-injection of insulin indicates to the nurse that clarification is needed about injection site selection and rotation? A. "The abdominal site is best because it is closest to the pancreas." B. "I can reach my thigh best, so I will use different areas of the same thigh." C. "By rotating sites within one area, my chance of having skin changes is less." D. "If I change my injection site from the thigh to an arm, the insulin absorption may be different."

Answer: A Rationale: The abdominal site has the fastest and most consistent rate of absorption because of the blood vessels in the area, not because of its proximity to the pancreas.

Ch.63 p. 1288, Physiological Integrity Which manifestations are most often seen in general hyperthyroidism? Select all that apply. A. Increased appetite B. Cold intolerance C. Constipation D. Increased sweating E. Insomnia F. Palpitations G. Tremors H. Weight gain

Answer: A, D, E, F, G Rationale: The person with any type of hyperthyroidism has increased metabolism, which causes an increased appetite, increased sweating, increased nervous system stimulation (tremors and insomnia), and increased cardiovascular responses (palpitations). In most people with hyperthyroidism, all other systems are also stimulated, causing increased bowel movements (not constipation) and an elevated body temperature (not cold intolerance). Even though appetite is increased, most people lose weight.

Ch.62 p. 1273, Physiological Integrity Which urine properties indicate to the nurse that the client with syndrome of inappropriate (SIADH) antidiuretic hormone is responding to interventions? A. Urine output volume increased; urine specific gravity increased B. Urine output volume increased; urine specific gravity decreased C. Urine output volume decreased; urine specific gravity increased D. Urine output volume decreased; urine specific gravity decreased

Answer: B Rationale: SIADH involves excessive secretion of vasopressin (ADH) when it is not needed. Water is reabsorbed, causing an increase in blood volume and a decrease in urine volume. Blood concentration is diluted, and urine concentration, as measured by specific gravity, is highly increased. When interventions to counter act SIADH are effective, the person slows water reabsorption so that urine output volume increases at the same time that urine concentration decreases, seen as a decreased urine specific gravity.

Ch.64 p. 1328, Health Promotion and Maintenance While assessing the client who has had diabetes for 15 years, the nurse finds that he has decreased sensory perception in both feet. What is the nurse's best first action? A. Document the finding as the only action. B. Examine the feet for manifestations of injury. C. Test the sensory perception of the client's hands. D. Tell the client that he now has peripheral neuropathy.

Answer: B Rationale: When reduced peripheral sensory perception is present, the likelihood of injury is high. Any open area or other problem on the foot of a person with diabetes is at great risk for infection and must be managed carefully and quickly. Checking for sensory perception on the hands and other areas is important but can come after a thorough foot examination.

Ch.63 p. 1291, Safe and Effective Care Environment For which assessment finding in a client who has severe hyperthyroidism does the nurse notify the Rapid Response Team? A. An increase in premature ventricular heart contractions from 4 per minute to 5 per minute B. An increase in or widening of pulse pressure from 40 mm Hg to 46 mm Hg C. An increase in temperature from 99.5° F (37.5° C) to 101.3° F (38.5° C) D. An increase of 20 mL of urine output per hour

Answer: C Rationale: Although all changes listed are concerning, the one most associated with impending thyroid storm (thyroid crisis) is the increase in body temperature. This client requires immediate attention.

Ch.64 p. 1306, Physiological Integrity Which health problems are considered results of microvascular complications from long-term or poorly controlled diabetes mellitus? A. Obesity and hyperglycemia B. Systolic hypertension and heart failure C. Retinal hemorrhage and male erectile dysfunction D. Diabetic ketoacidosis and hyperglycemic-hyperosmolar state

Answer: C Rationale: Both retinal hemorrhage and male erectile dysfunction are caused by microvascular complications. Structural problems in retinal vessels include areas of poor retinal circulation, edema, hard fatty deposits in the eye, and retinal hemorrhages. Microvascular changes cause hypoxia and death of the nerves needed for male erection. Systolic hypertension and heart failure are considered macrovascular complications. Obesity and hyperglycemia are causes of microvascular complications and are not caused by them. Diabetic ketoacidosis and hyperglycemic-hyperosmolar state are problems of hyperglycemia but are not caused by microvascular changes.

Ch.62 p. 1268, Safe and Effective Care Environment For which client does the nurse question the prescription of androgen replacement therapy? A. 35-year-old man who has had a vasectomy B. 48-year old man who takes prednisone for severe asthma C. 62-year-old man who has a history of prostate cancer D. 70-year-old man who has hypertension and type 2 diabetes

Answer: C Rationale: Prostate cancer tends to increase its growth rate in the presence of any type of androgen. Thus, the man who has a history of prostate cancer should avoid exogenous androgen because it could enhance the growth if the previously treated cancer returns. None of the other conditions are contraindicated for androgen replacement therapy.

p. 1264, Health Promotion and Maintenance Which precaution or action is most important for the nurse to teach the client who is to collect a 24-hour urine specimen for endocrine testing? A. Eat a normal diet during the collection period. B. Wear gloves when you urinate to prevent contamination of the specimen. C. Urinate at the end of 24 hours and add that sample to the collection container. D. Avoid walking, running, dancing, or any vigorous exercise during the collection period.

Answer: C Rationale: When a 24-hour urine specimen is started, the specimen should reflect all the urine produced during the specified time. The very first voiding is discarded because the urine has spent some time in the bladder and will not reflect what is happening during the actual 24 hours of the collection. The time of this discard is the beginning of the 24-hour collection period. The test requires that all urine voided after the start time be collected, including the specimen collected by emptying the bladder at end of the 24 hours, which marks the end of the test.

The nurse assessing a patient palpates enlargement of the thyroid gland, along with noticeable swelling of the neck. How does the nurse interpret this finding? a. Globe lag b. Myxedema c. Exophthalmos d. Goiter

d

Ch.61 p. 1259, Physiological Integrity What effect on circulating levels of sodium and glucose does the nurse expect in a client who has been taking an oral cortisol preparation for 2 years because of a respiratory problem? A. Decreased sodium; decreased glucose B. Decreased sodium; increased glucose C. Increased sodium; decreased glucose D. Increased sodium; increased glucose

Answer: D Rationale: Any of the glucocorticoids have some mineralocorticoid activity and increase the reabsorption of sodium from the kidney tubules, thus increasing the serum sodium level. Cortisol also increases liver production of glucose (gluconeogenesis) and inhibits peripheral glucose uptake by the cells. Both of these actions increase blood glucose levels.

p. 1259, Physiological Integrity What effect on circulating levels of sodium and glucose does the nurse expect in a client who has been taking an oral cortisol preparation for 2 years because of a respiratory problem? A. Decreased sodium; decreased glucose B. Decreased sodium; increased glucose C. Increased sodium; decreased glucose D. Increased sodium; increased glucose

Answer: D Rationale: Any of the glucocorticoids have some mineralocorticoid activity and increase the reabsorption of sodium from the kidney tubules, thus increasing the serum sodium level. Cortisol also increases liver production of glucose (gluconeogenesis) and inhibits peripheral glucose uptake by the cells. Both of these actions increase blood glucose levels.

Ch.64 p. 1302, Physiological Integrity How is hypoglycemia prevented in the healthy person who does not have diabetes even after fasting for 8 hours? A. Metabolism is so slow when a person sleeps without eating for 8 hours that blood glucose does not enter cells to be used for energy. As a result, hypoglycemia does not occur. B. Fasting for 8 hours triggers conversion of proteins into glycogen (glycogenesis) so that hyperglycemia develops rather than hypoglycemia. C. Lipolysis (fat breakdown) in fat stores occurs, converting fatty acids into glucose to maintain blood glucose levels. D. The secretion of glucagon prevents hypoglycemia by promoting glucose release from liver storage sites.

Answer: D Rationale: Glucagon is a counterregulatory hormone secreted by pancreatic alpha cells when blood glucose levels are low, as they would be during an 8-hour fast. The body's metabolic rate does decrease during sleep (which is not stated in this question) but not sufficiently to prevent hypoglycemia. Glucagon works on the glycogen stored in the liver, breaking it down to glucose (glycogenolysis) molecules that are then released into the blood to maintain blood glucose levels and prevent hypoglycemia. Although proteins can be broken down and converted to glucose, they are not converted to glycogen. Fat breakdown through lipolysis can provide fatty acids for fuel, but this is not glucose, and lipolysis does not occur until all stored glycogen is used.

Ch.63 p. 1298, Safe and Effective Care Environment When taking the blood pressure of a client receiving treatment for hyperparathyroidism, the nurse observes the client's hand to undergo flexion contractions. What is the nurse's interpretation of this observation? A. Hyperphosphatemia B. Hypophosphatemia C. Hypercalcemia D. Hypocalcemia

Answer: D Rationale: Hypocalcemia destabilizes excitable membranes and can lead to muscle twitches, spasms, and tetany. This effect of hypocalcemia is enhanced in the presence of tissue hypoxia. The flexion contractions occurring during blood pressure measurement are indicative of hypocalcemia and referred to as a positive Trousseau's sign.

A nurse assesses clients for potential endocrine dysfunction. Which client is at greatest risk for a deficiency of gonadotropin and growth hormone? a. A 36-year-old female who has used oral contraceptives for 5 years b. A 42-year-old male who experienced head trauma 3 years ago c. A 55-year-old female with a severe allergy to shellfish and iodine d. A 64-year-old male with adult-onset diabetes mellitus

B (Gonadotropin and growth hormone are anterior pituitary hormones. Head trauma is a common cause of anterior pituitary hypofunction. The other factors do not increase the risk of this condition.)

Ch.62 p. 1276, Patient-Centered Care; Quality Improvement; Safety The patient is a 32-year-old woman admitted to your unit after surgery for fractures of the left arm and leg resulting from a car crash. She is awake and able to verify her medical history of rheumatoid arthritis and her usual daily medications. These are 10 mg of prednisone, naproxen 800 mg twice daily, oral contraceptives, calcium 600 mg, and one multiple vitamin tablet. All of these are prescribed for her to receive during her hospitalization. She is concerned about pain management and how long the recovery will be for the fractures. She is friendly, somewhat anxious, asks many questions, and wants to do "her part" to ensure good recovery. Over the next 4 days, she has become quieter, mumbles that her head and stomach hurt, and now does not recognize the assistant who has been providing her daily care. When she receives her medications, she has difficulty picking them up. The nursing assistant remarks that taking her pulse is difficult because it is so slow and irregular. When you assess her, she is so weak that she is unable to lift her arm for a blood pressure check. Her blood pressure is 92/50, which is down from the 128/84 reading on admission. You also verify that her heart beat is slow and irregular. 5. What could be done to prevent this problem from happening again?

Any patient who routinely takes a corticosteroid should be automatically evaluated on a daily basis for manifestations of adrenal insufficiency. Ideally, the person would receive additional corticosteroid therapy in advance of changes to prevent adrenal insufficiency. At the very least, assessing for early manifestations could have identified this problem earlier and prevented a near tragedy.

A nurse is caring for a client who was prescribed high-dose corticosteroid therapy for 1 month to treat a severe inflammatory condition. The clients symptoms have now resolved and the client asks, When can I stop taking these medications? How should the nurse respond? a. It is possible for the inflammation to recur if you stop the medication. b. Once you start corticosteroids, you have to be weaned off them. c. You must decrease the dose slowly so your hormones will work again. d. The drug suppresses your immune system, which must be built back up.

B (One of the most common causes of adrenal insufficiency, a life-threatening problem, is the sudden cessation of long-term, high-dose corticosteroid therapy. This therapy suppresses the hypothalamic-pituitary-adrenal axis and must be withdrawn gradually to allow for pituitary production of adrenocorticotropic hormone and adrenal production of cortisol. Decreasing hormone therapy slowly ensures self-production of hormone, not hormone effectiveness. Building the clients immune system and rebound inflammation are not concerns related to stopping high-dose corticosteroids.)

A nurse teaches a client with a cortisol deficiency who is prescribed prednisone (Deltasone). Which statement should the nurse include in this clients instructions? a. You will need to learn how to rotate the injection sites. b. If you work outside in the heat, you may need another drug. c. You need to follow a diet with strict sodium restrictions. d. Take one tablet in the morning and two tablets at night.

B (Steroid dosage adjustment may be needed if the client works outdoors and might be difficult, especially in hot weather, when the client is sweating a great deal more than normal. Clients take prednisone orally, have no need for a salt restriction, and usually start the regimen with two tablets in the morning and one at night.)

A nurse cares for a male client with hypopituitarism who is prescribed testosterone hormone replacement therapy. The client asks, How long will I need to take this medication? How should the nurse respond? a. When your blood levels of testosterone are normal, the therapy is no longer needed. b. When your beard thickens and your voice deepens, the dose is decreased, but treatment will continue forever. c. When your sperm count is high enough to demonstrate fertility, you will no longer need this therapy. d. With age, testosterone levels naturally decrease, so the medication can be stopped when you are 50 years old

B (Testosterone therapy is initiated with high-dose testosterone derivatives and is continued until virilization is achieved. The dose is then decreased, but therapy continues throughout life. Therapy will continue throughout life; therefore, it will not be discontinued when blood levels are normal, at the age of 50 years, or when sperm counts are high.)

A nurse cares for a client who possibly has syndrome of inappropriate antidiuretic hormone (SIADH). The clients serum sodium level is 114 mEq/L. Which action should the nurse take first? a. Consult with the dietitian about increased dietary sodium. b. Restrict the clients fluid intake to 600 mL/day. c. Handle the client gently by using turn sheets for re-positioning. d. Instruct unlicensed assistive personnel to measure intake and output.

B (With SIADH, clients often have dilutional hyponatremia. The client needs a fluid restriction, sometimes to as little as 500 to 600 mL/24 hr. Adding sodium to the clients diet will not help if he or she is retaining fluid and diluting the sodium. The client is not at increased risk for fracture, so gentle handling is not an issue. The client should be on intake and output; however, this will monitor only the clients intake, so it is not the best answer. Reducing intake will help increase the clients sodium.)

1. A nurse evaluates the following laboratory results for a client who has hypoparathyroidism: Calcium 7.2 mg/dl Sodium 144 mEq/L Magnesium 1.2 mEq/L Potassium 5.7 mEq/L Based on these results, which medications should the nurse anticipate administering? (Select all that apply.) a. Oral potassium chloride b. Intravenous calcium chloride c. 3% normal saline IV solution d. 50% magnesium sulfate e. Oral calcitriol (Rocaltrol)

B, D. The client has hypocalcemia (treated with calcium chloride) and hypomagnesemia (treated with magnesium sulfate). The potassium level is high, so replacement is not needed. The client's sodium level is normal, so hypertonic IV solution is not needed. No information about a vitamin D deficiency is evident, so calcitriol is not needed.

The client is taking fludrocortisone (Florinef) for adrenal hypofunction. The nurse instructs the client to report which symptom while taking this drug? A. Anxiety B. Headache C. Nausea D. Weight loss

B. A side effect of fludrocortisone is hypertension. New onset of headache should be reported, and the client's blood pressure should be monitored. Anxiety is not a side effect of fludrocortisone and is not associated with adrenal hypofunction. Nausea is associated with adrenal hypofunction; it is not a side effect of fludrocortisone. Sodium-related fluid retention and weight gain, not loss, are possible with fludrocortisone therapy.

A client recently admitted with hyperparathyroidism has a very high urine output. Of these actions, what does the nurse do next? A. Calls the health care provider B. Monitors intake and output C. Performs an immediate cardiac assessment D. Slows the rate of IV fluids

B. Diuretic and hydration therapies are used most often for reducing serum calcium levels in clients with hyperparathyroidism. Usually, a diuretic that increases kidney excretion of calcium is used together with IV saline in large volumes to promote renal calcium excretion. The health care provider does not need to be notified in this situation, given the information available in the question. Cardiac assessment is part of the nurse's routine evaluation of the client. Slowing the rate of IV fluids is contraindicated because the client will become dehydrated due to the use of diuretics to increase kidney excretion of calcium.

A client had a parathyroidectomy 18 hours ago. Which finding requires immediate attention? A. Edema at the surgical site B. Hoarseness C. Pain on moving the head Incorrect D. Sore throat

B. Hoarseness or stridor is an indication of respiratory distress and requires immediate attention. Edema at the surgical site of any surgery is an expected finding. Pain when the client moves the head or attempts to lift the head off the bed is an expected finding after a parathyroidectomy. Any time a client has been intubated for surgery, a sore throat is a common occurrence in the postoperative period. This is especially true for clients who have had surgery involving the neck.

A client being treated for hyperthyroidism calls the home health nurse and mentions that his heart rate is slower than usual. What is the nurse's best response? A. Advise the client to go to a calming environment. B. Ask whether the client has increased cold sensitivity or weight gain. C. Instruct the client to see his health care provider immediately. D. Tell the client to check his pulse again and call back later.

B. Increased sensitivity to cold and weight gain are symptoms of hypothyroidism, indicating an overcorrection by the medication. The client must be assessed further because he may require a lower dose of medication. A calming environment will not have any effect on the client's heart rate. The client will want to notify the health care provider about the change in heart rate. If other symptoms such as chest pain, shortness of breath, or confusion accompany the slower heart rate, then the client should see the health care provider immediately. If the client was concerned enough to call because his heart rate was slower than usual, the nurse needs to stay on the phone with the client while he re-checks his pulse. This time could also be spent providing education about normal ranges for that client.

A client has been diagnosed with hypothyroidism. What medication is usually prescribed to treat this disorder? A. Atenolol (Tenormin) B. Levothyroxine sodium (Synthroid) C. Methimazole (Tapazole) D. Propylthiouracil

B. Levothyroxine is a synthetic form of thyroxine (T4) that is used to treat hypothyroidism. Atenolol is a beta blocker that is used to treat cardiovascular disease. Methimazole and propylthiouracil are used to treat hyperthyroidism.

A client presents to the emergency department with acute adrenal insufficiency and the following vital signs: P 118 beats/min, R 18 breaths/min, BP 84/44 mm Hg, pulse oximetry 98%, and T 98.8° F oral. Which nursing intervention is the highest priority for this client? A. Administering furosemide (Lasix) B. Providing isotonic fluids C. Replacing potassium losses D. Restricting sodium

B. Providing isotonic fluid is the priority intervention because this client's vital signs indicate volume loss that may be caused by nausea and vomiting and may accompany acute adrenal insufficiency. Isotonic fluids will be needed to administer IV medications such as hydrocortisone. Furosemide is a loop diuretic, which this client does not need. Potassium is normally increased in acute adrenal insufficiency, but potassium may have been lost if the client has had diarrhea; laboratory work will have to be obtained. Any restrictions, including sodium, should not be started without obtaining laboratory values to establish the client's baseline.

A client has undergone a transsphenoidal hypophysectomy. Which intervention does the nurse implement to avoid increasing intracranial pressure (ICP) in the client? A. Encourages the client to cough and deep-breathe B. Instructs the client not to strain during a bowel movement C. Instructs the client to blow the nose for postnasal drip D. Places the client in the Trendelenburg position

B. Straining during a bowel movement increases ICP and must be avoided. Laxatives may be given and fluid intake encouraged to help with this. Although deep breathing is encouraged, the client must avoid coughing early after surgery because this increases pressure in the incision area and may lead to a cerebrospinal fluid (CSF) leak. If the client has postnasal drip, he or she must inform the nurse and not blow the nose; postnasal drip may indicate leakage of CSF. The head of the bed must be elevated after surgery.

Family members of a client diagnosed with hyperthyroidism are alarmed at the client's frequent mood swings. What is the nurse's response? A. "How does that make you feel?" B. "The mood swings should diminish with treatment." C. "The medications will make the mood swings disappear completely." D. "Your family member is sick. You must be patient."

B. Telling the family that the client's mood swings should diminish over time with treatment will provide information to the family, as well as reassurance. Asking how the family feels is important; however, the response should focus on the client. Any medications or treatment may not completely remove the mood swings associated with hyperthyroidism. The family is aware that the client is sick; telling them to be patient introduces guilt and does not address the family's concerns.

A client admitted with hyperthyroidism is fidgeting with the bedcovers and talking extremely fast. What does the nurse do next? A. Calls the provider B. Encourages the client to rest C. Immediately assesses cardiac status D. Tells the client to slow down

B. The client with hyperthyroidism often has wide mood swings, irritability, decreased attention span, and manic behavior. The nurse should accept the client's behavior and provide a calm, quiet, and comfortable environment. Because the client's behavior is expected, there is no need to call the provider. Monitoring the client's cardiac status is part of the nurse's routine assessment. Telling the client to slow down is unsupportive and unrealistic.

After hospitalization for myxedema, a patient is prescribed thyroid replacement medication. Which statement by the patient demonstrates a correct understanding of this therapy? a. "I'll be taking this medication until my symptoms are completely resolved." b. "I'll be taking thyroid medication for the rest of my life." c. "Now that I'm feeling better, no changes in my medication will be necessary." d. "I'm taking this medication to prevent symptoms of an overactive thyroid gland."

b

After teaching a client with acromegaly who is scheduled for a hypophysectomy, the nurse assesses the clients understanding. Which statement made by the client indicates a need for additional teaching? a. I will no longer need to limit my fluid intake after surgery. b. I am glad no visible incision will result from this surgery. c. I hope I can go back to wearing size 8 shoes instead of size 12. d. I will wear slip-on shoes after surgery to limit bending over.

C (Although removal of the tissue that is oversecreting hormones can relieve many symptoms of hyperpituitarism, skeletal changes and organ enlargement are not reversible. It will be appropriate for the client to drink as needed postoperatively and avoid bending over. The client can be reassured that the incision will not be visible.)

Which action does the postanesthesia care unit (PACU) nurse perform first when caring for a client who has just arrived after a total thyroidectomy? A. Assess the wound dressing for bleeding. B. Give morphine sulfate 4 to 8 mg IV for pain. C. Monitor oxygen saturation using pulse oximetry. Correct D. Support the head and neck with sandbags.

C. Airway assessment and management is always the first priority with every client. This is especially important for a client who has had surgery that involves potential bleeding and edema near the trachea. Assessing the wound dressing for bleeding is a high priority, although this is not the first priority. Pain control and supporting the head and neck with sandbags are important priorities, but can be addressed after airway assessment.

28. Which statements about insulin secretion are correct? (Select all that apply.) a. Insulin levels increase following the ingestion of a meal. b. Insulin is stimulated primarily by fat ingestion. c. Basal levels are secreted continuously. d. Insulin promotes glycogenolysis and gluconeogenesis. e. Carbohydrate intake is the main trigger for insulin secretion.

a, c, e Insulin levels increase following the ingestion of a meal. Basal levels are secreted continuously. Carbohydrate intake is the main trigger for insulin secretion.

A client diagnosed with hyperpituitarism resulting from a prolactin-secreting tumor has been prescribed bromocriptine mesylate (Parlodel). As a dopamine agonist, what effect does this drug have by stimulating dopamine receptors in the brain? A. Decreases the risk for cerebrovascular disease B. Increases the risk for depression C. Inhibits the release of some pituitary hormones D. Stimulates the release of some pituitary hormones

C. Bromocriptine mesylate inhibits the release of both prolactin and growth hormone. It does not decrease the risk for cerebrovascular disease leading to stroke. Increased risk for depression is not associated with the use of bromocriptine mesylate; however, hallucinations have been reported as a side effect. Bromocriptine mesylate does not stimulate the release of any hormones.

Which client does the nurse identify as being at highest risk for acute adrenal insufficiency resulting from corticosteroid use? A. Client with hematemesis, upper epigastric pain for the past 3 days not relieved with food, and melena B. Client with right upper quadrant pain unrelieved for the past 2 days, dark-brown urine, and clay-colored stools C. Client with shortness of breath and chest tightness, nasal flaring, audible wheezing, and oxygen saturation of 85% for the second time this week D. Client with three emergency department visits in the past month for edema, shortness of breath, weight gain, and jugular venous distention

C. Corticosteroids may be used to treat signs and symptoms of asthma, such as shortness of breath and chest tightness, nasal flaring, audible wheezing, and oxygen saturation of 85%. This places the client at risk for adrenal insufficiency. Corticosteroids are not used to treat signs and symptoms of GI bleeding or peptic ulcer disease (hematemesis, upper epigastric pain for the past 3 days not relieved with food, and melena), gallbladder disease (right upper quadrant pain unrelieved for the past 2 days, dark brown urine, and clay-colored stools), or congestive heart failure (edema, shortness of breath, weight gain, and jugular venous distention).

These data are obtained by the RN who is assessing a client who had a transsphenoidal hypophysectomy yesterday. What information has the most immediate implications for the client's care? A. Dry lips and oral mucosa on examination B. Nasal drainage that tests negative for glucose C. Client report of a headache and stiff neck D. Urine specific gravity of 1.016

C. Headache and stiff neck (nuchal rigidity) are symptoms of meningitis that have immediate implications for the client's care. Dry lips and mouth are not unusual after surgery. Frequent oral rinses and the use of dental floss should be encouraged because the client cannot brush the teeth. Any nasal drainage should test negative for glucose; nasal drainage that tests positive for glucose indicates the presence of a cerebrospinal fluid leak. A urine specific gravity of 1.016 is within normal limits.

Which laboratory result indicates that fluid restrictions have been effective in treating syndrome of inappropriate antidiuretic hormone (SIADH)? A. Decreased hematocrit B. Decreased serum osmolality C. Increased serum sodium D. Increased urine specific gravity

C. Increased serum sodium due to fluid restriction indicates effective therapy. Hemoconcentration is a result of hypovolemic hyponatremia caused by SIADH and diabetes insipidus. Plasma osmolality is decreased as a result of SIADH. Urine specific gravity is decreased with diabetes insipidus and is increased with SIADH.

A client with hypothyroidism is being discharged. Which environmental change may the client experience in the home? A. Frequent home care B. Handrails in the bath C. Increased thermostat setting D. Strict infection-control measures

C. Manifestations of hypothyroidism include cold intolerance. Increased thermostat settings or additional clothing may be necessary. A client with a diagnosis of hypothyroidism can be safely managed at home with adequate discharge teaching regarding medications and instructions on when to notify the health care provider or home health nurse. In general, hypothyroidism does not cause mobility issues. Activity intolerance and fatigue may be an issue, however. A client with hypothyroidism is not immune-compromised or contagious, so no environmental changes need to be made to the home.

An older client with an elevated serum calcium level is receiving IV furosemide (Lasix) and an infusion of normal saline at 150 mL/hr. Which nursing action can the RN delegate to unlicensed assistive personnel (UAP)? A. Ask the client about any numbness or tingling. B. Check for bone deformities in the client's back. C. Measure the client's intake and output hourly. D. Monitor the client for shortness of breath.

C. Measuring intake and output is a commonly delegated nursing action that is within the UAP scope of practice. Numbness and tingling is part of the client assessment that needs to be completed by a licensed nurse. Bony deformities can be due to pathologic fractures; physical assessment is a complex task that cannot be delegated. An older client receiving an IV at 150 mL/hr is at risk for congestive heart failure; careful monitoring for shortness of breath is the responsibility of the RN.

An RN and LPN/LVN are caring for a group of clients on the medical-surgical unit. Which client will be the best to assign to the LPN/LVN? A. Client with Graves' disease who needs discharge teaching after a total thyroidectomy B. Client with hyperparathyroidism who is just being admitted for a parathyroidectomy C. Client with infiltrative ophthalmopathy who needs administration of high-dose prednisone (Deltasone) D. Newly diagnosed client with hypothyroidism who needs education about the use of thyroid supplements

C. Medication administration for the client with infiltrative ophthalmopathy is within the scope of practice of the LPN/LVN. Discharge teaching is a complex task that cannot be delegated to the LPN/LVN. A client being admitted for a parathyroidectomy needs preoperative teaching, which must be provided by the RN. A client who has a new diagnosis will have questions about the disease and prescribed medications; teaching is a complex task that is appropriate for the RN.

A client with Cushing's disease begins to laugh loudly and inappropriately, causing the family in the room to be uncomfortable. What is the nurse's best response? A. "Don't mind this. The disease is causing this." B. "I need to check the client's cortisol level." C. "The disease can sometimes affect emotional responses." D. "Medication is available to help with this."

C. The client may have neurotic or psychotic behavior as a result of high blood cortisol levels. Being honest with the family helps them to understand what is happening. Telling the family not to mind the laughter and that the disease is causing it is vague and minimizes the family's concern. This is the perfect opportunity for the nurse to educate the family about the disease. Cushing's disease is the hypersecretion of cortisol, which is abnormally elevated in this disease and, because the diagnosis has already been made, blood levels do not need to be redrawn. Telling the family that medication is available to help with inappropriate laughing does not assist them in understanding the cause of or the reason for the client's behavior.

The nurse is teaching a client about thyroid replacement therapy. Which statement by the client indicates a need for further teaching? A. "I should have more energy with this medication." B. "I should take it every morning." C. "If I continue to lose weight, I may need an increased dose." D. "If I gain weight and feel tired, I may need an increased dose."

C. Weight loss indicates a need for a decreased dose, not an increased dose. One of the symptoms of hypothyroidism is lack of energy; thyroid replacement therapy should help the client have more energy. The correct time to take thyroid replacement therapy is in the morning. If the client is gaining weight and continues to feel tired, that is an indication that the dose may need to be increased.

Ch.64 p. 1335, Prioritization, Delegation, and Supervision The patient, a 21-year-old college student, was brought to the emergency department (ED) by his roommate. He reports abdominal pain, polyuria for the past 2 days, vomiting several times prior to arrival, and extreme thirst. He appears flushed, and his lips and mucous membranes are dry and cracked. His skin turgor is poor. He demonstrates deep rapid respirations; there is a fruit odor to his breath. He has type 1 diabetes and "may have skipped a few doses of insulin because of cramming for final exams." He is alert and talking but is having trouble focusing on your questions. Vital signs: Blood pressure 110/60 Pulse 110/min Respirations 32/min Temperature 100.8° F Fingerstick glucose 485 mg/dL Oxygen saturation 99% 3. Should you call his primary health provider? Why or why not?

Calling the primary health care provider is usually done by the ED physician after the preliminary work-up is completed and interventions have started. (Policies for calling private physicians may vary among institutions. Be sure to check the policy at your facility.)

Ch.64 p. 1332, Safety; Quality Improvement; Teamwork and Collaboration The patient is a 60 year-old-woman who is 1 day postoperative after a total knee replacement. She has type 2 diabetes and just recently was switched from oral antidiabetic drugs to an insulin regimen. She let her nurse know that her on-demand lunch has been ordered. The nurse tests her blood and gives her the prescribed short-acting insulin dose. An hour later, the physical therapist finds her pale, confused, and clammy. Her lunch tray is on her table and appears totally untouched. 2. What is your first action? Provide a rationale.

Check her blood glucose level immediately because the methods to increase her blood glucose level are dependent on how low the current level is. a. As an alternative, if there is an easily digestible carbohydrate on her tray and she is able to swallow, you could give that to her immediately and then obtain a blood glucose measurement. However, this is less precise.

Ch.63 p. 1295, Patient-Centered Care; Safety The patient, a 45-year-old former school teacher, is residing in a skilled nursing facility to recover from a tibia-fibula fracture that is being managed with an external fixation system. On admission 2 weeks ago, she told you that she felt she was "getting old too fast." She explained that she had gained 54 pounds in the previous 6 months, had no energy, was often constipated, and was always cold. She teared up and said that her ability to concentrate was so bad that not only could she no longer help her high school children with their homework, but that she didn't recognize the step hazard that caused her to fall and break her ankle. Today the nursing assistant assigned to her care reports that the patient's pulse is only 42 beats per minute and that her temperature was 96° F even with two blankets. When you enter her room, she is sleeping with an untouched breakfast tray on her table. 1. What are the priority assessment data you should obtain? Provide a rationale for your choices.

Check her respiratory rate and effectiveness (use pulse oximetry). Then recheck her heart rate (apically) and blood pressure. She has indications of severe hypothyroidism. Respiratory arrest is a potential cause of death for patients with this problem. Assess her level of consciousness. Coma is possible, especially if she received any opioid drugs for pain control.

Ch.64 p. 1332, Safety; Quality Improvement; Teamwork and Collaboration The patient is a 60 year-old-woman who is 1 day postoperative after a total knee replacement. She has type 2 diabetes and just recently was switched from oral antidiabetic drugs to an insulin regimen. She let her nurse know that her on-demand lunch has been ordered. The nurse tests her blood and gives her the prescribed short-acting insulin dose. An hour later, the physical therapist finds her pale, confused, and clammy. Her lunch tray is on her table and appears totally untouched. 3. What is the most likely cause leading to this problem?

Clearly, there was a delay in eating after receiving the insulin. The tray may have been delayed longer than expected from food service, or perhaps she decided she was not hungry when it first arrived. She could have been interrupted (possible phone call or visitor) before she had a chance to eat it. In addition, it is possible because she has only recently been started on insulin that she did not understand the necessity of eating soon after receiving insulin.

A nurse cares for a client after a pituitary gland stimulation test using insulin. The clients post-stimulation laboratory results indicate elevated levels of growth hormone (GH) and adrenocorticotropic hormone (ACTH). How should the nurse interpret these results? a. Pituitary hypofunction b. Pituitary hyperfunction c. Pituitary-induced diabetes mellitus d. Normal pituitary response to insulin

D (Some tests for pituitary function involve administering agents that are known to stimulate the secretion of specific pituitary hormones and then measuring the response. Such tests are termed stimulation tests. The stimulation test for GH or ACTH assessment involves injecting the client with regular insulin (0.05 to 1 unit/kg of body weight) and checking circulating levels of GH and ACTH. The presence of insulin in clients with normal pituitary function causes increased release of GH and ACTH.)

The RN has just received change-of-shift report on the medical-surgical unit. Which client will need to be assessed first? A. Client with Hashimoto's thyroiditis and a large goiter B. Client with hypothyroidism and an apical pulse of 51 beats/min C. Client with parathyroid adenoma and flank pain due to a kidney stone D. Client who had a parathyroidectomy yesterday and has muscle twitching

D. A client who is 1 day postoperative for parathyroidectomy and has muscle twitching is showing signs of hypocalcemia and is at risk for seizures. Rapid assessment and intervention are needed. Clients with Hashimoto's thyroiditis are usually stable; this client does not need to be assessed first. Although an apical pulse of 51 is considered bradycardia, a low heart rate is a symptom of hypothyroidism. A client with a kidney stone will be uncomfortable and should be asked about pain medication as soon as possible, but this client does not need to be assessed first.

The nurse is caring for a client with hypercortisolism. The nurse begins to feel the onset of a cold but still has 4 hours left in the shift. What does the nurse do? A. Asks another nurse to care for the client B. Monitors the client for cold-like symptoms C. Refuses to care for the client D. Wears a facemask when caring for the client

D. A client with hypercortisolism will be immune-suppressed. Anyone with a suspected upper respiratory infection who must enter the client's room must wear a mask to prevent the spread of infection. Although asking another nurse to care for the client might be an option in some facilities, it is not generally realistic or practical. The nurse, not the client, feels the onset of the cold, so monitoring the client for cold-like symptoms is part of good client care for a client with hypercortisolism. Refusing to care for the client after starting care would be considered abandonment.

The charge nurse is making client assignments for the medical-surgical unit. Which client will be best to assign to an RN who has floated from the pediatric unit? A. Client in Addisonian crisis who is receiving IV hydrocortisone B. Client admitted with syndrome of inappropriate antidiuretic hormone (SIADH) secondary to lung cancer C. Client being discharged after a unilateral adrenalectomy to remove an adrenal tumor D. Client with Cushing's syndrome who has elevated blood glucose and requires frequent administration of insulin

D. An RN who works with pediatric clients would be familiar with glucose monitoring and insulin administration. A client in Addisonian crisis would best be monitored by an RN from the medical-surgical floor. Although the float RN could complete the admission history, the client with SIADH secondary to lung cancer might require teaching and orientation to the unit that a nurse more familiar with that area would be better able to provide. Discharge teaching specific to adrenalectomy should be provided by the RN who is regularly assigned to the medical-surgical floor and is more familiar with taking care of postoperative adult clients with endocrine disorders.

A client is referred to a home health agency after a transsphenoidal hypophysectomy. Which action does the RN case manager delegate to the home health aide who will see the client daily? A. Document symptoms of incisional infection or meningitis. B. Give over-the-counter laxatives if the client is constipated. C. Set up medications as prescribed for the day. D. Test any nasal drainage for the presence of glucose.

D. Cerebrospinal fluid (CSF) will test positive using a glucose "dipstick." Nasal drainage that is positive for glucose after a transsphenoidal hypophysectomy would indicate a CSF leak that would require immediate notification of the health care provider. Home health aides can be taught the correct technique to perform this procedure. Assessing for symptoms of infection and documenting them in the record, medication administration, and setting up medication are not within the scope of practice of the home health aide.

A client with thyroid cancer has just received 131I ablative therapy. Which statement by the client indicates a need for further teaching? A. "I cannot share my toothpaste with anyone." B. "I must flush the toilet three times after I use it." C. "I need to wash my clothes separately from everyone else's clothes." D. "I'm ready to hold my newborn grandson now."

D. Clients undergoing 131I therapy should avoid close contact with pregnant women, infants, and young children for 1 week after treatment. Clients should remain at least 1 meter (39 inches, or roughly 3 feet) away, and limit exposure to less than 1 hour per day. Some radioactivity will remain in the client's salivary glands for up to 1 week after treatment. Care should be taken to avoid exposing others to the saliva. Flushing the toilet three times after use will ensure that all urine has been diluted and removed. Clothing needs to be washed separately and the washing machine then needs to be run empty for a full cycle before it is used to wash the clothing of others.

How does the drug desmopressin (DDAVP) decrease urine output in a client with diabetes insipidus (DI)? A. Blocks reabsorption of sodium B. Increases blood pressure C. Increases cardiac output D. Works as an antidiuretic hormone (ADH) in the kidneys

D. Desmopressin is a synthetic form of ADH that binds to kidney receptors and enhances reabsorption of water, thus reducing urine output. Desmopressin does not have any effect on sodium reabsorption. It may cause a slight increase or a transient decrease in blood pressure, but this does not affect urine output. Desmopressin does not increase cardiac output.

A client with a possible adrenal gland tumor is admitted for testing and treatment. Which nursing action is most appropriate for the charge nurse to delegate to the nursing assistant? A. Assess skin turgor and mucous membranes for hydration status. B. Discuss the dietary restrictions needed for 24-hour urine testing. C. Plan ways to control the environment that will avoid stimulating the client. D. Remind the client to avoid drinking coffee and changing position suddenly.

D. Drinking caffeinated beverages and changing position suddenly are not safe for a client with a potential adrenal gland tumor because of the effects of catecholamines. Reminding the client about previous instructions is an appropriate role for a nursing assistant who may observe the client doing potentially risky activities. Client assessment, client teaching, and environment planning are higher-level skills that require the experience and responsibility of the RN, and are not within the scope of practice of the nursing assistant.

A client with Cushing's disease says that she has lost 1 pound. What does the nurse do next? A. Auscultates the lungs for crackles B. Checks urine for specific gravity C. Forces fluids D. Weighs the client

D. Fluid retention with weight gain is more of a problem than weight loss in clients with Cushing's disease. Weighing the client with Cushing's disease is part of the nurse's assessment. Crackles in the lungs indicate possible fluid retention, which would cause weight gain, not weight loss. Urine specific gravity will help assess hydration status, but this would not be the next step in the client's assessment. Forcing fluids is not appropriate because usually excess water and sodium reabsorption cause fluid retention in the client with Cushing's disease.

What effect can starting a dose of levothyroxine sodium (Synthroid) too high or increasing a dose too rapidly have on a client? A. Bradycardia and decreased level of consciousness B. Decreased respiratory rate C. Hypotension and shock D. Hypertension and heart failure

D. Hypertension and heart failure are possible if the levothyroxine sodium dose is started too high or raised too rapidly, because levothyroxine would essentially put the client into a hyperthyroid state. The client would be tachycardic, not bradycardic. The client may have an increased respiratory rate. Shock may develop, but only as a late effect and as the result of "pump failure."

The nurse reviews the vital signs of a client diagnosed with Graves' disease and sees that the client's temperature is up to 99.6° F. After notifying the health care provider, what does the nurse do next? A. Administers acetaminophen B. Alerts the Rapid Response Team C. Asks any visitors to leave D. Assesses the client's cardiac status completely

D. If the client's temperature has increased by even 1°, the nurse's first action is to notify the provider. Continuous cardiac monitoring should be the next step. Administering a nonsalicylate antipyretic such as acetaminophen is appropriate, but is not a priority action for this client. Alerting the Rapid Response Team is not needed at this time. Asking visitors to leave would not be the next action, and if visitors are providing comfort to the client, this would be contraindicated.

A client is taking methimazole (Tapazole) for hyperthyroidism and would like to know how soon this medication will begin working. What is the nurse's best response? A. "You should see effects of this medication immediately." B. "You should see effects of this medication within 1 week." C. "You should see full effects from this medication within 1 to 2 days." D. "You should see some effects of this medication within 2 weeks."

D. Methimazole is an iodine preparation that decreases blood flow through the thyroid gland. This action reduces the production and release of thyroid hormone. The client should see some effects within 2 weeks; however, it may take several more weeks before metabolism returns to normal. Although onset of action is 30 to 40 minutes after an oral dose, the client will not see effects immediately. Effects will take longer than 1 week to become apparent when methimazole is used. Methimazole needs to be taken every 8 hours for an extended period of time. Levels of triiodothyronine (T3) and thyroxine (T4) will be monitored and dosages adjusted as levels fall.

36. Which medications are used in SIADH to promote water excretion without causing sodium loss? (Select all that apply.) a. Tolvaptan (Samsca) b. Demeclocycline (Declomycin) c. Furosemide (Lasix) d. Conivaptan (Vaprisol) e. Spironolactone (Aldactone)

a, d

7. A nurse assesses a client who is prescribed levothyroxine (Synthroid) for hypothyroidism. Which assessment finding should alert the nurse that the medication therapy is effective? a. Thirst is recognized and fluid intake is appropriate. b. Weight has been the same for 3 weeks. c. Total white blood cell count is 6000 cells/mm3. d. Heart rate is 70 beats/min and regular.

D. Hypothyroidism decreases body functioning and can result in effects such as bradycardia, confusion, and constipation. If a client's heart rate is bradycardic while on thyroid hormone replacement, this is an indicator that the replacement may not be adequate. Conversely, a heart rate above 100 beats/min may indicate that the client is receiving too much of the thyroid hormone. Thirst, fluid intake, weight, and white blood cell count do not represent a therapeutic response to this medication.

Ch.62 p. 1276, Patient-Centered Care; Quality Improvement; Safety The patient is a 32-year-old woman admitted to your unit after surgery for fractures of the left arm and leg resulting from a car crash. She is awake and able to verify her medical history of rheumatoid arthritis and her usual daily medications. These are 10 mg of prednisone, naproxen 800 mg twice daily, oral contraceptives, calcium 600 mg, and one multiple vitamin tablet. All of these are prescribed for her to receive during her hospitalization. She is concerned about pain management and how long the recovery will be for the fractures. She is friendly, somewhat anxious, asks many questions, and wants to do "her part" to ensure good recovery. Over the next 4 days, she has become quieter, mumbles that her head and stomach hurt, and now does not recognize the assistant who has been providing her daily care. When she receives her medications, she has difficulty picking them up. The nursing assistant remarks that taking her pulse is difficult because it is so slow and irregular. When you assess her, she is so weak that she is unable to lift her arm for a blood pressure check. Her blood pressure is 92/50, which is down from the 128/84 reading on admission. You also verify that her heart beat is slow and irregular. 3. How could this problem been avoided?

Daily assessment of her salivary or serum cortisol levels could have indicated a need for a higher dose. Also, because she was receiving prednisone, daily blood glucose levels should have been performed. Examining these parameters would provide data to determine the adequacy of her therapy, as well as its potential side effects.

Ch.64 p. 1329, Patient-Centered Care During a clinic visit, you are reviewing the records of a 39-year-old patient who was diagnosed 5 years ago with type 2 diabetes. You discover that, although he has always been extremely near-sighted, he has not seen an ophthalmologist for 4 years. He has gained 12 lbs since his last visit a year ago. His laboratory values show a fasting blood glucose level of 96 mg/dL, an A1C of 8.2%, a total cholesterol of 322 mg/dL, and an LDL of 190 mg/dL. When you ask him about ophthalmology follow-up and point out his laboratory values, he replies that because he is taking prescribed antidiabetic medication, he believes that he won't have all the diabetes complications that his father had. He further tells you that he did have his eyes checked by an optometrist to make sure his prescription was accurate but that because he was younger than 40 years old, he does not need intraocular pressure measurements. 3. Is he correct in thinking that an ophthalmologist visit is not necessary at this time? Explain your response.

Even if he did not have a vision problem, his risk for ophthalmic complications leading to blindness is high. Just having diabetes is a reason to been seen by an ophthalmologist rather than an optometrist (who is not a medical doctor). Depending on the control of his disease (which right now is not very controlled), coupled with long-standing vision problems, he should be seen yearly by an ophthalmologist. The ophthalmologist can determine whether a less frequent evaluation cycle would be appropriate.

Ch.64 p. 1329, Patient-Centered Care During a clinic visit, you are reviewing the records of a 39-year-old patient who was diagnosed 5 years ago with type 2 diabetes. You discover that, although he has always been extremely near-sighted, he has not seen an ophthalmologist for 4 years. He has gained 12 lbs since his last visit a year ago. His laboratory values show a fasting blood glucose level of 96 mg/dL, an A1C of 8.2%, a total cholesterol of 322 mg/dL, and an LDL of 190 mg/dL. When you ask him about ophthalmology follow-up and point out his laboratory values, he replies that because he is taking prescribed antidiabetic medication, he believes that he won't have all the diabetes complications that his father had. He further tells you that he did have his eyes checked by an optometrist to make sure his prescription was accurate but that because he was younger than 40 years old, he does not need intraocular pressure measurements. 4. Is he correct in believing that taking antidiabetic medication will prevent complications of diabetes? Explain your response.

He is not correct in his thinking. Diabetes is a complex disorder and can only be controlled with a combination of antidiabetic medications and life-style changes that include nutrition therapy, maintenance of a healthy weight, blood pressure control, blood lipid control, and physical activity. The drugs are only part of the management plan. The fact that he was diagnosed at an earlier age and is taking medications is helpful, but without proper management, the complications of diabetes will not even be delayed let alone prevented.

Ch.64 p. 1329, Patient-Centered Care During a clinic visit, you are reviewing the records of a 39-year-old patient who was diagnosed 5 years ago with type 2 diabetes. You discover that, although he has always been extremely near-sighted, he has not seen an ophthalmologist for 4 years. He has gained 12 lbs since his last visit a year ago. His laboratory values show a fasting blood glucose level of 96 mg/dL, an A1C of 8.2%, a total cholesterol of 322 mg/dL, and an LDL of 190 mg/dL. When you ask him about ophthalmology follow-up and point out his laboratory values, he replies that because he is taking prescribed antidiabetic medication, he believes that he won't have all the diabetes complications that his father had. He further tells you that he did have his eyes checked by an optometrist to make sure his prescription was accurate but that because he was younger than 40 years old, he does not need intraocular pressure measurements. 1. How should you interpret his laboratory values in terms of his personal glucose regulation?

His fasting blood glucose level is acceptable and indicates that he has controlled his diabetes during the past 24 hours. However, his hemoglobin A1C is high, indicating that his overall control for the past several months is poor. It is possible that the current medication regimen is not sufficient to manage his disease. The fact that his weight is increasing rather than decreasing and that his blood lipid levels are quite high hint that his nutrition therapy is probably not being followed.

Ch.62 p. 1276, Patient-Centered Care; Quality Improvement; Safety The patient is a 32-year-old woman admitted to your unit after surgery for fractures of the left arm and leg resulting from a car crash. She is awake and able to verify her medical history of rheumatoid arthritis and her usual daily medications. These are 10 mg of prednisone, naproxen 800 mg twice daily, oral contraceptives, calcium 600 mg, and one multiple vitamin tablet. All of these are prescribed for her to receive during her hospitalization. She is concerned about pain management and how long the recovery will be for the fractures. She is friendly, somewhat anxious, asks many questions, and wants to do "her part" to ensure good recovery. Over the next 4 days, she has become quieter, mumbles that her head and stomach hurt, and now does not recognize the assistant who has been providing her daily care. When she receives her medications, she has difficulty picking them up. The nursing assistant remarks that taking her pulse is difficult because it is so slow and irregular. When you assess her, she is so weak that she is unable to lift her arm for a blood pressure check. Her blood pressure is 92/50, which is down from the 128/84 reading on admission. You also verify that her heart beat is slow and irregular. 1. What other assessment data should you obtain immediately and why?

Listen to her apical pulse to assess the true heart rate. With some dysrhythmias, especially if she is having premature contraction, the radial pulse can be very different from the apical pulse. Assess her oxygen saturation to determine whether cardiac function is adequate for the moment or whether the rapid response team is needed now. (Cardiac arrest is possible because of hyperkalemia.) Perform a finger stick blood glucose analysis immediately to determine whether she is hypoglycemic.

16. A patient is recovering from a transphenoidal hypophysectomy. What postoperative nursing interventions apply to this patient? (Select all that apply.) a. Encouraging the patient to perform deep-breathing exercises b. Vigorous coughing and deep-breathing exercises c. Instructing on the use of a soft-bristled toothbrush for brushing the teeth d. Struct monitoring of fluid balance e. Hourly neurologic checks for first 24 hours f. Instructing the patient to alert the nurse regarding postnasal drip

a, d, e, f

Ch.63 p. 1295, Patient-Centered Care; Safety The patient, a 45-year-old former school teacher, is residing in a skilled nursing facility to recover from a tibia-fibula fracture that is being managed with an external fixation system. On admission 2 weeks ago, she told you that she felt she was "getting old too fast." She explained that she had gained 54 pounds in the previous 6 months, had no energy, was often constipated, and was always cold. She teared up and said that her ability to concentrate was so bad that not only could she no longer help her high school children with their homework, but that she didn't recognize the step hazard that caused her to fall and break her ankle. Today the nursing assistant assigned to her care reports that the patient's pulse is only 42 beats per minute and that her temperature was 96° F even with two blankets. When you enter her room, she is sleeping with an untouched breakfast tray on her table. 4. What manifestations of hypothyroidism are in her history and present during this assessment?

Many of the problems that she has listed over the past 6 months are manifestations of hypothyroidism that is slowly getting worse. These include her weight gain, having no energy, being constipated frequently, always feeling cold, and having difficulty with concentration. It is also possible that her ankle fractured more easily because of hypothyroidism-induced bone thinning.

Ch.64 p. 1335, Prioritization, Delegation, and Supervision The patient, a 21-year-old college student, was brought to the emergency department (ED) by his roommate. He reports abdominal pain, polyuria for the past 2 days, vomiting several times prior to arrival, and extreme thirst. He appears flushed, and his lips and mucous membranes are dry and cracked. His skin turgor is poor. He demonstrates deep rapid respirations; there is a fruit odor to his breath. He has type 1 diabetes and "may have skipped a few doses of insulin because of cramming for final exams." He is alert and talking but is having trouble focusing on your questions. Vital signs: Blood pressure 110/60 Pulse 110/min Respirations 32/min Temperature 100.8° F Fingerstick glucose 485 mg/dL Oxygen saturation 99% 6. What intravenous (IV) solution do you anticipate the ED physician will order for initial fluid replacement?

Normal saline (0.9% sodium chloride) is the first fluid used to correct dehydration in most adults with diabetic ketoacidosis.

Ch.63 p. 1295, Patient-Centered Care; Safety The patient, a 45-year-old former school teacher, is residing in a skilled nursing facility to recover from a tibia-fibula fracture that is being managed with an external fixation system. On admission 2 weeks ago, she told you that she felt she was "getting old too fast." She explained that she had gained 54 pounds in the previous 6 months, had no energy, was often constipated, and was always cold. She teared up and said that her ability to concentrate was so bad that not only could she no longer help her high school children with their homework, but that she didn't recognize the step hazard that caused her to fall and break her ankle. Today the nursing assistant assigned to her care reports that the patient's pulse is only 42 beats per minute and that her temperature was 96° F even with two blankets. When you enter her room, she is sleeping with an untouched breakfast tray on her table. 3. What indications do you have that the changes in her health status are not related to complications of her fractured ankle?

Some of her health status changes could be related to her fracture, specifically sepsis and pulmonary embolism. However, because she has had some of these changes gradually over several weeks, they are more likely to be related to reduced thyroid function. By assessing her cardiac and respiratory status, you are also assessing for manifestations of sepsis. Usually, respirations are elevated for either pulmonary embolism or for sepsis. Heart rate is usually fast and thready with these two problems.

Ch.62 p. 1276, Patient-Centered Care; Quality Improvement; Safety The patient is a 32-year-old woman admitted to your unit after surgery for fractures of the left arm and leg resulting from a car crash. She is awake and able to verify her medical history of rheumatoid arthritis and her usual daily medications. These are 10 mg of prednisone, naproxen 800 mg twice daily, oral contraceptives, calcium 600 mg, and one multiple vitamin tablet. All of these are prescribed for her to receive during her hospitalization. She is concerned about pain management and how long the recovery will be for the fractures. She is friendly, somewhat anxious, asks many questions, and wants to do "her part" to ensure good recovery. Over the next 4 days, she has become quieter, mumbles that her head and stomach hurt, and now does not recognize the assistant who has been providing her daily care. When she receives her medications, she has difficulty picking them up. The nursing assistant remarks that taking her pulse is difficult because it is so slow and irregular. When you assess her, she is so weak that she is unable to lift her arm for a blood pressure check. Her blood pressure is 92/50, which is down from the 128/84 reading on admission. You also verify that her heart beat is slow and irregular. 2. What is the most likely cause of the changes in this patient's physical and mental status?

The most likely cause is acute adrenal insufficiency as a result of increased cortisol needs related to the stress of surgery and injury. Because she has been on prednisone long term, she has some degree of adrenal suppression and cannot increase the extra cortisol needed during the additional stress. Although she is receiving 10 mg of prednisone daily, it is not enough for her current needs.

Ch.64 p. 1332, Safety; Quality Improvement; Teamwork and Collaboration The patient is a 60 year-old-woman who is 1 day postoperative after a total knee replacement. She has type 2 diabetes and just recently was switched from oral antidiabetic drugs to an insulin regimen. She let her nurse know that her on-demand lunch has been ordered. The nurse tests her blood and gives her the prescribed short-acting insulin dose. An hour later, the physical therapist finds her pale, confused, and clammy. Her lunch tray is on her table and appears totally untouched. 4. What could be done on this nursing care unit to prevent such an incident from happening again?

The patient should receive more education about the relationship between insulin and eating. The unit needs to establish guidelines or policies about premeal insulin administration. Perhaps it should not be administered until the tray is actually in the patient's possession and the patient is ready to eat it. Also, whenever short-acting insulin is given, the nurse giving it should evaluate the patient within 20 minutes.

Ch.64 p. 1329, Patient-Centered Care During a clinic visit, you are reviewing the records of a 39-year-old patient who was diagnosed 5 years ago with type 2 diabetes. You discover that, although he has always been extremely near-sighted, he has not seen an ophthalmologist for 4 years. He has gained 12 lbs since his last visit a year ago. His laboratory values show a fasting blood glucose level of 96 mg/dL, an A1C of 8.2%, a total cholesterol of 322 mg/dL, and an LDL of 190 mg/dL. When you ask him about ophthalmology follow-up and point out his laboratory values, he replies that because he is taking prescribed antidiabetic medication, he believes that he won't have all the diabetes complications that his father had. He further tells you that he did have his eyes checked by an optometrist to make sure his prescription was accurate but that because he was younger than 40 years old, he does not need intraocular pressure measurements. 5. How do you propose to assist this patient in managing his diabetes?

The patient's comments and the laboratory data indicate that he does not understand the disease, its consequences, management techniques, and his role in the management plan. His issues are going to require more than your intervention, although you can get this started. Patient-centered evaluation and management with the entire diabetes management team is needed as soon as possible. He will need further testing to assess for early-stage complications and possible changes to his medication regimen. You should start by asking what he knows about the disease and correct any misunderstandings. Bringing in a diabetes educator, the health care provider and registered dietitian is really needed now. If classes are available, he should be strongly encouraged to start them. If he has a partner, try to include her or him in this process. Essentially, this patient requires that the team treat him as if he had just been newly diagnosed with diabetes.

Ch.62 p. 1276, Patient-Centered Care; Quality Improvement; Safety The patient is a 32-year-old woman admitted to your unit after surgery for fractures of the left arm and leg resulting from a car crash. She is awake and able to verify her medical history of rheumatoid arthritis and her usual daily medications. These are 10 mg of prednisone, naproxen 800 mg twice daily, oral contraceptives, calcium 600 mg, and one multiple vitamin tablet. All of these are prescribed for her to receive during her hospitalization. She is concerned about pain management and how long the recovery will be for the fractures. She is friendly, somewhat anxious, asks many questions, and wants to do "her part" to ensure good recovery. Over the next 4 days, she has become quieter, mumbles that her head and stomach hurt, and now does not recognize the assistant who has been providing her daily care. When she receives her medications, she has difficulty picking them up. The nursing assistant remarks that taking her pulse is difficult because it is so slow and irregular. When you assess her, she is so weak that she is unable to lift her arm for a blood pressure check. Her blood pressure is 92/50, which is down from the 128/84 reading on admission. You also verify that her heart beat is slow and irregular. 4. What specifically would be the nurse's role in preventing this problem?

This type of adrenal insufficiency develops over a period of days. Monitoring trends for level of consciousness, blood pressure, and heart rate and rhythm should be something all nurses do on every patient. It is very likely that changes were present earlier and not recognized.

5. Which hormones are secreted by the thyroid gland? (Select all that apply.) a. Calcitonin b. Somatostatin c. Glucagon d. Thyroxine (T4) e. Aldosterone f. Triiodothyronine (T3)

a, d, f Calcitonin, Thyroxine (T4), Triiodothyronine (T3)

Ch.64 p. 1335, Prioritization, Delegation, and Supervision The patient, a 21-year-old college student, was brought to the emergency department (ED) by his roommate. He reports abdominal pain, polyuria for the past 2 days, vomiting several times prior to arrival, and extreme thirst. He appears flushed, and his lips and mucous membranes are dry and cracked. His skin turgor is poor. He demonstrates deep rapid respirations; there is a fruit odor to his breath. He has type 1 diabetes and "may have skipped a few doses of insulin because of cramming for final exams." He is alert and talking but is having trouble focusing on your questions. Vital signs: Blood pressure 110/60 Pulse 110/min Respirations 32/min Temperature 100.8° F Fingerstick glucose 485 mg/dL Oxygen saturation 99% 4. Your work plan includes checking hourly vital signs, assessing blood glucose levels, updating the roommate about the patient's condition, and measuring the patient's emesis. Which task(s) is (are) appropriate to assign to the new nursing assistant? Provide a rationale for your choices.

Vital signs and measuring and recording output are within the scope of duties for the nursing assistant. Releasing information should not be done by the nursing assistant because of confidentiality issues. The RN should decide how to convey information to friends and family who are waiting keeping HIPAA standards in mind. Checking blood glucose is usually accomplished with a fingerstick, which is usually within the scope of practice for the experienced nursing assistant but not for one who is new.

Ch.63 p. 1295, Patient-Centered Care; Safety The patient, a 45-year-old former school teacher, is residing in a skilled nursing facility to recover from a tibia-fibula fracture that is being managed with an external fixation system. On admission 2 weeks ago, she told you that she felt she was "getting old too fast." She explained that she had gained 54 pounds in the previous 6 months, had no energy, was often constipated, and was always cold. She teared up and said that her ability to concentrate was so bad that not only could she no longer help her high school children with their homework, but that she didn't recognize the step hazard that caused her to fall and break her ankle. Today the nursing assistant assigned to her care reports that the patient's pulse is only 42 beats per minute and that her temperature was 96° F even with two blankets. When you enter her room, she is sleeping with an untouched breakfast tray on her table. 2. Should oxygen be applied? Why or why not?

Yes, oxygen should be applied. Her heart rate is so low that she is not able to maintain good oxygenation of vital organs. Applying additional oxygen can help oxygenate those tissues. Death from hypoxia is a strong possibility.

18. The patient who is prescribed methimazole (Tapazole) 4 mg orally every 8 hours tells the nurse that his heart rate is slow (60/minute), he has gained 7 pounds, and he wears a sweater even on warm days. What does the nurse suspect? a. Indications of hypothyroidism will require a lower dosage. b. Indications of hypothyroidism will require a higher dosage. c. Indications of hyperthyroidism will require a lower dosage. d. Indications of hyperthyroidism will require a higher dosage.

a

21. A patient with a hypophysectomy can postoperatively experience transient DI. Which manifestation alerts the nurse to this problem? a. Output much greater than intake b. Change in mental status indicating confusion c. Laboratory results indicating hyponatremia d. Nonpitting edema

a

25. After a visit to the health care provider's office, a patient is diagnosed with general thyroid enlargement and elevated thyroid hormone level. Which condition do these findings indicate? a. Hyperthyroidism and goiter b. Hypothyroidism and goiter c. Nodules on the parathyroid gland d. Thyroid or parathyroid cancer

a

27. The nurse is caring for a patient with DI. What is the priority goal of collaborative care? a. Correct the water metabolism problem. b. Control blood sugar and blood pH. c. Measure urine output, specific gravity, and osmolality hourly. d. Monitor closely for respiration distress.

a

28. Which medication is used to treat DI? a. Desmopressin acetate (DDAVP) b. Lithium (Eskalith) c. Vasopressin (Pitressin) d. Demeclocycline (Declomycin)

a

29. A patient has the following assessment findings: elevated TSH level, low T3 and T4 levels, difficulty with memory, lethargy, and muscle stiffness. These are clinical manifestations of which disorder? a. Hypothyroidism b. Hyperthyroidism c. Hypoparathyroidism d. Hyperparathyroidism

a

33. Which nursing intervention is the priority for a patient with SIADH? a. Restrict fluid intake. b. Monitor neurologic status at least every 2 hours. c. Offer ice chips frequently to ease discomfort by dry mouth . d. Monitor urine tests for decreased sodium levels and low specific gravity.

a

43. Production of which hormone causes lower levels of calcium? a. Calcitonin b. PTH c. T4 d. TSH

a

55. An ACTH stimulation test is the most definitive test for which disorder? a. Adrenal insufficiency b. Cushing's syndrome c. Pheochromocytoma d. Acromegaly

a

9. The clinical manifestations of hyperthyroidism are known as which condition? a. Thyrotoxicosis b. Euthyroid function c. Graves' disease d. Hypermetabolism

a

A patient has the following assessment findings: elevated TSH level, low T3 and T4 levels, difficulty with memory, lethargy, and muscle stiffness. These are clinical manifestations of which disorder? a. Hypothyroidism b. Hyperthyroidism c. Hypoparathyroidism d. Hyperparathyroidism

a

After a visit to the health care provider's office, a patient is diagnosed with general thyroid enlargement and elevated thyroid hormone level. Which condition do these findings indicate? a. Hyperthyroidism and goiter b. Hypothyroidism and goiter c. Nodules on the parathyroid gland d. Thyroid or parathyroid cancer

a

The clinical manifestations of hyperthyroidism are known as which condition? a. Thyrotoxicosis b. Euthyroid function c. Graves' disease d. Hypermetabolism

a

The patient who is prescribed methimazole (Tapazole) 4 mg orally every 8 hours tells the nurse that his heart rate is slow (60/minute), he has gained 7 pounds, and he wears a sweater even on warm days. What does the nurse suspect? a. Indications of hypothyroidism will require a lower dosage. b. Indications of hypothyroidism will require a higher dosage. c. Indications of hyperthyroidism will require a lower dosage. d. Indications of hyperthyroidism will require a higher dosage.

a

Ch.61 Question 9 of 16 A client with an endocrine disorder says, "I can't, you know, satisfy my wife anymore." What is the nurse's best response? a. "Can you please tell me more?" b. "Don't worry. That is normal." c. "How does she feel?" d. "Should I make an appointment with a counselor?"

a "Can you please tell me more?" Asking the client to explain his concerns in an open-ended question allows the nurse to explore his feelings more thoroughly. Telling a client that something is "normal" is dismissive; this is new to the client and is a concern for him. The focus of the nurse's response needs to be on the client, not on the wife initially. Referring the client to a counselor is not an appropriate first step; this dismisses the client's concerns and does not allow him to express his frustrations at the moment.

Question 9 of 16 A client with an endocrine disorder says, "I can't, you know, satisfy my wife anymore." What is the nurse's best response? a. "Can you please tell me more?" b. "Don't worry. That is normal." c. "How does she feel?" d. "Should I make an appointment with a counselor?"

a "Can you please tell me more?" Asking the client to explain his concerns in an open-ended question allows the nurse to explore his feelings more thoroughly. Telling a client that something is "normal" is dismissive; this is new to the client and is a concern for him. The focus of the nurse's response needs to be on the client, not on the wife initially. Referring the client to a counselor is not an appropriate first step; this dismisses the client's concerns and does not allow him to express his frustrations at the moment.

Ch.61 Question 7 of 16 The nurse is instructing a client who will undergo a suppression test. Which statement by the client indicates that teaching was effective? a. "I am being tested to see whether my hormone glands are hyperactive." b. "I am being tested to see whether my hormone glands are hypoactive." c. "I am being tested to see whether my kidneys work at all." d. "I will be given more hormones as a trigger."

a "I am being tested to see whether my hormone glands are hyperactive." Suppression tests are used when hormone levels are high or in the upper range of normal. Failure of suppression of hormone production during testing indicates hyperfunction. A stimulation test assesses whether hormone glands are hypoactive. The adrenal glands are endocrine glands that are located on the kidneys; a suppression test does not measure kidney function. Hormones are given as a trigger in a stimulation test.

Question 7 of 16 The nurse is instructing a client who will undergo a suppression test. Which statement by the client indicates that teaching was effective? a."I am being tested to see whether my hormone glands are hyperactive." b. "I am being tested to see whether my hormone glands are hypoactive." c. "I am being tested to see whether my kidneys work at all." d. "I will be given more hormones as a trigger."

a "I am being tested to see whether my hormone glands are hyperactive." Suppression tests are used when hormone levels are high or in the upper range of normal. Failure of suppression of hormone production during testing indicates hyperfunction. A stimulation test assesses whether hormone glands are hypoactive. The adrenal glands are endocrine glands that are located on the kidneys; a suppression test does not measure kidney function. Hormones are given as a trigger in a stimulation test.

Ch.64 Question 20 of 25 The nurse is providing discharge teaching to a client with diabetes about injury prevention for peripheral neuropathy. Which statement by the client indicates a need for further teaching? a. "I can break in my shoes by wearing them all day." b. "I need to monitor my feet daily for blisters or skin breaks." c. "I should never go barefoot." d. "I should quit smoking."

a "I can break in my shoes by wearing them all day." Shoes should be properly fitted and worn for a few hours a day to break them in, with frequent inspection for irritation or blistering. People with diabetes have decreased peripheral circulation, so even small injuries to the feet must be managed early. Going barefoot is contraindicated. Tobacco use further decreases peripheral circulation in a client with diabetes.

Ch.63Question 6 of 21 A client is being discharged with propylthiouracil (PTU). Which statement by the client indicates a need for further teaching by the nurse? a. "I can return to my job at the nursing home." b. "I must call if my urine is dark." c. "I must faithfully take the drug every 8 hours." d. "I need to report weight gain."

a "I can return to my job at the nursing home." The client should avoid large crowds and people who are ill because PTU reduces blood cell counts and the immune response, which increases the risk for infection. The client does not, however, need to remain completely at home. Dark urine may indicate liver toxicity or failure, and the client must notify the provider immediately. Taking PTU regularly at the same time each day provides better drug levels and ensures better drug action. The client must notify the provider of weight gain because this may indicate hypothyroidism; a lower drug dose may be required.

Ch.64 Question 21 of 25 The nurse is teaching a client with type 2 diabetes about the importance of weight control. Which comment by the client indicates a need for further teaching? a. "I should begin exercising for at least an hour a day." b. "I should monitor my diet." c. "If I lose weight, I may not need to use the insulin anymore." d. "Weight loss can be a sign of diabetic ketoacidosis."

a "I should begin exercising for at least an hour a day." For long-term maintenance of major weight loss, large amounts of exercise (7 hr/wk) or moderate or vigorous aerobic physical activity may be helpful, but the client must start slowly. Monitoring the diet is key to type 2 diabetes management. Weight loss can minimize the need for insulin and can also be a sign of diabetic ketoacidosis.

Ch.62 Question 9 of 23 A client with syndrome of inappropriate antidiuretic hormone is admitted with a serum sodium level of 105 mEq/L. Which request by the health care provider does the nurse address first? a. Administer infusion of 150 mL of 3% NaCl over 3 hours. b. Draw blood for hemoglobin and hematocrit. c. Insert retention catheter and monitor urine output. d. Weigh the client on admission and daily thereafter.

a Administer infusion of 150 mL of 3% NaCl over 3 hours. The client with a sodium level of 105 mEq/L is at high risk for seizures and coma. The priority intervention is to increase the sodium level to a more normal range. Ideally, 3% NaCl should be infused through a central line or with a small needle through a large vein to prevent irritation. Monitoring laboratory values for fluid balance and monitoring urine output are important, but are not the top priority. Monitoring client weight will help in the assessment of fluid balance; however, this is also not the top priority.

The nurse assesses a patient in the emergency department (ED) and finds the following: constipation, fatigue with increased sleeping time, impaired memory, facial puffiness, and weight gain. Which deficiency does the nurse recognize? a. Hyperthyroidism b. Hypothyroidism c. Hyperparathyroidism d. Hypoparathyroidism

b

Ch.62 Question 8 of 23 A client presents to the emergency department with a history of adrenal insufficiency. The following laboratory values are obtained: Na+ 130 mEq/L, K+ 5.6 mEq/L, and glucose 72 mg/dL. Which is the first request that the nurse anticipates? a. Administer insulin and dextrose in normal saline to shift potassium into cells. b. Give spironolactone (Aldactone) 100 mg orally. c. Initiate histamine2 (H2) blocker therapy with ranitidine for ulcer prophylaxis. d. Obtain arterial blood gases to assess for peaked T waves.

a Administer insulin and dextrose in normal saline to shift potassium into cells. This client is hyperkalemic. The nurse should anticipate a request to administer 20 to 50 units of insulin with 20 to 50 mg of dextrose in normal saline as an IV infusion to shift potassium into the cells. Spironolactone is a potassium-sparing diuretic that helps the body keep potassium, which the client does not need. Although H2 blocker therapy would be appropriate for this client, it is not the first priority. Arterial blood gases are not used to assess for peaked T waves associated with hyperkalemia; an electrocardiogram needs to be obtained instead.

Ch.61 Question 8 of 16 Which gland releases catecholamines? a. Adrenal b. Pancreas c. Parathyroid d. Thyroid

a Adrenal The adrenal medulla releases catecholamines in response to stimulation of the sympathetic nervous system. The principal hormones of the pancreas are insulin, glucagon, and somatostatin. Parathyroid hormone is the principal hormone of the parathyroid gland. Triiodothyronine (T3), thyroxine (T4), and calcitonin are the principal hormones of the thyroid.

Question 8 of 16 Which gland releases catecholamines? a. Adrenal b. Pancreas c. Parathyroid d. Thyroid

a Adrenal The adrenal medulla releases catecholamines in response to stimulation of the sympathetic nervous system. The principal hormones of the pancreas are insulin, glucagon, and somatostatin. Parathyroid hormone is the principal hormone of the parathyroid gland. Triiodothyronine (T3), thyroxine (T4), and calcitonin are the principal hormones of the thyroid.

Ch.64 Question 19 of 25 Which is the best referral that the nurse can suggest to a client who has been newly diagnosed with diabetes? a. American Diabetes Association b. Centers for Disease Control and Prevention c. Health care provider office d. Pharmaceutical representative

a American Diabetes Association The American Diabetes Association can provide national and regional support and resources to clients with diabetes and their families. The Centers for Disease Control and Prevention does not focus on diabetes. The client's health care provider's office is not the best resource for diabetes information and support. A pharmaceutical representative is not an appropriate resource for diabetes information and support.

Ch.61 Question 10 of 16 The nurse should encourage fluids every 2 hours for older adult clients because of a decrease in which factor? a. Antidiuretic hormone (ADH) production b. General metabolism c. Glucose tolerance d. Ovarian production of estrogen

a Antidiuretic hormone (ADH) production A decrease in ADH production causes urine to be more dilute, so urine might not concentrate when fluid intake is low. The older adult is at greater risk for dehydration as a result of urine loss. A decrease in general metabolism causes decreased tolerance to cold, decreased appetite, and decreased heart rate and blood pressure; it is not related to fluid intake or hydration. A decrease in glucose tolerance does not affect fluid intake or hydration. A decrease in estrogen production causes a decrease in bone density and is not related to fluid intake and hydration.

Question 10 of 16 The nurse should encourage fluids every 2 hours for older adult clients because of a decrease in which factor? a. Antidiuretic hormone (ADH) production b. General metabolism c. Glucose tolerance d. Ovarian production of estrogen

a Antidiuretic hormone (ADH) production A decrease in ADH production causes urine to be more dilute, so urine might not concentrate when fluid intake is low. The older adult is at greater risk for dehydration as a result of urine loss. A decrease in general metabolism causes decreased tolerance to cold, decreased appetite, and decreased heart rate and blood pressure; it is not related to fluid intake or hydration. A decrease in glucose tolerance does not affect fluid intake or hydration. A decrease in estrogen production causes a decrease in bone density and is not related to fluid intake and hydration.

Ch.62 Question 19 of 23 A client with iatrogenic Cushing's syndrome is a resident in a long-term care facility. Which nursing action included in the client's care would be best to delegate to unlicensed assistive personnel (UAP)? a. Assist with personal hygiene and skin care. b. Develop a plan of care to minimize risk for infection. c. Instruct the client on the reasons to avoid overeating. d. Monitor for signs and symptoms of fluid retention.

a Assist with personal hygiene and skin care. Assisting a client with bathing and skin care is included in UAP scope of practice. It is not within their scope of practice to develop a plan of care, although they will play a very important role in following the plan of care. Client teaching requires a broad education and should not be delegated to UAP. Monitoring for signs and symptoms of fluid retention is part of client assessment, which requires a higher level of education and clinical judgment.

Ch.61 Question 6 of 23 A client with pheochromocytoma is admitted for surgery. What does the nurse do for the admitting assessment? a. Avoids palpating the abdomen b. Monitors for pulmonary edema with a chest x-ray c. Obtains a 24-hour urine specimen on admission d. Places the client in a room with a roommate for distraction

a Avoids palpating the abdomen The abdomen must not be palpated in a client with pheochromocytoma because this action could cause a sudden release of catecholamines and severe hypertension. The tumor on the adrenal gland causes sympathetic hyperactivity, increasing blood pressure and heart rate, not pulmonary edema. A 24-hour urine collection will already have been completed to determine the diagnosis of pheochromocytoma. A client diagnosed with a pheochromocytoma may feel anxious as part of the disease process; providing a roommate for distraction will not reduce the client's anxiety.

Ch.64 Question 8 of 25 A client with type 1 diabetes arrives in the emergency department breathing deeply and stating, "I can't catch my breath." The client's vital signs are: T 98.4° F (36.9° C), P 112 beats/min, R 38 breaths/min, BP 91/54 mm Hg, and O2 saturation 99% on room air. Which action does the nurse take first? a. Check the blood glucose. b. Administer oxygen. c. Offer reassurance. d. Attach a cardiac monitor.

a Check the blood glucose. The client's clinical presentation is consistent with diabetic ketoacidosis, so the nurse should initially check the client's glucose level. Based on the oxygen saturation, oxygen administration is not necessary. The nurse provides support, but it is early in the course of assessment and intervention to offer reassurance without more information. Cardiac monitoring may be implemented, but the first action should be to obtain the glucose level.

Ch.61 Question 14 of 16 The charge nurse on the medical-surgical unit is making client assignments for the shift. Which client is the most appropriate to assign to an LPN/LVN? a. Client with Cushing's syndrome who requires orthostatic vital signs assessments b. Client with diabetes mellitus who was admitted with a blood glucose of 45 mg/dL c. Client with exophthalmos who has many questions about endocrine function d. Client with possible pituitary adenoma who has just arrived on the nursing unit

a Client with Cushing's syndrome who requires orthostatic vital signs assessments An LPN/LVN will be familiar with Cushing's syndrome and the method for assessment of orthostatic vital signs. The client with a blood glucose of 45 mg/dL, the client with questions about endocrine function, and the client with a possible pituitary adenoma all have complex needs that require the experience and scope of practice of an RN.

Ch.63 Question 4 of 21 The nurse manager for the medical-surgical unit is making staff assignments. Which client will be most appropriate to assign to a newly graduated RN who has completed a 6-week unit orientation? a. Client with chronic hypothyroidism and dementia who takes levothyroxine (Synthroid) daily b. Client with follicular thyroid cancer who has vocal hoarseness and difficulty swallowing c. Client with Graves' disease who is experiencing increasing anxiety and diaphoresis d. Client with hyperparathyroidism who has just arrived on the unit after a parathyroidectomy

a Client with Hashimoto's thyroiditis and a large goiter The client with chronic hypothyroidism and dementia is the most stable of the clients described and would be most appropriate to assign to an inexperienced RN. A client with vocal hoarseness and difficulty swallowing is at higher risk for complications and requires close observation by a more experienced nurse. Increasing anxiety and diaphoresis in a client with Graves' disease can be an indication of impending thyroid storm, which is an emergency; this is not a situation to be managed by a newly graduated RN. A client who has just arrived on the unit after a parathyroidectomy requires close observation for bleeding and airway compromise and requires assessment by an experienced nurse.

Ch.62 Question 23 of 23 After receiving change-of-shift report about these four clients, which client does the nurse attend to first? a. Client with acute adrenal insufficiency who has a blood glucose of 36 mg/dL b. Client with diabetes insipidus who has a dose of desmopressin (DDAVP) due c. Client with hyperaldosteronism who has a serum potassium of 3.4 mEq/L d. Client with pituitary adenoma who is reporting a severe headache

a Client with acute adrenal insufficiency who has a blood glucose of 36 mg/dL A glucose level of 36 mg/dL is considered an emergency; this client must be assessed and treated immediately. Although it is important to maintain medications on schedule, the client requiring a dose of desmopressin is not the first client who needs to be seen. A serum potassium of 3.4 mEq/L in the client with hyperaldosteronism may be considered normal (or slightly hypokalemic), based on specific hospital levels. The client reporting a severe headache needs to be evaluated as soon as possible after the client with acute adrenal insufficiency. As an initial measure, the RN could delegate obtaining vital signs to unlicensed assistive personnel.

Ch.64 Question 18 of 25 The nurse has just received change-of-shift report on the endocrine unit. Which client does the nurse see first? a. Client with type 1 diabetes whose insulin pump is beeping "occlusion" b. Newly diagnosed client with type 1 diabetes who is reporting thirst c. Client with type 2 diabetes who has a blood glucose of 150 mg/dL d. Client with type 2 diabetes with a blood pressure of 150/90 mm Hg

a Client with type 1 diabetes whose insulin pump is beeping "occlusion" Because glucose levels will increase quickly in clients who use continuous insulin pumps, the nurse should assess this client and the insulin pump first to avoid diabetic ketoacidosis. Thirst is a symptom of hyperglycemia and, although important, is not a priority; the nurse could delegate a fingerstick blood glucose to unlicensed assistive personnel while assessing the client whose insulin pump is beeping. Although a blood glucose reading of 150 mg/dL is mildly elevated, this is not an emergency. Mild hypertension is also not an emergency.

Ch.62 Question 15 of 23 The nurse is teaching a client about how to monitor therapy effectiveness for syndrome of inappropriate antidiuretic hormone. What does the nurse tell the client to look for? a. Daily weight gain of less than 2 pounds b. Dry mucous membranes c. Increasing heart rate d. Muscle spasms

a Daily weight gain of less than 2 pounds The client must monitor daily weights because this assesses the degree of fluid restriction needed. A weight gain of 2 pounds or more daily or a gradual increase over several days is cause for concern. Dry mucous membranes are a sign of dehydration and an indication that therapy is not effective. An increased heart rate indicates increased fluid retention or dehydration and hypovolemia, and either condition is an indication that therapy is not effective. Muscle spasms are associated with hyponatremia and are an indication of a change in the client's neurologic status. Untreated hyponatremia can lead to seizures and coma.

34. In the older adult female, which physiologic changes occur as a result of decreased function of the ovaries? a. Decreased bone density, decreased production of estrogen. b. Decreased sensitivity of peripheral tissues to the effects of insulin c. Decreased urine-concentrating ability of the kidneys d. Decreased metabolic rate

a Decreased bone density, decreased production of estrogen.

Ch.62 Question 17 of 23 A client has been admitted to the medical intensive care unit with a diagnosis of diabetes insipidus (DI) secondary to lithium overdose. Which medication is used to treat the DI? a. Desmopressin (DDAVP) b. Dopamine hydrochloride (Intropin) c. Prednisone d. Tolvaptan (Samsca)

a Desmopressin (DDAVP) Desmopressin is the drug of choice for treatment of severe DI. It may be administered orally, nasally, or by intramuscular or intravenous routes. Dopamine hydrochloride is a naturally occurring catecholamine and inotropic vasopressor; it would not be used to treat DI. Prednisone would not be used to treat DI. Tolvaptan is a selective competitive arginine vasopressin receptor 2 antagonist and is not used with DI.

5. A patient with a PRL-secreting tumor is likely to be treated with which medications? a. Dopamine agonists b. Vasopressin c. Steroids d. Growth hormone (GH)

a Dopamine agonists

A patient who has been diagnosed with Graves' disease is to receive RAI in the oral form of 131I as a treatment. What instructions does the nurse include in the teaching plan about preventing radiation exposure to others? (Select all that apply.) a. Do not share a toilet with others for 2 weeks after treatment. b. Flush the toilet three times after each use. c. Wash clothing separately from others in the household. d. Limit contact with pregnant women, infants, and children. e. Do not use a laxative within 2 weeks of having the treatment.

a, b, c, d

Ch.62 Question 1 of 23 A client with diabetes insipidus (DI) has dry lips and mucous membranes and poor skin turgor. Which intervention does the nurse provide first? a. Force fluids b. Offer lip balm c. Perform a 24-hour urine test d. Withhold desmopressin acetate (DDAVP)

a Force fluids Dry lips and mucous membranes and poor skin turgor are indications of dehydration, which can occur with DI. This is a serious condition that must be treated rapidly. Encouraging fluids is the initial step, provided the client is able to tolerate oral intake. Lip balm may make the client more comfortable, but does not address the problem of dehydration. A 24-hour urine test will identify loss of electrolytes and adrenal androgen metabolites, but will not correct the dehydration that this client is experiencing. Desmopressin acetate is a synthetic form of antidiuretic hormone that is given to reduce urine production; it improves DI and should not be withheld.

35. An older adult reports a lack of energy and not being able to do the usual daily activities without several naps during the day. Which problem may these symptoms indicate that is often seen in the older adult? a. Hypothyroidism b. Hyperparathyroidism c. Overproduction of cortisol d. Underproduction of glucagon

a Hypothyroidism

Ch.61 Question 3 of 16 The nurse is teaching a client about maintaining a proper diet to prevent an endocrine disorder. Which food does the nurse suggest after the client indicates a dislike of fish? a. Iodized salt for cooking b. More red meat c. More green vegetables d. Salt substitute for cooking

a Iodized salt for cooking Dietary deficiencies in iodide-containing foods may be a cause of an endocrine disorder. For clients who do not eat saltwater fish on a regular basis, teach them to use iodized salt in food preparation. The client should eat a well-balanced diet that includes less animal fat. Eating vegetables contributes to a proper diet; however, this does not prevent an endocrine disorder. Using a salt substitute does not prevent an endocrine disorder; in addition, salt substitutes may contain high levels of potassium, which may lead to electrolyte imbalances.

Question 3 of 16 The nurse is teaching a client about maintaining a proper diet to prevent an endocrine disorder. Which food does the nurse suggest after the client indicates a dislike of fish? a. Iodized salt for cooking b. More red meat c. More green vegetables d. Salt substitute for cooking

a Iodized salt for cooking Dietary deficiencies in iodide-containing foods may be a cause of an endocrine disorder. For clients who do not eat saltwater fish on a regular basis, teach them to use iodized salt in food preparation. The client should eat a well-balanced diet that includes less animal fat. Eating vegetables contributes to a proper diet; however, this does not prevent an endocrine disorder. Using a salt substitute does not prevent an endocrine disorder; in addition, salt substitutes may contain high levels of potassium, which may lead to electrolyte imbalances.

22. Which hormone responds to a low serum calcium blood level by increasing bone resorption? a. Parathyroid hormone (PTH) b. T4 c. T3 d. Calcitonin

a Parathyroid hormone (PTH)

Ch.64 Question 17 of 25 The nurse caring for four diabetic clients has all of these activities to perform. Which is appropriate to delegate to unlicensed assistive personnel (UAP)? a. Perform hourly bedside blood glucose checks for a client with hyperglycemia. b. Verify the infusion rate on a continuous infusion insulin pump. c. Monitor a client with blood glucose of 68 mg/dL for tremors and irritability. d. Check on a client who is reporting palpitations and anxiety.

a Perform hourly bedside blood glucose checks for a client with hyperglycemia. Performing bedside glucose monitoring is an activity that may be delegated because it does not require extensive clinical judgment to perform; the nurse will follow up with the results. Intravenous therapy and medication administration are not within the scope of practice for UAP. The client with blood glucose of 68 mg/dL will need further monitoring, assessment, and intervention not within the scope of practice for UAP. The client reporting palpitations and anxiety may have hypoglycemia, requiring further intervention; this client must be assessed by licensed nursing staff.

14. The anterior pituitary gland secretes tropic hormones in response to which hormones from the hypothalamus? a. Releasing hormones b. Target tissue hormones c. Growth hormones d. Demand hormones

a Releasing hormones

22. The nurse is assessing a patient with a diagnosis of Hashimoto's disease. What are the primary manifestations of this disease? (Select all that apply.) a. Dysphagia b. Painless enlargement of the thyroid gland c. Painful enlargement of the thyroid gland d. Weight loss e. Intolerance to heat

a, b

26. A hospital patient is prescribed desmopressin acetate metered dose spray as a replacement hormone for ADH. Which is an indication for another dose? (Select all that apply.) a. Excessive urination b. Specific gravity of 1.003 c. Dark, concentrated urine d. Edema in the legs e. Decreased urination

a, b

51. Which are assessment findings of hypocalcemia? (Select all that apply.) a. Numbness and tingling around the mouth b. Muscle cramping c. Bone fractures d. Fever e. Tachycardia

a, b

The nurse is assessing a patient with a diagnosis of Hashimoto's disease. What are the primary manifestations of this disease? (Select all that apply.) a. Dysphagia b. Painless enlargement of the thyroid gland c. Painful enlargement of the thyroid gland d. Weight loss e. Intolerance to heat

a, b

15. Which statements about hyperthyroidism are accurate? (Select all that apply.) a. It is most commonly caused by Graves' disease . b. It can be caused by overuse of thyroid replacement medication. c. It occurs more often in men between the ages of 20-40. d. Weight gain is a common manifestation. e. Serum T3 and T4 results will be elevated.

a, b, c

60. The nurse should instruct a patient who is taking hydrocortisone to report which symptoms to the health care provider for possible does adjustment? (Select all that apply.) a. Rapid weight gain b. Round face c. Fluid retnetion d. Gastrointestinal irritation e. Urinary incontinence

a, b, c

17. Following a hypophysectomy, the patient requires instruction on hormone replacement for which hormones? (Select all that apply.) a. Cortisol b. Thyroid c. Gonadal d. Vasopressin e. PRL

a, b, c, d

38. Which are diagnostic methods to measure patient hormone levels? (Select all that apply.) a. Stimulation testing b. Suppression testing c. 24-hour urine testing d. Chromatographic assay e. Needle biopsy

a, b, c, d Stimulation testing, Suppression testing, 24-hour urine testing, Chromatographic assay

12. Which statements about the etiology of hypopituitarism are correct? (Select all that apply.) a. Dysfunction can result from radiation treatment to the head or brain. b. Dysfunction can result from infection or a brain tumor. c. Infarction following systemic shock can result in hypopituitarism. d. Severe malnutrition and body fat depletion can depress pituitary gland function. e. There is always an underlying cause of hypopituitarism.

a, b, c, d Dysfunction can result from radiation treatment to the head or brain. Dysfunction can result from infection or a brain tumor. Infarction following systemic shock can result in hypopituitarism. Severe malnutrition and body fat depletion can depress pituitary gland function.

20. Which assessment findings does the nurse monitor in response to catecholamines released by the adrenal medulla? (Select all that apply.) a. Increased heart rate related to vasoconstriction b. Increased blood pressure related to vasoconstriction c. Increased perspiration d. Constriction of pupils e. Increased blood glucose in response to glycogenolysis

a, b, c, e Increased heart rate related to vasoconstriction. Increased blood pressure related to vasoconstriction. Increased perspiration. Increased blood glucose in response to glycogenolysis.

24. Which statements about T3 and T4 hormones are correct? (Select all that apply.) a. The basal metabolic rate is affected. b. Hypothalamus is stimulated by cold and stress to secret thyrotropin-releasing hormone (TRH). c. These hormones need intake of protein and iodine for production. d. Circulating hormone in the blood directly affects the production of TSH. e. T3 and T4 increase oxygen use in tissue.

a, b, c, e The basal metabolic rate is affected. Hypothalamus is stimulated by cold and stress to secret thyrotropin-releasing hormone (TRH). These hormones need intake of protein and iodine for production. T3 and T4 increase oxygen use in tissue.

10. In caring for a patient with hyperpituirarism, which symptoms does the nurse expect the patient to report? a. Joint pain b. Visual disturbances c. Changes in menstruation d. Increase libido e. Headache f. Fatigue

a, b, c, e, f Joint pain, Visual disturbances, Changes in menstruation, Headache, Fatigue

49. Which disorders/conditions can cause hyperparathyroidism? (Select all that apply.) a. Chronic kidney disease b. Neck trauma c. Thyroidectomy d. Vitamin D deficiency e. Parathyroidectomy

a, b, d

21. Which statements about hypothyroidism are accurate? (Select all that apply.) a. It occurs more often in women. b. It can be caused by iodine deficiency. c. Weight loss is a common manifestation. d. It can be caused by autoimmune thyroid destruction. e. Myxedema coma is a rare but serious complication.

a, b, d, e

39. Which statements about thyroiditis are accurate? (Select all that apply.) a. It is an inflammation of the thyroid gland. b. Hashimoto's disease is the most common type. c. It always resolves with antibiotic therapy. d. There are three types: acute, subacute, and chronic. e. The patient must take thyroid hormones.

a, b, d, e

Which statements about hypothyroidism are accurate? (Select all that apply.) a. It occurs more often in women. b. It can be caused by iodine deficiency. c. Weight loss is a common manifestation. d. It can be caused by autoimmune thyroid destruction. e. Myxedema coma is a rare but serious complication.

a, b, d, e

40. Which statement about acute thyroiditis are accurate? (Select all that apply.) a. It is caused by a bacterial infection of the thyroid gland. b. It is treated with antibiotic therapy. c. It results from a viral infection of the thyroid gland. d. Subtotal thyroidectomy is a form of treatment. e. Manifestations include neck tenderness, fever, and dysphagia.

a, b, e

13. Which statements about the pituitary glands are correct? (Select all that apply.) a. The main role of the anterior pituitary is to secrete tropic hormones. b. The posterior pituitary gland stores hormones produced by the hypothalamus. c. The anterior pituitary is connected to the thalamus gland. d. The anterior pituitary releases stored hormones produced by the hypothalamus. e. The anterior pituitary gland secretes gonadotropins.

a, b, e The main role of the anterior pituitary is to secrete tropic hormones. The posterior pituitary gland stores hormones produced by the hypothalamus. The anterior pituitary gland secretes gonadotropins.

8. Which statements about hormones and the endocrine system are accurate? (Select all that apply.) a. There are specific normal blood levels of each hormone. b. Hormones exert their effects on specific target tissues. c. Each hormone can bind with multiple receptor sites. d. The endocrine system works independently to regulate homeostasis. e. More than one hormone can be stimulated before the target tissue is affected.

a, b, e There are specific normal blood levels of each hormone. Hormones exert their effects on specific target tissues. More than one hormone can be stimulated before the target tissue is affected.

Ch.63 Question 8 of 21 The nurse is preparing the room for the client returning from a thyroidectomy. Which items are important for the nurse to have available for this client? Select all that apply. a. Calcium gluconate b. Emergency tracheotomy kit c. Furosemide (Lasix) d. Hypertonic saline e. Oxygen f. Suction

a, b, e, f Calcium gluconate, Emergency tracheotomy kit, Oxygen, Suction Calcium gluconate should be available at the bedside to treat hypocalcemia and tetany that might occur if the parathyroid glands have been injured during the surgery. Equipment for an emergency tracheotomy must be kept at the bedside in the event that hemorrhage or edema should occlude the airway. Respiratory distress can result from swelling or damage to the laryngeal nerve leading to spasm, so it is important that the nurse work with respiratory therapy to have oxygen ready at the bedside for the client on admission. Because of the potential for increased secretions, it is important that a working suction device is present at the bedside for admission of the client from the operating room. Furosemide might be useful in the postoperative client to assist with urine output; however, this is not of added importance for this client. Hypertonic saline would not be of benefit to this client as the client is not hyponatremic.

53. Discharge planning for a patient with chronic hypoparathyroidism includes which instructions? (Select all that apply.) a. Prescribed medications must be taken for the patient's entire life. b. Eat foods low in vitamin D and high in phosphorus. c. Eat foods high in calcium, but low in phosphorus. d. After several weeks, medications can be discontinued. e. Kidney stones are no longer a risk to the patient.

a, c

6. A patient is prescribed bromocriptine mesylate (Parlodel). Which information does the nurse teach the patient? (Select all that apply.) a. Get up slowly from a lying position. b. Take medication on an empty stomach. c. Take daily for purposes of raising GH levels to reduce symptom of acromegaly. d. Begin therapy with a maintenance level dose. e. Report watery nasal discharge to the health care provider immediately.

a, e Get up slowly from a lying position. Report watery nasal discharge to the health care provider immediately.

17. Which statements about the adrenal glands are correct? (Select all that apply.) a. The cortex secretes androgens in men and women. b. Catecholamines are secreted from the cortex. c. Glucocorticoids are secreted by the medulla. d. The medulla secretes hormones essential for life. e. The cortex secrets aldosterone that maintains extracellular fluid volume.

a, e The cortex secretes androgens in men and women. The cortex secrets aldosterone that maintains extracellular fluid volume.

Question 14 of 16 The charge nurse on the medical-surgical unit is making client assignments for the shift. Which client is the most appropriate to assign to an LPN/LVN? a. Client with Cushing's syndrome who requires orthostatic vital signs assessments b. Client with diabetes mellitus who was admitted with a blood glucose of 45 mg/dL c. Client with exophthalmos who has many questions about endocrine function d. Client with possible pituitary adenoma who has just arrived on the nursing unit

a. Client with Cushing's syndrome who requires orthostatic vital signs assessments An LPN/LVN will be familiar with Cushing's syndrome and the method for assessment of orthostatic vital signs. The client with a blood glucose of 45 mg/dL, the client with questions about endocrine function, and the client with a possible pituitary adenoma all have complex needs that require the experience and scope of practice of an RN.

20. While caring for a postoperative patient following a transsphenoidal hypophysectomy, the nurse observes nasal drainage that is clear with yellow color at the edge. This "halo sign" is indicative of which condition? a. Worsening neurologic status of the patient b. Drainage of CSF from the patient's nose c. Onset of postoperative infection d. An expected finding following this surgery

b

23. What is the disorder that results from a deficiency of vasopressin (ADH) from the posterior pituitary gland called? a. SIADH b. DI c. Cushing's syndrome d. Addison's disease

b

3. The nurse assesses a patient in the emergency department (ED) and finds the following: constipation, fatigue with increased sleeping time, impaired memory, facial puffiness, and weight gain. Which deficiency does the nurse recognize? a. Hyperthyroidism b. Hypothyroidism c. Hyperparathyroidism d. Hypoparathyroidism

b

30. A patient has been prescribed thyroid hormone for treatment of hypothyroidism. Within what time frame does the patient expect improvement in mental awareness with this treatment? a. A few days b. 2 weeks c. 1 month d. 3 months

b

30. Which statement about the pathophysiology of SIADH is correct? a. ADH secretion is inhibited in the presence of low plasma osmolality. b. Water retention results in dilutional hyponatremia and expanded extracellular fluid (ECF) volume. c. The glumerulus is unable to increase its filtration rate to reduce the excess plasma volume. d. Renin and aldosterone are released and help decrease the loss of urinary sodium.

b

38. A patient in the emergency department is diagnosed with possible pheochromcytoma. What is the priority nursing intervention for this patient? a. Monitor the patient's intake and output and urine specific gravity. b. Monitor blood pressure for severe hypertension. c. Monitor blood pressure for severe hypotension. d. Administer medication to increase cardiac output.

b

40. Which intervention applies to a patient with pheochromocytoma? a. Assist to sit in a chair for blood pressure monitoring. b. Instruct not to smoke, drink coffee, or change positions suddenly. c. Encourage to maintain an active exercise schedule including activity such as running. d. Encourage one glass of red wine nightly to promote rest.

b

41. Which intervention is contraindicated for a patient with pheochromocytoma? a. Monitoring blood pressure b. Palpating the abdomen c. Collecting 24-hour urine specimens d. Instructing the patient to limit activity

b

46. A patient has positive Trousseau's and Chvostek's signs resulting from hypoparathyrodism. What condition does this assessment finding indicate? a. Hypercalcemia b. Hypocalcemia c. Hyperphosphatemia d. Hypophosphatemia

b

50. Which drug decreases cortisol production? a. Mitotane (Lysodren) b. Aminoglutethimide (Cytadren) c. Cyproheptadine (Periactin) d. Hydrocortisone (Cortef)

b

51. A patient is scheduled for bilateral adrenalectomy. Before surgery, steroids are to be given. Which is the reasoning behind the administration of this drug? a. To promote glycogen storage by the liver for body energy reserves b. To compensate for sudden lack of adrenal hormones following surgery c. To increase the body's inflammatory response to promote scar formation d. To enhance urinary excretion of salt and water following surgery

b

Ch.61 Question 1 of 16 The nurse is assessing a client for endocrine dysfunction. Which comment by the client indicates a need for further assessment? a. "I am worried about losing my job because of cutbacks." b. "I don't have any patience with my kids. I lose my temper faster." c. "I don't seem to have any stressors now." d. "My weight has been stable these past few years."

b "I don't have any patience with my kids. I lose my temper faster." Many endocrine problems can change a client's behavior, personality, and psychological responses; the client stating that he or she has become short-tempered warrants further assessment. Worrying about losing a job is a normal concern but does not give any indication of a need for further assessment. The nurse will need to assess the client's claim that he or she has no stressors at present because the client's response does not provide enough information to make this determination; however, the client's statement about losing patience is the priority. Weight gain or loss may or may not be an indication of an endocrine disorder.

Question 1 of 16 The nurse is assessing a client for endocrine dysfunction. Which comment by the client indicates a need for further assessment? a. "I am worried about losing my job because of cutbacks." b. "I don't have any patience with my kids. I lose my temper faster." c. "I don't seem to have any stressors now." d. "My weight has been stable these past few years."

b "I don't have any patience with my kids. I lose my temper faster." Many endocrine problems can change a client's behavior, personality, and psychological responses; the client stating that he or she has become short-tempered warrants further assessment. Worrying about losing a job is a normal concern but does not give any indication of a need for further assessment. The nurse will need to assess the client's claim that he or she has no stressors at present because the client's response does not provide enough information to make this determination; however, the client's statement about losing patience is the priority. Weight gain or loss may or may not be an indication of an endocrine disorder.

Ch.64 Question 9 of 25 The nurse is teaching a client with diabetes about proper foot care. Which statement by the client indicates that teaching was effective? a. "I should go barefoot in my house so that my feet are exposed to air." b. "I must inspect my shoes for foreign objects before putting them on." c. "I will soak my feet in warm water to soften calluses before trying to remove them." d. "I must wear canvas shoes as much as possible to decrease pressure on my feet."

b "I must inspect my shoes for foreign objects before putting them on." To avoid injury or trauma to the feet, shoes should be inspected for foreign objects before they are put on. Diabetic clients should not go barefoot because foot injuries can occur. To avoid injury or trauma, a callus should be removed by a podiatrist, not by the client. The diabetic client must wear firm support shoes to prevent injury.

Ch.64 Question 4 of 25 The nurse is teaching a client about the manifestations and emergency treatment of hypoglycemia. In assessing the client's knowledge, the nurse asks the client what he or she should do if feeling hungry and shaky. Which response by the client indicates a correct understanding of hypoglycemia management? a. "I should drink a glass of water." b. "I should eat three graham crackers." c. "I should give myself 1 mg of glucagon." d. "I should sit down and rest."

b "I should eat three graham crackers." Eating three graham crackers is a correct management strategy for mild hypoglycemia. Water or resting does not remedy hypoglycemia. Glucagon should be administered only in cases of severe hypoglycemia.

Ch.61 Question 11 of 16 The nurse is teaching a client about the correct procedure for a 24-hour urine test for creatinine clearance. Which statement by the client indicates a need for further teaching? a. "I should keep the urine container cool in a separate refrigerator or cooler." b. "I should not eat any protein when I am collecting urine for this test." c. "I won't save the first urine sample." d. "To end the collection, I must empty my bladder, adding it to the collection."

b "I should not eat any protein when I am collecting urine for this test." Eating protein does not interfere with collection or testing of the urine sample. Because the specimen must be kept cool, it can be placed in an inexpensive cooler with ice; the client should not keep the specimen container with food or beverages. The timing of the 24-hour collection begins after the initial void. To end a 24-hour urine specimen, emptying the bladder and adding it to the collection is the proper procedure.

Question 11 of 16 The nurse is teaching a client about the correct procedure for a 24-hour urine test for creatinine clearance. Which statement by the client indicates a need for further teaching? a. "I should keep the urine container cool in a separate refrigerator or cooler." b. "I should not eat any protein when I am collecting urine for this test." c. "I won't save the first urine sample." d. "To end the collection, I must empty my bladder, adding it to the collection."

b "I should not eat any protein when I am collecting urine for this test." Eating protein does not interfere with collection or testing of the urine sample. Because the specimen must be kept cool, it can be placed in an inexpensive cooler with ice; the client should not keep the specimen container with food or beverages. The timing of the 24-hour collection begins after the initial void. To end a 24-hour urine specimen, emptying the bladder and adding it to the collection is the proper procedure.

Ch.64 Question 24 of 25 The nurse is providing discharge teaching to a client with newly diagnosed diabetes. Which statement by the client indicates a correct understanding about the need to wear a MedicAlert bracelet? a. "If I become hyperglycemic, it is a medical emergency." b. "If I become hypoglycemic, I could become unconscious." c. "Medical personnel may need confirmation of my insurance." d. "I may need to be admitted to the hospital suddenly."

b "If I become hypoglycemic, I could become unconscious." Hypoglycemia is the most common cause of medical emergency in clients with diabetes. A MedicAlert bracelet is helpful if the client becomes hypoglycemic and is unable to provide self-care. Hyperglycemia is not a medical emergency unless it is acidosis; people with diabetes tolerate mild hyperglycemia routinely. Insurance information and information needed for hospital admission do not appear on a MedicAlert bracelet.

Ch.61 Question 5 of 16 To best determine how well a client with diabetes mellitus is controlling blood glucose, which test does the nurse monitor? a. Fasting blood glucose b. Glycosylated hemoglobin (HbA1c) c. Oral glucose tolerance test d. Urine glucose level

b Glycosylated hemoglobin (HbA1c) Glycosylated hemoglobin indicates the average blood glucose over several months and is the best indicator of overall blood glucose control. Fasting blood glucose can be used to monitor glucose control, but it is not the best method (although this may be the method that clients are most familiar with). Oral glucose testing and urine glucose levels look at one period of time and are not the best methods to look at overall effectiveness of treatment.

Ch.63 Question 11 of 21 Family members of a client diagnosed with hyperthyroidism are alarmed at the client's frequent mood swings. What is the nurse's response? a. "How does that make you feel?" b. "The mood swings should diminish with treatment." c. "The medications will make the mood swings disappear completely." d. "Your family member is sick. You must be patient."

b "The mood swings should diminish with treatment." Telling the family that the client's mood swings should diminish over time with treatment will provide information to the family, as well as reassurance. Asking how the family feels is important; however, the response should focus on the client. Any medications or treatment may not completely remove the mood swings associated with hyperthyroidism. The family is aware that the client is sick; telling them to be patient introduces guilt and does not address the family's concerns.

Ch.64 Question 7 of 25 A client with type 1 diabetes mellitus received regular insulin at 7:00 a.m. The client should be monitored for hypoglycemia at which time? a. 7:30 a.m. b. 11:00 a.m. c. 2:00 p.m. d. 7:30 p.m.

b 11:00 a.m. Onset of regular insulin is ½ to 1 hour; peak is 2 to 4 hours. Therefore, 11:00 a.m. is the anticipated peak time for regular insulin received at 7:00 a.m. For regular insulin received at 7:00 a.m., 7:30 a.m., 2:00 p.m., and 7:30 p.m. are not the anticipated peak times.

6. A patient has a low serum cortisol level. Which hormone would the nurse expect to be secreted to correct this? a. Thyroid-stimulating hormone (TSH) b. Adrenocorticotropic hormone c. Parathyroid hormone d. Antidiuretic hormone

b Adrenocorticotropic hormone

Ch.63 Question 15 of 21 A client being treated for hyperthyroidism calls the home health nurse and mentions that his heart rate is slower than usual. What is the nurse's best response? a. Advise the client to go to a calming environment. b. Ask whether the client has increased cold sensitivity or weight gain. c. Instruct the client to see his health care provider immediately. d. Tell the client to check his pulse again and call back later.

b Ask whether the client has increased cold sensitivity or weight gain. Increased sensitivity to cold and weight gain are symptoms of hypothyroidism, indicating an overcorrection by the medication. The client must be assessed further because he may require a lower dose of medication. A calming environment will not have any effect on the client's heart rate. The client will want to notify the health care provider about the change in heart rate. If other symptoms such as chest pain, shortness of breath, or confusion accompany the slower heart rate, then the client should see the health care provider immediately. If the client was concerned enough to call because his heart rate was slower than usual, the nurse needs to stay on the phone with the client while he re-checks his pulse. This time could also be spent providing education about normal ranges for that client.

3. Which mechanism is used to transport the substance produced by the endocrine glands to their target tissue? a. Lymph system b. Bloodstream c. Direct seeding d. Gastrointestinal system

b Bloodstream

Ch.64 Question 14 of 25 A client newly diagnosed with diabetes is not ready or willing to learn diabetes control during the hospital stay. Which information is the priority for the nurse to teach the client and the client's family? a. Causes and treatment of hyperglycemia b. Causes and treatment of hypoglycemia c. Dietary control d. Insulin administration

b Causes and treatment of hypoglycemia The causes and treatment of hypoglycemia must be understood by the client and family to manage the client's diabetes effectively. The causes and treatment of hyperglycemia is a topic for secondary teaching and is not the priority for the client with diabetes. Dietary control and insulin administration are important, but are not the priority in this situation.

39. What is the correct nursing action before beginning a 24-hour urine collection for endocrine studies? a. Place each voided specimen in a separate collection container. b. Check whether any preservatives are needed in the collection container. c. Start the collection with the first voided urine. d. Weigh the patient before beginning the collection.

b Check whether any preservatives are needed in the collection container.

Ch.64 Question 16 of 25 The nurse has just taken change-of-shift report on a group of clients on the medical-surgical unit. Which client does the nurse assess first? a. Client taking repaglinide (Prandin) who has nausea and back pain b. Client taking glyburide (Diabeta) who is dizzy and sweaty c. Client taking metformin (Glucophage) who has abdominal cramps d. Client taking pioglitazone (Actos) who has bilateral ankle swelling

b Client taking glyburide (Diabeta) who is dizzy and sweaty The client taking glyburide (Diabeta) who is dizzy and sweaty has symptoms consistent with hypoglycemia and should be assessed first because this client displays the most serious adverse effect of antidiabetic medications. Although the client taking repaglinide who has nausea and back pain requires assessment, the client taking glyburide takes priority. Metformin may cause abdominal cramping and diarrhea, but the client taking it does not require immediate assessment. Ankle swelling is an expected side effect of pioglitazone.

18. Which is the major function of the hormones produced by the adrenal cortex? a. "Fight-or-flight" response b. Control of potassium, sodium, and water c. Regulation of cell growth d. Calcium and stress regulation

b Control of potassium, sodium, and water

Ch.64 Question 11 of 25 A client has just been diagnosed with diabetes. Which factor is most important for the nurse to assess in the client before providing instruction about the disease and its management? a. Current lifestyle b. Educational and literacy level c. Sexual orientation d. Current energy level

b Educational and literacy level A large amount of information must be synthesized; typically written instructions are given. The client's educational and literacy level is essential information. Although lifestyle should be taken into account, it is not the priority. Sexual orientation will have no bearing on the ability of the client to provide self-care. Although energy level will influence the ability to exercise, it is not essential.

Ch.63 Question 21 of 21 A client admitted with hyperthyroidism is fidgeting with the bedcovers and talking extremely fast. What does the nurse do next? a. Calls the provider b. Encourages the client to rest c. Immediately assesses cardiac status d. Tells the client to slow down

b Encourages the client to rest The client with hyperthyroidism often has wide mood swings, irritability, decreased attention span, and manic behavior. The nurse should accept the client's behavior and provide a calm, quiet, and comfortable environment. Because the client's behavior is expected, there is no need to call the provider. Monitoring the client's cardiac status is part of the nurse's routine assessment. Telling the client to slow down is unsupportive and unrealistic.

Question 5 of 16 To best determine how well a client with diabetes mellitus is controlling blood glucose, which test does the nurse monitor? a. Fasting blood glucose b. Glycosylated hemoglobin (HbA1c) c. Oral glucose tolerance test d. Urine glucose level

b Glycosylated hemoglobin (HbA1c) Glycosylated hemoglobin indicates the average blood glucose over several months and is the best indicator of overall blood glucose control. Fasting blood glucose can be used to monitor glucose control, but it is not the best method (although this may be the method that clients are most familiar with). Oral glucose testing and urine glucose levels look at one period of time and are not the best methods to look at overall effectiveness of treatment.

Ch.62 Question 18 of 23 The client is taking fludrocortisone (Florinef) for adrenal hypofunction. The nurse instructs the client to report which symptom while taking this drug? a. Anxiety b. Headache c. Nausea d. Weight loss

b Headache A side effect of fludrocortisone is hypertension. New onset of headache should be reported, and the client's blood pressure should be monitored. Anxiety is not a side effect of fludrocortisone and is not associated with adrenal hypofunction. Nausea is associated with adrenal hypofunction; it is not a side effect of fludrocortisone. Sodium-related fluid retention and weight gain, not loss, are possible with fludrocortisone therapy.

Ch.63 Question 16 of 21 A client had a parathyroidectomy 18 hours ago. Which finding requires immediate attention? a. Edema at the surgical site b. Hoarseness c. Pain on moving the head d. Sore throat

b Hoarseness Hoarseness or stridor is an indication of respiratory distress and requires immediate attention. Edema at the surgical site of any surgery is an expected finding. Pain when the client moves the head or attempts to lift the head off the bed is an expected finding after a parathyroidectomy. Any time a client has been intubated for surgery, a sore throat is a common occurrence in the postoperative period. This is especially true for clients who have had surgery involving the neck.

Ch.61 Question 4 of 16 In type 1 diabetes, insulin injections are necessary to maintain which action between insulin and glucose? a. Glucose intolerance b. Homeostasis c. Insulin intolerance d. Negative feedback

b Homeostasis Insulin injections maintain homeostasis, or normal balance, between insulin and glucose in the client with type 1 diabetes. Type 1 diabetes is a lack of insulin production, not glucose intolerance, and requires frequent doses of insulin. Negative feedback does not occur in type 1 diabetes because of lack of insulin.

Question 4 of 16 In type 1 diabetes, insulin injections are necessary to maintain which action between insulin and glucose? a. Glucose intolerance b. Homeostasis c. Insulin intolerance d. Negative feedback

b Homeostasis Insulin injections maintain homeostasis, or normal balance, between insulin and glucose in the client with type 1 diabetes. Type 1 diabetes is a lack of insulin production, not glucose intolerance, and requires frequent doses of insulin. Negative feedback does not occur in type 1 diabetes because of lack of insulin.

Ch.61 Question 15 of 16 Which statement is true about hormones and their receptor sites? a. Hormone activity is dependent only on the function of the receptor site. b. Hormones need a specific receptor site to work. c. Hormones need to be plasma-bound to activate the receptor site. d. Hormone stores are available for activation until needed.

b Hormones need a specific receptor site to work. In general, each receptor site type is specific for only one hormone. Hormone receptor actions work in a "lock and key" manner, in that only the correct hormone (key) can bind to and activate the receptor site (lock). Hormones travel through the blood to all body areas, but exert their actions only on target tissues. Not all hormones are plasma-bound; for example, thyroid hormones are plasma protein-bound, whereas posterior pituitary hormones are transported by axons. Only certain cells manufacture specific hormones and store the hormones in vesicles.

Question 15 of 16 Which statement is true about hormones and their receptor sites? a. Hormone activity is dependent only on the function of the receptor site. b. Hormones need a specific receptor site to work. c. Hormones need to be plasma-bound to activate the receptor site. d. Hormone stores are available for activation until needed.

b Hormones need a specific receptor site to work. In general, each receptor site type is specific for only one hormone. Hormone receptor actions work in a "lock and key" manner, in that only the correct hormone (key) can bind to and activate the receptor site (lock). Hormones travel through the blood to all body areas, but exert their actions only on target tissues. Not all hormones are plasma-bound; for example, thyroid hormones are plasma protein-bound, whereas posterior pituitary hormones are transported by axons. Only certain cells manufacture specific hormones and store the hormones in vesicles.

Ch.61 Question 6 of 16 A client is hospitalized for pituitary function testing. Which nursing action included in the client's plan of care will be most appropriate for the RN to delegate to the LPN/LVN? a. Assess the client for clinical manifestations of hypopituitarism. b. Inject regular insulin for the growth hormone stimulation test. c. Palpate the thyroid gland for size and firmness. d. Teach the client about the adrenocorticotropic hormone stimulation test.

b Inject regular insulin for the growth hormone stimulation test. Injection of insulin is within the LPN/LVN scope of practice. Client assessment for clinical manifestations of hypopituitarism, palpating the thyroid gland, and client education are complex skills requiring training and expertise, and are best performed by an RN.

Question 6 of 16 A client is hospitalized for pituitary function testing. Which nursing action included in the client's plan of care will be most appropriate for the RN to delegate to the LPN/LVN? a. Assess the client for clinical manifestations of hypopituitarism. b. Inject regular insulin for the growth hormone stimulation test. c. Palpate the thyroid gland for size and firmness. d. Teach the client about the adrenocorticotropic hormone stimulation test.

b Inject regular insulin for the growth hormone stimulation test. Injection of insulin is within the LPN/LVN scope of practice. Client assessment for clinical manifestations of hypopituitarism, palpating the thyroid gland, and client education are complex skills requiring training and expertise, and are best performed by an RN.

27. A patient with exophthalmos from hyperthyroidism reports dry eyes, especially in the morning. The nurse teaches the patient to perform which intervention to help correct this problem? a. Wear sunglasses at all times when outside in the bright sun. b. Use cool compresses to the eye four times a day. c. Tape the eyes closed with nonallergenic tape. d. There is nothing that can be done to relieve this problem.

c

Ch.62 Question 21 of 23 A client has undergone a transsphenoidal hypophysectomy. Which intervention does the nurse implement to avoid increasing intracranial pressure (ICP) in the client? a. Encourages the client to cough and deep-breathe b. Instructs the client not to strain during a bowel movement c. Instructs the client to blow the nose for postnasal drip d. Places the client in the Trendelenburg position

b Instructs the client not to strain during a bowel movement Straining during a bowel movement increases ICP and must be avoided. Laxatives may be given and fluid intake encouraged to help with this. Although deep breathing is encouraged, the client must avoid coughing early after surgery because this increases pressure in the incision area and may lead to a cerebrospinal fluid (CSF) leak. If the client has postnasal drip, he or she must inform the nurse and not blow the nose; postnasal drip may indicate leakage of CSF. The head of the bed must be elevated after surgery.

Ch.63 Question 13 of 21 A client has been diagnosed with hypothyroidism. What medication is usually prescribed to treat this disorder? a. Atenolol (Tenormin) b. Levothyroxine sodium (Synthroid) c. Methimazole (Tapazole) d. Propylthiouracil

b Levothyroxine sodium (Synthroid) Levothyroxine is a synthetic form of thyroxine (T4) that is used to treat hypothyroidism. Atenolol is a beta blocker that is used to treat cardiovascular disease. Methimazole and propylthiouracil are used to treat hyperthyroidism.

42. A patient is suspected of having a pituitary tumor. Which radiographic test aids in determining this diagnosis? a. Skull x-rays b. MRI/CT c. Angiography d. Ultrasound

b MRI/CT

Ch.63 Question 3 of 21 A client recently admitted with hyperparathyroidism has a very high urine output. Of these actions, what does the nurse do next? a. Calls the health care provider b. Monitors intake and output c. Performs an immediate cardiac assessment d. Slows the rate of IV fluids

b Monitors intake and output Diuretic and hydration therapies are used most often for reducing serum calcium levels in clients with hyperparathyroidism. Usually, a diuretic that increases kidney excretion of calcium is used together with IV saline in large volumes to promote renal calcium excretion. The health care provider does not need to be notified in this situation, given the information available in the question. Cardiac assessment is part of the nurse's routine evaluation of the client. Slowing the rate of IV fluids is contraindicated because the client will become dehydrated due to the use of diuretics to increase kidney excretion of calcium.

Ch.62 Question 22 of 23 A client presents to the emergency department with acute adrenal insufficiency and the following vital signs: P 118 beats/min, R 18 breaths/min, BP 84/44 mm Hg, pulse oximetry 98%, and T 98.8° F oral. Which nursing intervention is the highest priority for this client? a. Administering furosemide (Lasix) b. Providing isotonic fluids c. Replacing potassium losses d. Restricting sodium

b Providing isotonic fluids Providing isotonic fluid is the priority intervention because this client's vital signs indicate volume loss that may be caused by nausea and vomiting and may accompany acute adrenal insufficiency. Isotonic fluids will be needed to administer IV medications such as hydrocortisone. Furosemide is a loop diuretic, which this client does not need. Potassium is normally increased in acute adrenal insufficiency, but potassium may have been lost if the client has had diarrhea; laboratory work will have to be obtained. Any restrictions, including sodium, should not be started without obtaining laboratory values to establish the client's baseline.

37. Which statement about performing a physical assessment of the thyroid gland is correct? a. The thyroid gland is easily palpated in all patients. b. The patient is instructed to swallow sips of water to aid palpation. c. The anterior approach is preferred for thyroid palpation. d. The thumbs are used to palpate the thyroid lobes.

b The patient is instructed to swallow sips of water to aid palpation.

8. The nurse is performing an assessment of an adult patient with new-onset acromegaly. What does the nurse expect to find? a. Extremely long arms and legs b. Thickened lips c. Changes in menses with infertility d. Rough, extremely dry skin

b Thickened lips

29. In addition to the pancreas that secretes insulin, which gland secretes hormones that affect protein, carbohydrate, and fat metabolism? a. Posterior pituitary b. Thyroid c. Ovaries d. Parathyroid

b Thyroid

36. The nurse is performing a physical assessment of a patient's endocrine system. Which gland can be palpated? a. Pancreas b. Thyroid c. Adrenal glands d. Parathyroids

b Thyroid

33. Which statement about age-related changes in older adults and the endocrine system is true? a. All hormone levels are elevated. b. Thyroid hormone levels decrease. c. Adrenal glands enlarge. d. The thyroid gland enlarges.

b Thyroid hormone levels decrease.

44. A patient is at risk for falling related to the effect of pathologic fractures as a result of bone demineralization. Which endocrine problem is this pertinent to? a. Underproduction of PTH b. Overproductive of PTH c. Underproduction of thyroid hormone d. Overproduction of thyroid hormone

b Overproductive of PTH

25. Which are the target organs of PTH in the regulation of calcium and phosphorus? (Select all that apply.) a. Stomach b. Kidney c. Bone d. Gastrointestinal tract e. Thyroid gland

b, c, d Kidney, Bone, Gastrointestinal tract

21. Which statements about the thyroid gland and its hormones are correct? (Select all that apply.) a. The gland is located in the posterior neck below the cricoid cartilage. b. The gland has two lobes joined by a thin tissue called the isthmus. c. T4 and T3 are two thyroid hormones. d. Thyroid hormones increase red blood cell production. e. Thyroid hormone production depends on dietary intake of iodine and potassium.

b, c, d The gland has two lobes joined by a thin tissue called the isthmus. T4 and T3 are two thyroid hormones. Thyroid hormones increase red blood cell production.

29. Which patient's history puts him or her at risk for developing SIADH? a. 27-year-old patient on high-dose steroids b. 47-year-old hospitalized adult patient with acute renal failure c. 58-year-old with metastatic lung or breast cancer d. Older adult with history of a stroke within the last year

c

31. The effect of increased ADH in the blood results in which effect on the kidney? a. Urine concentration tends to decrease. b. Glomerular filtration tends to decrease. c. Tubular reabsorption of water increases. d. Tubular reabsorption of sodium increases.

c

33. Which are preoperative instructions for a patient having thyroid surgery? (Select all that apply.) a. Teach postoperative restrictions such as no coughing and deep-breathing exercises to prevent strain on the suture line. b. Teach the moving and turning technique of manually supporting the head and avoiding neck extension to minimize strain on the suture line. c. Inform the patient that hoarseness for a few days after surgery is usually the result of a breathing tube (endotracheal tube). d. Humidification of air may be helpful to promote expectoration of secretions. Suctioning may also be used. e. Clarify any questions regarding placement of incision, complications, and postoperative care. f. A supine position and lying flat will be maintained postoperatively to avoid strain on suture line.

b, c, d, e

Which are preoperative instructions for a patient having thyroid surgery? (Select all that apply.) a. Teach postoperative restrictions such as no coughing and deep-breathing exercises to prevent strain on the suture line. b. Teach the moving and turning technique of manually supporting the head and avoiding neck extension to minimize strain on the suture line. c. Inform the patient that hoarseness for a few days after surgery is usually the result of a breathing tube (endotracheal tube) used during surgery. d. Humidification of air may be helpful to promote expectoration of secretions. Suctioning may also be used. e. Clarify any questions regarding placement of incision, complications, and postoperative care. f. A supine position and lying flat will be maintained postoperatively to avoid strain on suture line.

b, c, d, e

32. Which endocrine tissues are most commonly found to have reduced function as a result of aging? (Select all that apply.) a. Hypothalamus b. Ovaries c. Testes d. Pancreas e. Thyroid gland

b, c, d, e Ovaries, Testes, Pancreas, Thyroid gland

18. After a hypophysectomy, home care monitoring by the nurse includes assessing which factors? (Select all that apply.) a. Hypoglycemia b. Bowel habits c. Possible leakage of cerebrospinal fluid (CSF) d. 24-hour intake of fluids and urine output e. 24-hour diet recall f. Activity level

b, c, d, e, f

19. Which statements about the hormone cortisol being secreted by the adrenal cortex are accurate? (Select all that apply.) a. Cortisol peaks occur late in the day, with lowest points 12 hours after each peak. b. Cortisol has an effect on the body's immune function. c. Stress causes an increase in the production of cortisol. d. Blood levels of cortisol have no effect on its secretion. e. Cortisol affects carbohydrate, protein, and fat metabolism.

b, c, e Cortisol has an effect on body's immune function. Stress causes an increase in the production of cortisol. Cortisol affects carbohydrate, protein, and fat metabolism.

4. Which hormones are secreted by the posterior pituitary gland? (Select all that apply.) a. Testosterone b. Oxytocin c. Growth hormone (GH) d. Antidiuretic hormone (ADH) e. Cortisol

b, d Oxytocin, Antidiuretic hormone (ADH)

47. Which foods will the nurse instruct a patient with hypoparathyroidism to avoid? (Select all that apply.) a. Canned vegetables b. Yogurt c. Fresh fruit d. Red meat e. Milk f. Processed cheese

b, e, f

9. A nurse assesses clients for potential endocrine disorders. Which client is at greatest risk for hyperparathyroidism? a. A 29-year-old female with pregnancy-induced hypertension b. A 41-year-old male receiving dialysis for end-stage kidney disease c. A 66-year-old female with moderate heart failure d. A 72-year-old male who is prescribed home oxygen therapy

b. Clients who have chronic kidney disease do not completely activate vitamin D and poorly absorb calcium from the GI tract. They are chronically hypocalcemic, and this triggers overstimulation of the parathyroid glands. Pregnancy-induced hypertension, moderate heart failure, and home oxygen therapy do not place a client at higher risk for hyperparathyroidism.

10. A nurse plans care for a client with hyperparathyroidism. Which intervention should the nurse include in this client's plan of care? a. Ask the client to ambulate in the hallway twice a day. b. Use a lift sheet to assist the client with position changes. c. Provide the client with a soft-bristled toothbrush for oral care. d. Instruct the unlicensed assistive personnel to strain the client's urine for stones.

b. Hyperparathyroidism causes increased resorption of calcium from the bones, increasing the risk for pathologic fractures. Using a lift sheet when moving or positioning the client, instead of pulling on the client, reduces the risk of bone injury. Hyperparathyroidism can cause kidney stones, but not every client will need to have urine strained. The priority is preventing injury. Ambulating in the hall and using a soft toothbrush are not specific interventions for this client.

15. A patient requires 100 g of oral glucose for suppression testing and GH levels are measured serially for 120 minutes. The results of the suppression testing are abnormal. The nurse assesses for the signs and symptoms of which endocrine disorder? a. Adrenal insufficiency b. DI c. Hyperpituitarism d. Hypothyroidism

c

16. The nurse is providing instructions to a patient taking levothyroxine (Synthroid). When does the nurse tell the patient to take this medication? a. With breakfast in the morning. b. At lunchtime immediately after eating. c. In the morning on an empty stomach. d. At dinnertime within 15 minutes after eating.

c

22. The action of antidiuretic hormone (ADH) influences normal kidney function by stimulating which mechanism? a. Glomerulus to control the filtration rate b. Proximal nephron tubules to reabsorb water c. Distal nephron tubules and collecting ducts to reabsorb water d. Constriction of glomerular capillaries to prevent loss of protein in urine

c

24. The patient has multiple thyroid nodules resulting in thyroid hyperfunction. What is the most likely cause of this hyperthyroidism? a. Thyroid carcinoma b. Graves' disease c. Toxic multinodular goiter d. Pituitary hyperthyroidism

c

27. A patient with exophthalmos from hyperthyroidism reports dry eyes, especially in the morning. THe nurse teaches the patient to perform which intervention to help correct this problem? a. Wear sunglasses at all times when outside in the bright sun. b. Use coll compresses to the eye four times a day. c. Tape the eyes closed with nonallergenic tape. d. There is nothing that can be done to relieve this problem.

c

34. Which type of IV fluid does the nurse use to treat a patient with SIADH when the serum sodium level is very low? a. D5 1/2 normal saline b. D5W c. 3% normal saline d. Normal saline

c

A patient with exophthalmos from hyperthyroidism reports dry eyes, especially in the morning. The nurse teaches the patient to perform which intervention to help correct this problem? a. Wear sunglasses at all times when outside in the bright sun. b. Use cool compresses to the eye four times a day. c. Tape the eyes closed with nonallergenic tape. d. There is nothing that can be done to relieve this problem.

c

The nurse is assessing a patient after thyroid surgery and discovers harsh, high-pitched respiratory sounds. What is the nurse's best first action? a. Administer oxygen at 5 L via nasal cannula. b. Administer IV calcium chloride. c. Notify the Rapid Response Team. d. Suction the patient for oral secretions.

c

The nurse is providing instructions to a patient taking levothyroxine (Synthroid). When does the nurse tell the patient to take this medication? a. With breakfast in the morning b. At lunchtime immediately after eating c. In the morning on an empty stomach d. At dinnertime within 15 minutes after eating

c

Ch.62 Question 20 of 23 The nurse is providing discharge instructions to a client on spironolactone (Aldactone) therapy. Which comment by the client indicates a need for further teaching? a. "I must call the provider if I am more tired than usual." b. "I need to increase my salt intake." c. "I should eat a banana every day." d. "This drug will not control my heart rate."

c "I should eat a banana every day." Spironolactone increases potassium levels, so potassium supplements and foods rich in potassium, such as bananas, should be avoided to prevent hyperkalemia. While taking spironolactone, symptoms of hyponatremia such as drowsiness and lethargy must be reported; the client may need increased dietary sodium. Spironolactone will not have an effect on the client's heart rate.

Ch.63 Question 18 of 21 The nurse is teaching a client about thyroid replacement therapy. Which statement by the client indicates a need for further teaching? a. "I should have more energy with this medication." b. "I should take it every morning." c. "If I continue to lose weight, I may need an increased dose." d. "If I gain weight and feel tired, I may need an increased dose."

c "If I continue to lose weight, I may need an increased dose." Weight loss indicates a need for a decreased dose, not an increased dose. One of the symptoms of hypothyroidism is lack of energy; thyroid replacement therapy should help the client have more energy. The correct time to take thyroid replacement therapy is in the morning. If the client is gaining weight and continues to feel tired, that is an indication that the dose may need to be increased.

Ch.64 Question 2 of 25 A client expresses fear and anxiety over the life changes associated with diabetes, stating, "I am scared I can't do it all and I will get sick and be a burden on my family." What is the nurse's best response? a. "It is overwhelming, isn't it?" b. "Let's see how much you can learn today, so you are less nervous." c. "Let's tackle it piece by piece. What is most scary to you?" d. "Other people do it just fine."

c "Let's tackle it piece by piece. What is most scary to you?" Suggesting the client tackle it piece by piece and asking what is most scary to him or her is the best response; this approach will allow the client to have a sense of mastery with acceptance. Referring to the illness as overwhelming is supportive, but is not therapeutic or helpful to the client. Trying to see how much the client can learn in one day may actually cause the client to become more nervous; an overload of information is overwhelming. Suggesting that other people handle the illness just fine is belittling and dismisses the client's concerns.

Ch.62 Question 10 of 23 A client with Cushing's disease begins to laugh loudly and inappropriately, causing the family in the room to be uncomfortable. What is the nurse's best response? a. "Don't mind this. The disease is causing this." b. "I need to check the client's cortisol level." c. "The disease can sometimes affect emotional responses." d. "Medication is available to help with this."

c "The disease can sometimes affect emotional responses." The client may have neurotic or psychotic behavior as a result of high blood cortisol levels. Being honest with the family helps them to understand what is happening. Telling the family not to mind the laughter and that the disease is causing it is vague and minimizes the family's concern. This is the perfect opportunity for the nurse to educate the family about the disease. Cushing's disease is the hypersecretion of cortisol, which is abnormally elevated in this disease and, because the diagnosis has already been made, blood levels do not need to be redrawn. Telling the family that medication is available to help with inappropriate laughing does not assist them in understanding the cause of or the reason for the client's behavior.

Ch.64 Question 1 of 25 A diabetic client has a glycosylated hemoglobin (HbA1C) level of 9.4%. What does the nurse say to the client regarding this finding? a. "Keep up the good work." b. "This is not good at all." c. "What are you doing differently?" d. "You need more insulin."

c "What are you doing differently?" Assessing the client's regimen or changes he or she may have made is the basis for formulating interventions to gain control of blood glucose. HbA1C levels for diabetic clients should be less than 7%; a value of 9.4% shows poor control over the past 3 months. Telling the client this is not good, although true, does not take into account problems that the client may be having with the regimen and sounds like scolding. Although it may be true that the client needs more insulin, an assessment of the client's regimen is needed before decisions are made about medications.

Ch.64 Question 6 of 25 A client with type 2 diabetes has been admitted for surgery, and the health care provider has placed the client on insulin in addition to the current dose of metformin (Glucophage). The client wants to know the purpose of taking the insulin. What is the nurse's best response? a. "Your diabetes is worse, so you will need to take insulin." b. "You can't take your metformin while in the hospital." c. "Your body is under more stress, so you'll need insulin to support your medication." d. "You must take insulin from now on because the surgery will affect your diabetes."

c "Your body is under more stress, so you'll need insulin to support your medication." Because of the stress of surgery and NPO status, short-term insulin therapy may be needed perioperatively for the client who uses oral antidiabetic agents. For those receiving insulin, dosage adjustments may be required until the stress of surgery subsides. No evidence suggests that the client's diabetes has worsened; however, surgery is stressful and may increase insulin requirements. Metformin may be taken in the hospital; however, not on days when the client is NPO for surgery. When the client returns to his or her previous health state, oral agents will be resumed.

Ch.64 Question 10 of 25 Which of these clients with diabetes does the endocrine unit charge nurse assign to an RN who has floated from the labor/delivery unit? a. A 58-year-old with sensory neuropathy who needs teaching about foot care b. A 68-year-old with diabetic ketoacidosis who has an IV running at 250 mL/hr c. A 70-year-old who needs blood glucose monitoring and insulin before each meal d. A 76-year-old who was admitted with fatigue and shortness of breath

c A 70-year-old who needs blood glucose monitoring and insulin before each meal A nurse from the labor/delivery unit would be familiar with blood glucose monitoring and insulin administration because clients with type 1 and gestational diabetes are frequently cared for in the labor/delivery unit. The 58-year-old with sensory neuropathy, the 68-year-old with diabetic ketoacidosis, and the 76-year-old with fatigue and shortness of breath all have specific teaching or assessment needs that are better handled by nurses more familiar with caring for older adults with diabetes.

Ch.61 Question 13 of 16 A client is hospitalized with a possible disorder of the adrenal cortex. Which nursing activity is best for the charge nurse to delegate to an experienced nursing assistant? a. Ask about risk factors for adrenocortical problems. b. Assess the client's response to physiologic stressors. c. Check the client's blood glucose levels every 4 hours. d. Teach the client how to do a 24-hour urine collection.

c Check the client's blood glucose levels every 4 hours. Blood glucose monitoring is within the nursing assistant's scope of practice if the nursing assistant has received education and evaluation in the skill. Assessing risk factors for adrenocortical problems is not part of a nursing assistant's education. Assessing the client's response to physiologic stressors requires the more complex skill set of licensed nursing staff. Teaching the proper method for a 24-hour urine collection is a multi-step process; this task should not be delegated.

Question 13 of 16 A client is hospitalized with a possible disorder of the adrenal cortex. Which nursing activity is best for the charge nurse to delegate to an experienced nursing assistant? a. Ask about risk factors for adrenocortical problems. b. Assess the client's response to physiologic stressors. c. Check the client's blood glucose levels every 4 hours. d. Teach the client how to do a 24-hour urine collection.

c Check the client's blood glucose levels every 4 hours. Blood glucose monitoring is within the nursing assistant's scope of practice if the nursing assistant has received education and evaluation in the skill. Assessing risk factors for adrenocortical problems is not part of a nursing assistant's education. Assessing the client's response to physiologic stressors requires the more complex skill set of licensed nursing staff. Teaching the proper method for a 24-hour urine collection is a multi-step process; this task should not be delegated.

Ch.62 Question 12 of 23 These data are obtained by the RN who is assessing a client who had a transsphenoidal hypophysectomy yesterday. What information has the most immediate implications for the client's care? a. Dry lips and oral mucosa on examination b. Nasal drainage that tests negative for glucose c. Client report of a headache and stiff neck d. Urine specific gravity of 1.016

c Client report of a headache and stiff neck Headache and stiff neck (nuchal rigidity) are symptoms of meningitis that have immediate implications for the client's care. Dry lips and mouth are not unusual after surgery. Frequent oral rinses and the use of dental floss should be encouraged because the client cannot brush the teeth. Any nasal drainage should test negative for glucose; nasal drainage that tests positive for glucose indicates the presence of a cerebrospinal fluid leak. A urine specific gravity of 1.016 is within normal limits.

Ch.63 Question 12 of 21 An RN and LPN/LVN are caring for a group of clients on the medical-surgical unit. Which client will be the best to assign to the LPN/LVN? a. Client with Graves' disease who needs discharge teaching after a total thyroidectomy b. Client with hyperparathyroidism who is just being admitted for a parathyroidectomy c. Client with infiltrative ophthalmopathy who needs administration of high-dose prednisone (Deltasone) d. Newly diagnosed client with hypothyroidism who needs education about the use of thyroid supplements

c Client with infiltrative ophthalmopathy who needs administration of high-dose prednisone (Deltasone) Medication administration for the client with infiltrative ophthalmopathy is within the scope of practice of the LPN/LVN. Discharge teaching is a complex task that cannot be delegated to the LPN/LVN. A client being admitted for a parathyroidectomy needs preoperative teaching, which must be provided by the RN. A client who has a new diagnosis will have questions about the disease and prescribed medications; teaching is a complex task that is appropriate for the RN.

Ch.62 Question 2 of 23 Which client does the nurse identify as being at highest risk for acute adrenal insufficiency resulting from corticosteroid use? a. Client with hematemesis, upper epigastric pain for the past 3 days not relieved with food, and melena b. Client with right upper quadrant pain unrelieved for the past 2 days, dark-brown urine, and clay-colored stools c. Client with shortness of breath and chest tightness, nasal flaring, audible wheezing, and oxygen saturation of 85% for the second time this week d. Client with three emergency department visits in the past month for edema, shortness of breath, weight gain, and jugular venous distention

c Client with shortness of breath and chest tightness, nasal flaring, audible wheezing, and oxygen saturation of 85% for the second time this week Corticosteroids may be used to treat signs and symptoms of asthma, such as shortness of breath and chest tightness, nasal flaring, audible wheezing, and oxygen saturation of 85%. This places the client at risk for adrenal insufficiency. Corticosteroids are not used to treat signs and symptoms of GI bleeding or peptic ulcer disease (hematemesis, upper epigastric pain for the past 3 days not relieved with food, and melena), gallbladder disease (right upper quadrant pain unrelieved for the past 2 days, dark brown urine, and clay-colored stools), or congestive heart failure (edema, shortness of breath, weight gain, and jugular venous distention).

Ch.61 Question 12 of 16 A client has suspected alterations in antidiuretic hormone (ADH) function. Which diagnostic test does the nurse anticipate will be requested for this client? a. Adrenocorticotropic hormone (ACTH) suppression test b. Chest x-ray c. Cranial computed tomography (CT) d. Renal sonography

c Cranial computed tomography (CT) ADH is a hormone of the posterior pituitary. Brain abscess, tumor, or subarachnoid hemorrhage could cause alterations in ADH levels. These can be seen on a CT scan of the brain. ACTH triggers the release of cortisol from the adrenal cortex and is not related to ADH. A chest x-ray would not show a pituitary tumor or brain abscess. Even though ADH acts on distal convoluted tubules in the kidneys, a renal sonogram would diagnose the cause of syndrome of inappropriate antidiuretic hormone.

Question 12 of 16 A client has suspected alterations in antidiuretic hormone (ADH) function. Which diagnostic test does the nurse anticipate will be requested for this client? a. Adrenocorticotropic hormone (ACTH) suppression test b. Chest x-ray c. Cranial computed tomography (CT) d. Renal sonography

c Cranial computed tomography (CT) ADH is a hormone of the posterior pituitary. Brain abscess, tumor, or subarachnoid hemorrhage could cause alterations in ADH levels. These can be seen on a CT scan of the brain. ACTH triggers the release of cortisol from the adrenal cortex and is not related to ADH. A chest x-ray would not show a pituitary tumor or brain abscess. Even though ADH acts on distal convoluted tubules in the kidneys, a renal sonogram would diagnose the cause of syndrome of inappropriate antidiuretic hormone.

Ch. 61 Question 2 of 16 Which negative feedback response is responsible for preventing hypoglycemia during sleep in nondiabetic clients? a. Alpha cells of the pancreas b. Beta cells of the pancreas c. Glucagon release d. Insulin release

c Glucagon release Glucagon is the hormone that binds to receptors on liver cells. This causes the liver cells to convert glycogen to glucose, which keeps blood sugar levels normal during sleep. Alpha cells are responsible for synthesizing and secreting the hormone glucagon. Beta cells are responsible for synthesizing and secreting the hormone insulin. Insulin is the hormone responsible for lowering blood glucose. Insulin improves glucose uptake by the cell.

Question 2 of 16 Which negative feedback response is responsible for preventing hypoglycemia during sleep in nondiabetic clients? a. Alpha cells of the pancreas b. Beta cells of the pancreas c. Glucagon release d. Insulin release

c Glucagon release Glucagon is the hormone that binds to receptors on liver cells. This causes the liver cells to convert glycogen to glucose, which keeps blood sugar levels normal during sleep. Alpha cells are responsible for synthesizing and secreting the hormone glucagon. Beta cells are responsible for synthesizing and secreting the hormone insulin. Insulin is the hormone responsible for lowering blood glucose. Insulin improves glucose uptake by the cell.

2. What is the name of the substance secreted by the endocrine glands? a. Vasoactive amines b. Chemotaxins c. Hormones d. Cytotoxins

c Hormones

10. What are tropic hormones? a. Hormones that rigger female and male sex characteristics. b. Hormones that have a direct effect on final target tissues. c. Hormones produced by the anterior pituitary gland that stimulate other endocrine glands. d. Hormones that are synthesized in the hypothalamus and stored in the posterior pituitary gland

c Hormones produced by the anterior pituitary gland that stimulate other endocrine glands.

Ch.62 Question 16 of 23 Which laboratory result indicates that fluid restrictions have been effective in treating syndrome of inappropriate antidiuretic hormone (SIADH)? a. Decreased hematocrit b. Decreased serum osmolality c. Increased serum sodium d. Increased urine specific gravity

c Increased serum sodium Increased serum sodium due to fluid restriction indicates effective therapy. Hemoconcentration is a result of hypovolemic hyponatremia caused by SIADH and diabetes insipidus. Plasma osmolality is decreased as a result of SIADH. Urine specific gravity is decreased with diabetes insipidus and is increased with SIADH.

Ch.62 Question 7 of 21 A client with hypothyroidism is being discharged. Which environmental change may the client experience in the home? a. Frequent home care b. Handrails in the bath c. Increased thermostat setting d. Strict infection-control measures

c Increased thermostat setting Manifestations of hypothyroidism include cold intolerance. Increased thermostat settings or additional clothing may be necessary. A client with a diagnosis of hypothyroidism can be safely managed at home with adequate discharge teaching regarding medications and instructions on when to notify the health care provider or home health nurse. In general, hypothyroidism does not cause mobility issues. Activity intolerance and fatigue may be an issue, however. A client with hypothyroidism is not immune-compromised or contagious, so no environmental changes need to be made to the home.

Ch.61 Question 3 of 23 A client diagnosed with hyperpituitarism resulting from a prolactin-secreting tumor has been prescribed bromocriptine mesylate (Parlodel). As a dopamine agonist, what effect does this drug have by stimulating dopamine receptors in the brain? a. Decreases the risk for cerebrovascular disease b. Increases the risk for depression c. Inhibits the release of some pituitary hormones d. Stimulates the release of some pituitary hormones

c Inhibits the release of some pituitary hormones Bromocriptine mesylate inhibits the release of both prolactin and growth hormone. It does not decrease the risk for cerebrovascular disease leading to stroke. Increased risk for depression is not associated with the use of bromocriptine mesylate; however, hallucinations have been reported as a side effect. Bromocriptine mesylate does not stimulate the release of any hormones.

Ch.63 Question 2 of 21 An older client with an elevated serum calcium level is receiving IV furosemide (Lasix) and an infusion of normal saline at 150 mL/hr. Which nursing action can the RN delegate to unlicensed assistive personnel (UAP)? a. Ask the client about any numbness or tingling. b. Check for bone deformities in the client's back. c. Measure the client's intake and output hourly. d. Monitor the client for shortness of breath.

c Measure the client's intake and output hourly. Measuring intake and output is a commonly delegated nursing action that is within the UAP scope of practice. Numbness and tingling is part of the client assessment that needs to be completed by a licensed nurse. Bony deformities can be due to pathologic fractures; physical assessment is a complex task that cannot be delegated. An older client receiving an IV at 150 mL/hr is at risk for congestive heart failure; careful monitoring for shortness of breath is the responsibility of the RN.

Ch.63 Question 19 of 21 Which type of thyroid cancer often occurs as part of multiple endocrine neoplasia (MEN) type II? a. Anaplastic b. Follicular c. Medullary d. Papillary

c Medullary Medullary carcinoma commonly occurs as part of MEN type II, which is a familial endocrine disorder. Anaplastic carcinoma is an aggressive tumor that invades surrounding tissue. Follicular carcinoma occurs more frequently in older clients and may metastasize to bone and lung. Papillary carcinoma is the most common type of thyroid cancer. It is slow growing and, if the tumor is confined to the thyroid gland, the outlook for a cure is good with surgical management.

Ch.63 Question 9 of 21 Which action does the postanesthesia care unit (PACU) nurse perform first when caring for a client who has just arrived after a total thyroidectomy? a. Assess the wound dressing for bleeding. b. Give morphine sulfate 4 to 8 mg IV for pain. c. Monitor oxygen saturation using pulse oximetry. d. Support the head and neck with sandbags.

c Monitor oxygen saturation using pulse oximetry. Airway assessment and management is always the first priority with every client. This is especially important for a client who has had surgery that involves potential bleeding and edema near the trachea. Assessing the wound dressing for bleeding is a high priority, although this is not the first priority. Pain control and supporting the head and neck with sandbags are important priorities, but can be addressed after airway assessment.

Ch.64 Question 13 of 25 An intensive care client with diabetic ketoacidosis (DKA) is receiving an insulin infusion. The cardiac monitor shows ventricular ectopy. Which assessment does the nurse make? a. Urine output b. 12-lead electrocardiogram (ECG) c. Potassium level d. Rate of IV fluids

c Potassium level With insulin therapy, serum potassium levels fall rapidly as potassium shifts into the cells. Detecting and treating the underlying cause is essential. Insulin treats symptoms of diabetes by putting glucose into the cell as well as potassium; ectopy, indicative of cardiac irritability, is not associated with changes in urine output. A 12-lead ECG can verify the ectopy, but the priority is to detect and fix the underlying cause. Increased fluids treat the symptoms of dehydration secondary to DKA, but do not treat the cause.

Ch.64 Question 15 of 25 Which action is correct when drawing up a single dose of insulin? a. Wash hands thoroughly and don sterile gloves. b. Shake the bottle of insulin vigorously to mix the insulin. c. Pull back plunger to draw air into the syringe equal to the insulin dose. d. Recap the needle and save the syringe for the next dose of insulin.

c Pull back plunger to draw air into the syringe equal to the insulin dose. The plunger is pulled back to draw an amount of air into the syringe that is equal to the insulin dose. The air is then injected into the insulin bottle before withdrawing the insulin dose. Although handwashing is important before any medication administration, sterile gloves are not required. The bottle of insulin should be rolled gently in the palms of the hands to mix the insulin, not shaken. Insulin syringes are never recapped or reused; the syringe and needle should be disposed of (without recapping) in a puncture-proof container.

13. Which statement about hormone replacement therapy for hypopituitarism is correct? a. Once manifestations of hypofunction are corrected, treatment is no longer needed. b. The most effective route of androgen replacement is the oral route. c. Testosterone replacement is contraindicated in men with prostate cancer. d. Clomiphene citrate (Clomid) is used to to suppress ovulation in women.

c Testosterone replacement is contraindicated in men with prostate cancer.

Question 23 of 25 Which explanation best assists a client in differentiating type 1 diabetes from type 2 diabetes? a. Most clients with type 1 diabetes are born with it. b. People with type 1 diabetes are often obese. c. Those with type 2 diabetes make insulin, but in inadequate amounts. d. People with type 2 diabetes do not develop typical diabetic complications.

c Those with type 2 diabetes make insulin, but in inadequate amounts. People with type 2 diabetes make some insulin but in inadequate amounts, or they have resistance to existing insulin. Although type 1 diabetes may occur early in life, it may be caused by immune responses. Obesity is typically associated with type 2 diabetes. People with type 2 diabetes are at risk for complications, especially cardiovascular complications.

Ch.63 Question 1 of 21 A client has hyperparathyroidism. Which incident witnessed by the nurse requires the nurse's intervention? a. The client eating a morning meal of cereal and fruit b. The physical therapist walking with the client in the hallway c. Unlicensed assistive personnel pulling the client up in bed by the shoulders d. Visitors talking with the client about going home

c Unlicensed assistive personnel pulling the client up in bed by the shoulders The client with hyperparathyroidism is at risk for pathologic fracture. All members of the health care team must move the client carefully. A lift sheet should be used to re-position the client. The client with hyperparathyroidism is not restricted from eating and should maintain a balanced diet. The client can benefit from moderate exercise and physical therapy, and is not restricted from having visitors.

36. The nurse assesses a patient postthyroidectomy for layrngeal nerve damage. Which findings indicate this complication? (Select all that apply.) a. Dyspnea b. Sore throat c. Hoarseness d. Weak voice e. Dry cough

c, d

The nurse assesses a patient postthyroidectomy for laryngeal nerve damage. Which findings indicate this complication? (Select all that apply.) a. Dyspnea b. Sore throat c. Hoarseness d. Weak voice e. Dry cough

c, d

2. A malfunctioning posterior pituitary gland can result in which disorders? (Select all that apply.) a. Hypothyroidism b. Altered sexual function c. Diabetes insipidus (DI) d. Growth retardation e. Syndrome of inappropriate antidiuretic hormone (SIADH)

c, e Diabetes insipidus Syndrome of inappropriate antidiuretic hormone (SIADH)

2. A nurse cares for a client with elevated triiodothyronine and thyroxine, and normal thyroid-stimulating hormone levels. Which actions should the nurse take? (Select all that apply.) a. Administer levothyroxine (Synthroid). b. Administer propranolol (Inderal). c. Monitor the apical pulse. d. Assess for Trousseau's sign. e. Initiate telemetry monitoring.

c, e. The client's laboratory findings suggest that the client is experiencing hyperthyroidism. The increased metabolic rate can cause an increase in the client's heart rate, and the client should be monitored for the development of dysrhythmias. Placing the client on a telemetry monitor might also be a precaution. Levothyroxine is given for hypothyroidism. Propranolol is a beta blocker often used to lower sympathetic nervous system activity in hyperthyroidism. Trousseau's sign is a test for hypocalcemia.

The nurse is performing a physical examination of a patient's thyroid gland. Precautions are taken in performing the correct technique because palpation can result in which occurrence? a. Damage to the esophagus causing gastric reflux b. Obstruction of the carotid arteries causing a stroke c. Pressure on the trachea and laryngeal nerve causing hoarseness d. Exacerbation of symptoms by releasing additional thyroid hormone

d

15. A nurse plans care for a client who has hypothyroidism and is admitted for pneumonia. Which priority intervention should the nurse include in this client's plan of care? a. Monitor the client's intravenous site every shift. b. Administer acetaminophen (Tylenol) for fever. c. Ensure that working suction equipment is in the room. d. Assess the client's vital signs every 4 hours.

c. A client with hypothyroidism who develops another illness is at risk for myxedema coma. In this emergency situation, maintaining an airway is a priority. The nurse should ensure that suction equipment is available in the client's room because it may be needed if myxedema coma develops. The other interventions are necessary for any client with pneumonia, but having suction available is a safety feature for this client.

8. A nurse cares for a client who has hypothyroidism as a result of Hashimoto's thyroiditis. The client asks, "How long will I need to take this thyroid medication?" How should the nurse respond? a. "You will need to take the thyroid medication until the goiter is completely gone." b. "Thyroiditis is cured with antibiotics. Then you won't need thyroid medication." c. "You'll need thyroid pills for life because your thyroid won't start working again." d. "When blood tests indicate normal thyroid function, you can stop the medication."

c. Hashimoto's thyroiditis results in a permanent loss of thyroid function. The client will need lifelong thyroid replacement therapy. The client will not be able to stop taking the medication.

6. A nurse plans care for a client with hypothyroidism. Which priority problem should the nurse plan to address first for this client? a. Heat intolerance b. Body image problems c. Depression and withdrawal d. Obesity and water retention

c. Hypothyroidism causes many problems in psychosocial functioning. Depression is the most common reason for seeking medical attention. Memory and attention span may be impaired. The client's family may have great difficulty accepting and dealing with these changes. The client is often unmotivated to participate in self-care. Lapses in memory and attention require the nurse to ensure that the client's environment is safe. Heat intolerance is seen in hyperthyroidism. Body image problems and weight issues do not take priority over mental status and safety.

1. The nurse is performing a physical examination of a patient's thyroid gland. Precautions are taken in performing the correct technique because palpitation can result in which occurrence? a. Damage to the esophagus causing gastric reflux b. Obstruction of the carotid arteries causing a stroke c. Pressure on the trachea and laryngeal nerve causing hoarseness d. Exacerbation of symptoms by releasing additional thyroid hormone

d

11. The nurse assessing a patient palpates enlargement of the thyroid gland, along with noticeable swelling of the neck. How does the nurse interpret this finding? a. Globe lag b. Myxedema c. Exophthalmos d. Goiter

d

14. A female patient has been prescribed hormone replacement therapy. What does the nurse instruct the patient to do regarding this therapy? a. Report any recurrence of symptoms, such as decreased libido, between injections. b. Monitor blood pressure at least weekly for potential hypotension. c. Treat leg pain, especially in the calves, with gentle muscle stretching. d. Take measures to reduce risk for hypertension and thrombosis.

d

23. Laboratory findings of elevated T3 and T4, decreased TSH, and high thyrotropin receptor antibody titer indicate which condition? a. Multinodular goiter b. Hyperthyroidism related to overmedication c. Pituitary tumor suppressing TSH d. Graves' disease

d

35. After a thyroidectomy, a patient reports tingling around the mouth and muscle twitching. Which complication do these assessment findings indicate to the nurse? a. Hemorrhage b. Respiratory distress c. Thyroid storm d. Hypocalcemia

d

39. The nurse expects to perform which diagnostic test for pheochromocytoma? a. 24-hour urine collection for sodium, potassium, and glucose b. Catecholamine-stimulation test c. Administration of beta-adrenergic blocking agent and monitor results. d. 24- hour urine collection for fractionated metanephrine and catecholamine levels.

d

42. Serum calcium levels are maintained by which hormone? a. Cortisol b. Luteinizing hormone c. Antidiuretic hormone (ADH) d. Parathyroid hormone (PTH)

d

42. Which diuretic is ordered by the health care provider to treat hyperaldosteronism? a. Furosemide (Lasix) b. Ethacrynic acid (Edecrin) c. Bumetanide (Bumex) d. Spironolactone (Aldactone)

d

43. Which statement about hyperaldosteronism is correct? a. Painful "charley horses" are common from hyperkalemia. b. It occurs more often in men than in women. c. It is a common cause of hypertension in the population. d. Hypokalemia and hypertension are the main issues.

d

44. When diagnosed with Cushing's syndrome, the manifestations are most likely related to an excess production of which hormone? a. Insulin from the pancreas b. ADH from posterior pituitary gland c. PRL from anterior pituitary gland d. Cortisol from the adrenal cortex

d

58. The nurse determines that the administration of hydrocortisone for the Addisonian crisis is effective when which assessment is made? a. Increased urine output b. No signs of pitting edema c. Weight gain d. Lethargy improving; patient alert and oriented

d

59. Which nursing intervention is a preventive measure for adrenocortical insufficiency? a. Maintaining diuretic therapy b. Instructing the patient on salt restriction c. Reducing high-dose glucocorticoid therapy quickly d. Reducing high-dose glucocorticoid doses gradually

d

7. Which laboratory result is consistent with a diagnosis of hyperthyroidism? a. Decreased serum triiodothyronine (T3) and thyroxine (T4) levels b. Elevated serum thyrotropin-releasing hormone (TRH) level c. Decreased radioactive idoine uptake d. Increased serum T3 and T4

d

After a thyroidectomy, a patient reports tingling around the mouth and muscle twitching. Which complication do these assessment findings indicate to the nurse? a. Hemorrhage b. Respiratory distress c. Thyroid storm d. Hypocalcemcia

d

Laboratory findings of elevated T3 and T4, decreased TSH, and high thyrotropin receptor antibody titer indicate which condition? a. Multinodular goiter b. Hyperthyroidism related to overmedication c. Pituitary tumor suppressing TSH d. Graves' disease

d

Ch.63 Question 14 of 21 A client with thyroid cancer has just received 131I ablative therapy. Which statement by the client indicates a need for further teaching? a. "I cannot share my toothpaste with anyone." b. "I must flush the toilet three times after I use it." c. "I need to wash my clothes separately from everyone else's clothes." d. "I'm ready to hold my newborn grandson now."

d "I'm ready to hold my newborn grandson now." Clients undergoing 131I therapy should avoid close contact with pregnant women, infants, and young children for 1 week after treatment. Clients should remain at least 1 meter (39 inches, or roughly 3 feet) away, and limit exposure to less than 1 hour per day. Some radioactivity will remain in the client's salivary glands for up to 1 week after treatment. Care should be taken to avoid exposing others to the saliva. Flushing the toilet three times after use will ensure that all urine has been diluted and removed. Clothing needs to be washed separately and the washing machine then needs to be run empty for a full cycle before it is used to wash the clothing of others.

Ch.63 Question 20 of 21 A client is taking methimazole (Tapazole) for hyperthyroidism and would like to know how soon this medication will begin working. What is the nurse's best response? a. "You should see effects of this medication immediately." b. "You should see effects of this medication within 1 week." c. "You should see full effects from this medication within 1 to 2 days." d. "You should see some effects of this medication within 2 weeks."

d "You should see some effects of this medication within 2 weeks." Methimazole is an iodine preparation that decreases blood flow through the thyroid gland. This action reduces the production and release of thyroid hormone. The client should see some effects within 2 weeks; however, it may take several more weeks before metabolism returns to normal. Although onset of action is 30 to 40 minutes after an oral dose, the client will not see effects immediately. Effects will take longer than 1 week to become apparent when methimazole is used. Methimazole needs to be taken every 8 hours for an extended period of time. Levels of triiodothyronine (T3) and thyroxine (T4) will be monitored and dosages adjusted as levels fall.

Ch.64 Question 12 of 25 In reviewing the health care provider admission requests for a client admitted in a hyperglycemic-hyperosmolar state, which request is inconsistent with this diagnosis? a. 20 mEq KCl for each liter of IV fluid b. IV regular insulin at 2 units/hr c. IV normal saline at 100 mL/hr d. 1 ampule NaHCO3 IV now

d 1 ampule NaHCO3 IV now NaHCO3 is given for the acid-base imbalance of diabetic ketoacidosis, not the hyperglycemic-hyperosmolar state, which presents with hyperglycemia and absence of ketosis/acidosis. KCl 20 mEq for each liter of IV fluid will correct hypokalemia from diuresis. IV regular insulin at 2 units/hr will correct hyperglycemia. IV normal saline at 100 mL/hr will correct dehydration.

Ch.64 Question 25 of 25 A client with type 2 diabetes who is taking metformin (Glucophage) is seen in the diabetic clinic. The fasting blood glucose is 108 mg/dL, and the glycosylated hemoglobin (HbA1C) is 8.2%. Which action does the nurse plan to take next? a. Instruct the client to continue with the current diet and metformin use. b. Discuss the need to check blood glucose several times every day. c. Talk about the possibility of adding rapid-acting insulin to the regimen. d. Ask the client about current dietary intake and medication use.

d Ask the client about current dietary intake and medication use. The nurse's first action should be to assess whether the client is adherent to the currently prescribed diet and medications. The client's current diet and medication use have not been successful in keeping glucose in the desired range. Checking blood glucose more frequently and/or using rapid-acting insulin may be appropriate, but this will depend on the assessment data. The HbA1C indicates that the client's average glucose level is not in the desired range, but discussing the need to check blood glucose several times every day assumes that the client is not compliant with the therapy and glucose monitoring. The nurse should not assume that adding insulin, which must be prescribed by the provider, is the answer without assessing the underlying reason for the treatment failure.

Ch.63 Question 10 of 21 The nurse reviews the vital signs of a client diagnosed with Graves' disease and sees that the client's temperature is up to 99.6° F. After notifying the health care provider, what does the nurse do next? a. Administers acetaminophen b. Alerts the Rapid Response Team c. Asks any visitors to leave d. Assesses the client's cardiac status completely

d Assesses the client's cardiac status completely If the client's temperature has increased by even 1°, the nurse's first action is to notify the provider. Continuous cardiac monitoring should be the next step. Administering a nonsalicylate antipyretic such as acetaminophen is appropriate, but is not a priority action for this client. Alerting the Rapid Response Team is not needed at this time. Asking visitors to leave would not be the next action, and if visitors are providing comfort to the client, this would be contraindicated.

Ch.64 Question 22 of 25 Which nursing action can the home health nurse delegate to a home health aide who is making daily visits to a client with newly diagnosed type 2 diabetes? a. Assist the client's spouse in choosing appropriate dietary items. b. Evaluate the client's use of a home blood glucose monitor. c. Inspect the extremities for evidence of poor circulation. d. Assist the client with washing the feet and applying moisturizing lotion.

d Assist the client with washing the feet and applying moisturizing lotion. Assisting with personal hygiene is included in the role of home health aides. Assisting with dietary choices, evaluating the effectiveness of teaching, and performing assessments are complex actions that should be implemented by licensed nurses.

23. Which hormone responds to elevated serum calcium blood level by decreasing bone resorption? a. PTH b. T4 c. T3 d. Calcitonin

d Calcitonin

Ch.63 Question 5 of 21 The RN has just received change-of-shift report on the medical-surgical unit. Which client will need to be assessed first? a. Client with Hashimoto's thyroiditis and a large goiter b. Client with hypothyroidism and an apical pulse of 51 beats/min c. Client with parathyroid adenoma and flank pain due to a kidney stone d. Client who had a parathyroidectomy yesterday and has muscle twitching

d Client who had a parathyroidectomy yesterday and has muscle twitching A client who is 1 day postoperative for parathyroidectomy and has muscle twitching is showing signs of hypocalcemia and is at risk for seizures. Rapid assessment and intervention are needed. Clients with Hashimoto's thyroiditis are usually stable; this client does not need to be assessed first. Although an apical pulse of 51 is considered bradycardia, a low heart rate is a symptom of hypothyroidism. A client with a kidney stone will be uncomfortable and should be asked about pain medication as soon as possible, but this client does not need to be assessed first.

Ch.62 Question 7 of 23 The charge nurse is making client assignments for the medical-surgical unit. Which client will be best to assign to an RN who has floated from the pediatric unit? a. Client in Addisonian crisis who is receiving IV hydrocortisone b. Client admitted with syndrome of inappropriate antidiuretic hormone (SIADH) secondary to lung cancer c. Client being discharged after a unilateral adrenalectomy to remove an adrenal tumor d. Client with Cushing's syndrome who has elevated blood glucose and requires frequent administration of insulin

d Client with Cushing's syndrome who has elevated blood glucose and requires frequent administration of insulin An RN who works with pediatric clients would be familiar with glucose monitoring and insulin administration. A client in Addisonian crisis would best be monitored by an RN from the medical-surgical floor. Although the float RN could complete the admission history, the client with SIADH secondary to lung cancer might require teaching and orientation to the unit that a nurse more familiar with that area would be better able to provide. Discharge teaching specific to adrenalectomy should be provided by the RN who is regularly assigned to the medical-surgical floor and is more familiar with taking care of postoperative adult clients with endocrine disorders.

Ch.64 Question 3 of 25 The nurse receives report on a 52-year-old client with type 2 diabetes: Physical Assessment - cold to touch - Right great toe mottled - Lungs- clear - Client states wears eyeglasses to read Diagnostic Findings - Glucose- 179 mg/dL - Hemoglobin A1c 6.9% mg/dL Provider Prescriptions - Regular insulin 8 units if blood glucose 250 to 275 mg/dL - Regular insulin 10 units if glucose 275 to 300 Which complication of diabetes does the nurse report to the provider? a. Poor glucose control b. Visual changes c. Respiratory distress d. Decreased peripheral perfusion

d Decreased peripheral perfusion A cold, mottled toe may indicate arterial occlusion secondary to arterial occlusive disease or embolization; this must be reported to avoid potential gangrene and amputation. Although one glucose reading is elevated, the hemoglobin A1c indicates successful glucose control over the past 3 months. After the age of 40, reading glasses may be needed due to difficulty in accommodating to close objects. Lungs are clear and no evidence of distress is noted.

7. Patients diagnosed with an anterior pituitary tumor can have symptoms of acremegaly or gigantism. These symptoms are a result of overproduction of which hormone? a. ACTH b. PRL c. Gonadotropins d. GH

d GH

Ch.63 Question 17 of 21 What effect can starting a dose of levothyroxine sodium (Synthroid) too high or increasing a dose too rapidly have on a client? a. Bradycardia and decreased level of consciousness b. Decreased respiratory rate c. Hypotension and shock d. Hypertension and heart failure

d Hypertension and heart failure Hypertension and heart failure are possible if the levothyroxine sodium dose is started too high or raised too rapidly, because levothyroxine would essentially put the client into a hyperthyroid state. The client would be tachycardic, not bradycardic. The client may have an increased respiratory rate. Shock may develop, but only as a late effect and as the result of "pump failure."

30. The bloodstream delivers glucose to the cells for energy production. Which hormone controls the cells' use of glucose? a. T4 b. Growth hormone c. Adrenal steroids d. Insulin

d Insulin

27. Which statement about glucagon secretion is correct? a. It is stimulated by an increase in blood glucose levels. b. It is stimulated by a decrease in amino acid levels. c. It exerts its primary effect on the pancreas. d. It acts to increase blood glucose levels.

d It acts to increase blood glucose levels.

7. The target tissue for ADH is which organ? a. Hypothalamus b. Thyroid c. Ovary d. Kidney

d Kidney

43. After an ultrasound of the thyroid gland, which diagnostic test determines the need for surgical intervention for thyroid nodules? a. CT scan b. MRI c. Angiography d. Needle biopsy

d Needle biopsy

Ch.62 Question 11 of 23 A client with a possible adrenal gland tumor is admitted for testing and treatment. Which nursing action is most appropriate for the charge nurse to delegate to the nursing assistant? a. Assess skin turgor and mucous membranes for hydration status. b. Discuss the dietary restrictions needed for 24-hour urine testing. c. Plan ways to control the environment that will avoid stimulating the client. d. Remind the client to avoid drinking coffee and changing position suddenly.

d Remind the client to avoid drinking coffee and changing position suddenly. Drinking caffeinated beverages and changing position suddenly are not safe for a client with a potential adrenal gland tumor because of the effects of catecholamines. Reminding the client about previous instructions is an appropriate role for a nursing assistant who may observe the client doing potentially risky activities. Client assessment, client teaching, and environment planning are higher-level skills that require the experience and responsibility of the RN, and are not within the scope of practice of the nursing assistant.

1. Problems in the hypothalamus that change the function of the anterior pituitary gland result in which condition? a. Adenohypophysis b. Panhypopituitarism c. Primary pituitary dysfunction d. Secondary pituitary dysfunction

d Secondary pituitary dysfunction

Ch.61 Question 16 of 16 The nurse is reviewing the laboratory test results for a client admitted with a possible pituitary disorder. Which information has the most immediate implication for the client's care? a. Blood glucose 125 mg/dL b. Blood urea nitrogen (BUN) 40 mg/dL c. Serum potassium 5.2 mEq/L d. Serum sodium 110 mEq/L

d Serum sodium 110 mEq/L The normal range for serum sodium is 135 to 145 mEq/L; a result of 110 mEq/L is considered hyponatremia and is extremely dangerous. The client is at risk for increased intracranial pressure, seizures, and death. The RN must act rapidly because this situation requires immediate intervention. The normal range for fasting blood glucose is 60 to 110 mg/dL; 125 mg/dL is high, but is not considered dangerous. The normal range for BUN is 7 to 20 mg/dL; 40 mg/dL is high. An elevated BUN can be an indication of kidney failure, dehydration, fever, increased protein intake, and shock, so the client should have a creatinine drawn for a more complete picture of kidney function. The normal range for serum potassium is 3.5 to 5.2 mEq/L; 5.2 mEq/L is high normal.

Question 16 of 16 The nurse is reviewing the laboratory test results for a client admitted with a possible pituitary disorder. Which information has the most immediate implication for the client's care? a. Blood glucose 125 mg/dL b. Blood urea nitrogen (BUN) 40 mg/dL c. Serum potassium 5.2 mEq/L d. Serum sodium 110 mEq/L

d Serum sodium 110 mEq/L The normal range for serum sodium is 135 to 145 mEq/L; a result of 110 mEq/L is considered hyponatremia and is extremely dangerous. The client is at risk for increased intracranial pressure, seizures, and death. The RN must act rapidly because this situation requires immediate intervention. The normal range for fasting blood glucose is 60 to 110 mg/dL; 125 mg/dL is high, but is not considered dangerous. The normal range for BUN is 7 to 20 mg/dL; 40 mg/dL is high. An elevated BUN can be an indication of kidney failure, dehydration, fever, increased protein intake, and shock, so the client should have a creatinine drawn for a more complete picture of kidney function. The normal range for serum potassium is 3.5 to 5.2 mEq/L; 5.2 mEq/L is high normal.

Ch.62 Question 13 of 23 A client is referred to a home health agency after a transsphenoidal hypophysectomy. Which action does the RN case manager delegate to the home health aide who will see the client daily? a. Document symptoms of incisional infection or meningitis. b. Give over-the-counter laxatives if the client is constipated. c. Set up medications as prescribed for the day. d. Test any nasal drainage for the presence of glucose.

d Test any nasal drainage for the presence of glucose. Cerebrospinal fluid (CSF) will test positive using a glucose "dipstick." Nasal drainage that is positive for glucose after a transsphenoidal hypophysectomy would indicate a CSF leak that would require immediate notification of the health care provider. Home health aides can be taught the correct technique to perform this procedure. Assessing for symptoms of infection and documenting them in the record, medication administration, and setting up medication are not within the scope of practice of the home health aide.

Ch.62 Question 4 of 23 The nurse is caring for a client with hypercortisolism. The nurse begins to feel the onset of a cold but still has 4 hours left in the shift. What does the nurse do? a. Asks another nurse to care for the client b. Monitors the client for cold-like symptoms c. Refuses to care for the client d. Wears a facemask when caring for the client

d Wears a facemask when caring for the client A client with hypercortisolism will be immune-suppressed. Anyone with a suspected upper respiratory infection who must enter the client's room must wear a mask to prevent the spread of infection. Although asking another nurse to care for the client might be an option in some facilities, it is not generally realistic or practical. The nurse, not the client, feels the onset of the cold, so monitoring the client for cold-like symptoms is part of good client care for a client with hypercortisolism. Refusing to care for the client after starting care would be considered abandonment.

Ch.62 Question 14 of 23 A client with Cushing's disease says that she has lost 1 pound. What does the nurse do next? a. Auscultates the lungs for crackles b. Checks urine for specific gravity c. Forces fluids d. Weighs the client

d Weighs the client Fluid retention with weight gain is more of a problem than weight loss in clients with Cushing's disease. Weighing the client with Cushing's disease is part of the nurse's assessment. Crackles in the lungs indicate possible fluid retention, which would cause weight gain, not weight loss. Urine specific gravity will help assess hydration status, but this would not be the next step in the client's assessment. Forcing fluids is not appropriate because usually excess water and sodium reabsorption cause fluid retention in the client with Cushing's disease.

Ch.64 Question 5 of 25 A client recently admitted with new-onset type 2 diabetes will be discharged with a self-monitoring blood glucose machine. When is the best time for the nurse to explain to the client the proper use of the machine? a. Day of discharge b. On admission c. When the client states readiness d. While performing the test in the hospital

d While performing the test in the hospital Teaching the client about the operation of the machine while performing the test in the hospital is the best way for the client to learn. The teaching can be reinforced before discharge. Instructing the client on the day of admission or the day of discharge would be overwhelming to the client because of all of the other activities taking place on those days. The client may never feel ready to learn this daunting task; the nurse must be more proactive.

Ch.62 Question 5 of 23 How does the drug desmopressin (DDAVP) decrease urine output in a client with diabetes insipidus (DI)? a. Blocks reabsorption of sodium b. Increases blood pressure c. Increases cardiac output d. Works as an antidiuretic hormone (ADH) in the kidneys

d Works as an antidiuretic hormone (ADH) in the kidneys Desmopressin is a synthetic form of ADH that binds to kidney receptors and enhances reabsorption of water, thus reducing urine output. Desmopressin does not have any effect on sodium reabsorption. It may cause a slight increase or a transient decrease in blood pressure, but this does not affect urine output. Desmopressin does not increase cardiac output.

11. A deficiency of which anterior pituitary hormones is considered life-threatening? (Select all that apply.) a. GH b. Melanocyte-stimulating hormone (MSH) c. PRL d. Thyroid-stimulating hormone (TSH) e. ACTH

d, e Thyroid-stimulating hormone (TSH), ACTH

4. A nurse assesses a client on the medical-surgical unit. Which statement made by the client should alert the nurse to the possibility of hypothyroidism? a. "My sister has thyroid problems." b. "I seem to feel the heat more than other people." c. "Food just doesn't taste good without a lot of salt." d. "I am always tired, even with 12 hours of sleep."

d. Clients with hypothyroidism usually feel tired or weak despite getting many hours of sleep. Thyroid problems are not inherited. Heat intolerance is indicative of hyperthyroidism. Loss of taste is not a manifestation of hypothyroidism.


Related study sets

Chapter 12 Organizing your presentation

View Set

Understanding Food Chapter 14: Fruits

View Set

business essentials objective 2.01 and 2.02

View Set

Physics: Chapter 3 Projectile Motion

View Set

Cultural Anthropology Chapter 3: Economics and Exchange

View Set